LayerRx Mapping ID
281
Slot System
Featured Buckets
Featured Buckets Admin
Reverse Chronological Sort
Allow Teaser Image

Cardiorenal syndrome

Article Type
Changed
Mon, 05/21/2018 - 09:42
Display Headline
Cardiorenal syndrome

To the Editor: I read with interest the thoughtful review of cardiorenal syndrome by Drs. Thind, Loehrke, and Wilt1 and the accompanying editorial by Dr. Grodin.2 These articles certainly add to our growing knowledge of the syndrome and the importance of treating volume overload in these complex patients.

Indeed, we and others have stressed the primary importance of renal dysfunction in patients with volume overload and acute decompensated heart failure.3,4 We have learned that even small rises in serum creatinine predict poor outcomes in these patients. And even if the serum creatinine level comes back down during hospitalization, acute kidney injury (AKI) is still associated with risk.5

Nevertheless, clinicians remain frustrated with the practical management of patients with volume overload and worsening AKI. When faced with a rising serum creatinine level in a patient being treated for decompensated heart failure with signs or symptoms of volume overload, I suggest the following:

Perform careful bedside and chart review searching for evidence of AKI related to causes other than cardiorenal syndrome. Ask whether the rise in serum creatinine could be caused by new obstruction (eg, urinary retention, upper urinary tract obstruction), a nephrotoxin (eg, nonsteroidal anti-inflammatory drugs), a primary tubulointerstitial or glomerular process (eg, drug-induced acute interstitial nephritis, acute glomerulonephritis), acute tubular necrosis, or a new hemodynamic event threatening renal perfusion (eg, hypotension, a new arrhythmia). It is often best to arrive at a diagnosis of AKI due to cardiorenal dysfunction by exclusion, much like the working definitions of hepatorenal syndrome.6 This requires review of the urine sediment (looking for evidence of granular casts of acute tubular necrosis, or evidence of glomerulonephritis or interstitial nephritis), electronic medical record, vital signs, telemetry, and perhaps renal ultrasonography.

In the absence of frank evidence of “overdiuresis” such as worsening hypernatremia, with dropping blood pressure, clinical hypoperfusion, and contraction alkalosis, avoid the temptation to suspend diuretics. Alternatively, an increase in diuretic dose, or addition of a distal diuretic (ie, metolazone) may be needed to address persistent renal venous congestion as the cause of the AKI.3 In this situation, be sure to monitor electrolytes, volume status, and renal function closely while diuretic treatment is augmented. In many such cases, the serum creatinine may actually start to decrease after a more robust diuresis is generated. In these patients, it may also be prudent to temporarily suspend antagonists of the renin-angiotensin-aldosterone system, although this remains controversial.

Management of such patients should be done collaboratively with cardiologists well versed in the treatment of cardiorenal syndrome. It may be possible that the worsening renal function in these patients represents important changes in cardiac rhythm or function (eg, low cardiac output state, new or worsening valvular disease, ongoing myocardial ischemia, cardiac tamponade, uncontrolled bradycardia or tachyarrythmia). Interventions aimed at reversing such perturbations could be the most important steps in improving cardiorenal function and reversing AKI.

References
  1. Thind GS, Loehrke M, Wilt JL. Acute cardiorenal syndrome: mechanisms and clinical implications. Cleve Clin J Med 2018; 85(3):231–239. doi:10.3949/ccjm.85a.17019
  2. Grodin JL. Hemodynamically, the kidney is at the heart of cardiorenal syndrome. Cleve Clin J Med 2018; 85(3):240–242. doi:10.3949/ccjm.85a.17126
  3. Freda BJ, Slawsky M, Mallidi J, Braden GL. Decongestive treatment of acute decompensated heart failure: cardiorenal implications of ultrafiltration and diuretics. Am J Kid Dis 2011; 58(6):1005–1017. doi:10.1053/j.ajkd.2011.07.023
  4. Tang WH, Kitai T. Intrarenal blood flow: a window into the congestive kidney failure phenotype of heart failure? JACC Heart Fail 2016; 4(8):683–686. doi:10.1016/j.jchf.2016.05.009
  5. Freda BJ, Knee AB, Braden GL, Visintainer PF, Thakaer CV. Effect of transient and sustained acute kidney injury on readmissions in acute decompensated heart failure. Am J Cardiol 2017; 119(11):1809–1814. doi:10.1016/j.amjcard.2017.02.044
  6. Bucsics T, Krones E. Renal dysfunction in cirrhosis: acute kidney injury and the hepatorenal syndrome. Gastroenterol Rep (Oxf) 2017; 5(2):127–137. doi:10.1093/gastro/gox009
Article PDF
Author and Disclosure Information

Benjamin J. Freda, DO
Tufts University School of Medicine, Springfield, MA

Issue
Cleveland Clinic Journal of Medicine - 85(5)
Publications
Topics
Page Number
360-361
Legacy Keywords
cardiorenal syndrome, heart failure, acute kidney injury, AKI, volume overload, serum creatinine, diuretics, Benjamin Freda
Sections
Author and Disclosure Information

Benjamin J. Freda, DO
Tufts University School of Medicine, Springfield, MA

Author and Disclosure Information

Benjamin J. Freda, DO
Tufts University School of Medicine, Springfield, MA

Article PDF
Article PDF
Related Articles

To the Editor: I read with interest the thoughtful review of cardiorenal syndrome by Drs. Thind, Loehrke, and Wilt1 and the accompanying editorial by Dr. Grodin.2 These articles certainly add to our growing knowledge of the syndrome and the importance of treating volume overload in these complex patients.

Indeed, we and others have stressed the primary importance of renal dysfunction in patients with volume overload and acute decompensated heart failure.3,4 We have learned that even small rises in serum creatinine predict poor outcomes in these patients. And even if the serum creatinine level comes back down during hospitalization, acute kidney injury (AKI) is still associated with risk.5

Nevertheless, clinicians remain frustrated with the practical management of patients with volume overload and worsening AKI. When faced with a rising serum creatinine level in a patient being treated for decompensated heart failure with signs or symptoms of volume overload, I suggest the following:

Perform careful bedside and chart review searching for evidence of AKI related to causes other than cardiorenal syndrome. Ask whether the rise in serum creatinine could be caused by new obstruction (eg, urinary retention, upper urinary tract obstruction), a nephrotoxin (eg, nonsteroidal anti-inflammatory drugs), a primary tubulointerstitial or glomerular process (eg, drug-induced acute interstitial nephritis, acute glomerulonephritis), acute tubular necrosis, or a new hemodynamic event threatening renal perfusion (eg, hypotension, a new arrhythmia). It is often best to arrive at a diagnosis of AKI due to cardiorenal dysfunction by exclusion, much like the working definitions of hepatorenal syndrome.6 This requires review of the urine sediment (looking for evidence of granular casts of acute tubular necrosis, or evidence of glomerulonephritis or interstitial nephritis), electronic medical record, vital signs, telemetry, and perhaps renal ultrasonography.

In the absence of frank evidence of “overdiuresis” such as worsening hypernatremia, with dropping blood pressure, clinical hypoperfusion, and contraction alkalosis, avoid the temptation to suspend diuretics. Alternatively, an increase in diuretic dose, or addition of a distal diuretic (ie, metolazone) may be needed to address persistent renal venous congestion as the cause of the AKI.3 In this situation, be sure to monitor electrolytes, volume status, and renal function closely while diuretic treatment is augmented. In many such cases, the serum creatinine may actually start to decrease after a more robust diuresis is generated. In these patients, it may also be prudent to temporarily suspend antagonists of the renin-angiotensin-aldosterone system, although this remains controversial.

Management of such patients should be done collaboratively with cardiologists well versed in the treatment of cardiorenal syndrome. It may be possible that the worsening renal function in these patients represents important changes in cardiac rhythm or function (eg, low cardiac output state, new or worsening valvular disease, ongoing myocardial ischemia, cardiac tamponade, uncontrolled bradycardia or tachyarrythmia). Interventions aimed at reversing such perturbations could be the most important steps in improving cardiorenal function and reversing AKI.

To the Editor: I read with interest the thoughtful review of cardiorenal syndrome by Drs. Thind, Loehrke, and Wilt1 and the accompanying editorial by Dr. Grodin.2 These articles certainly add to our growing knowledge of the syndrome and the importance of treating volume overload in these complex patients.

Indeed, we and others have stressed the primary importance of renal dysfunction in patients with volume overload and acute decompensated heart failure.3,4 We have learned that even small rises in serum creatinine predict poor outcomes in these patients. And even if the serum creatinine level comes back down during hospitalization, acute kidney injury (AKI) is still associated with risk.5

Nevertheless, clinicians remain frustrated with the practical management of patients with volume overload and worsening AKI. When faced with a rising serum creatinine level in a patient being treated for decompensated heart failure with signs or symptoms of volume overload, I suggest the following:

Perform careful bedside and chart review searching for evidence of AKI related to causes other than cardiorenal syndrome. Ask whether the rise in serum creatinine could be caused by new obstruction (eg, urinary retention, upper urinary tract obstruction), a nephrotoxin (eg, nonsteroidal anti-inflammatory drugs), a primary tubulointerstitial or glomerular process (eg, drug-induced acute interstitial nephritis, acute glomerulonephritis), acute tubular necrosis, or a new hemodynamic event threatening renal perfusion (eg, hypotension, a new arrhythmia). It is often best to arrive at a diagnosis of AKI due to cardiorenal dysfunction by exclusion, much like the working definitions of hepatorenal syndrome.6 This requires review of the urine sediment (looking for evidence of granular casts of acute tubular necrosis, or evidence of glomerulonephritis or interstitial nephritis), electronic medical record, vital signs, telemetry, and perhaps renal ultrasonography.

In the absence of frank evidence of “overdiuresis” such as worsening hypernatremia, with dropping blood pressure, clinical hypoperfusion, and contraction alkalosis, avoid the temptation to suspend diuretics. Alternatively, an increase in diuretic dose, or addition of a distal diuretic (ie, metolazone) may be needed to address persistent renal venous congestion as the cause of the AKI.3 In this situation, be sure to monitor electrolytes, volume status, and renal function closely while diuretic treatment is augmented. In many such cases, the serum creatinine may actually start to decrease after a more robust diuresis is generated. In these patients, it may also be prudent to temporarily suspend antagonists of the renin-angiotensin-aldosterone system, although this remains controversial.

Management of such patients should be done collaboratively with cardiologists well versed in the treatment of cardiorenal syndrome. It may be possible that the worsening renal function in these patients represents important changes in cardiac rhythm or function (eg, low cardiac output state, new or worsening valvular disease, ongoing myocardial ischemia, cardiac tamponade, uncontrolled bradycardia or tachyarrythmia). Interventions aimed at reversing such perturbations could be the most important steps in improving cardiorenal function and reversing AKI.

References
  1. Thind GS, Loehrke M, Wilt JL. Acute cardiorenal syndrome: mechanisms and clinical implications. Cleve Clin J Med 2018; 85(3):231–239. doi:10.3949/ccjm.85a.17019
  2. Grodin JL. Hemodynamically, the kidney is at the heart of cardiorenal syndrome. Cleve Clin J Med 2018; 85(3):240–242. doi:10.3949/ccjm.85a.17126
  3. Freda BJ, Slawsky M, Mallidi J, Braden GL. Decongestive treatment of acute decompensated heart failure: cardiorenal implications of ultrafiltration and diuretics. Am J Kid Dis 2011; 58(6):1005–1017. doi:10.1053/j.ajkd.2011.07.023
  4. Tang WH, Kitai T. Intrarenal blood flow: a window into the congestive kidney failure phenotype of heart failure? JACC Heart Fail 2016; 4(8):683–686. doi:10.1016/j.jchf.2016.05.009
  5. Freda BJ, Knee AB, Braden GL, Visintainer PF, Thakaer CV. Effect of transient and sustained acute kidney injury on readmissions in acute decompensated heart failure. Am J Cardiol 2017; 119(11):1809–1814. doi:10.1016/j.amjcard.2017.02.044
  6. Bucsics T, Krones E. Renal dysfunction in cirrhosis: acute kidney injury and the hepatorenal syndrome. Gastroenterol Rep (Oxf) 2017; 5(2):127–137. doi:10.1093/gastro/gox009
References
  1. Thind GS, Loehrke M, Wilt JL. Acute cardiorenal syndrome: mechanisms and clinical implications. Cleve Clin J Med 2018; 85(3):231–239. doi:10.3949/ccjm.85a.17019
  2. Grodin JL. Hemodynamically, the kidney is at the heart of cardiorenal syndrome. Cleve Clin J Med 2018; 85(3):240–242. doi:10.3949/ccjm.85a.17126
  3. Freda BJ, Slawsky M, Mallidi J, Braden GL. Decongestive treatment of acute decompensated heart failure: cardiorenal implications of ultrafiltration and diuretics. Am J Kid Dis 2011; 58(6):1005–1017. doi:10.1053/j.ajkd.2011.07.023
  4. Tang WH, Kitai T. Intrarenal blood flow: a window into the congestive kidney failure phenotype of heart failure? JACC Heart Fail 2016; 4(8):683–686. doi:10.1016/j.jchf.2016.05.009
  5. Freda BJ, Knee AB, Braden GL, Visintainer PF, Thakaer CV. Effect of transient and sustained acute kidney injury on readmissions in acute decompensated heart failure. Am J Cardiol 2017; 119(11):1809–1814. doi:10.1016/j.amjcard.2017.02.044
  6. Bucsics T, Krones E. Renal dysfunction in cirrhosis: acute kidney injury and the hepatorenal syndrome. Gastroenterol Rep (Oxf) 2017; 5(2):127–137. doi:10.1093/gastro/gox009
Issue
Cleveland Clinic Journal of Medicine - 85(5)
Issue
Cleveland Clinic Journal of Medicine - 85(5)
Page Number
360-361
Page Number
360-361
Publications
Publications
Topics
Article Type
Display Headline
Cardiorenal syndrome
Display Headline
Cardiorenal syndrome
Legacy Keywords
cardiorenal syndrome, heart failure, acute kidney injury, AKI, volume overload, serum creatinine, diuretics, Benjamin Freda
Legacy Keywords
cardiorenal syndrome, heart failure, acute kidney injury, AKI, volume overload, serum creatinine, diuretics, Benjamin Freda
Sections
Disallow All Ads
Content Gating
No Gating (article Unlocked/Free)
Alternative CME
Disqus Comments
Default
Gate On Date
Thu, 04/26/2018 - 09:00
Un-Gate On Date
Thu, 04/26/2018 - 09:00
Use ProPublica
CFC Schedule Remove Status
Thu, 04/26/2018 - 09:00
Article PDF Media

In reply: Cardiorenal syndrome

Article Type
Changed
Mon, 05/21/2018 - 09:41
Display Headline
In reply: Cardiorenal syndrome

In Reply: We thank Dr. Freda for his remarks and observations. Certainly, the clinical importance of this entity and the challenge it poses to clinicians cannot be overemphasized. We concur with the overall message and reply to his specific comments:

We completely agree that clinical data-gathering is of paramount importance. This includes careful history-taking, physical examination, electronic medical record review, laboratory data review, and imaging. As discussed in our article, renal electrolytes will reveal a prerenal state in acute cardiorenal syndrome, and other causes of prerenal acute kidney injury (AKI) should be ruled out. The role of point-of-care ultrasonography (eg, to measure the size and respirophasic variation of the inferior vena cava) as a vital diagnostic tool has been well described, and we endorse it.1 Moreover, apart from snapshot values, trends are also very important. This is especially pertinent when the patient care is being transferred to a new service (eg, from hospitalist service to the critical care service). In this case, careful review of diuretic dosage, renal function trend, intake and output, and weight trend would help in the diagnosis.

Inadequate diuretic therapy is perhaps one of the most common errors made in the management of patients with acute cardiorenal syndrome. As mentioned in our article, diuretics should be correctly dosed based on the patient’s renal function. It is a common misconception that diuretics are nephrotoxic: in reality, there is no direct renal toxicity from the drug itself. Certainly, overdiuresis may lead to AKI, but this is not a valid concern in patients with acute cardiorenal syndrome, who are fluid-overloaded by definition.

Another challenging clinical scenario is when a patient is diagnosed with acute cardiorenal syndrome but renal function worsens with diuretic therapy. In our experience, this is a paradoxical situation and often stems from misinterpretation of clinical data. The most common example is diuretic underdosage leading to inadequate diuretic response. Renal function will continue to decline in these patients, as renal congestion has not yet been relieved. This reiterates the importance of paying close attention to urine output and intake-output data. When the diuretic regimen is strengthened and a robust diuretic response is achieved, renal function should improve as systemic congestion diminishes.

Acute cardiorenal syndrome stems from hemodynamic derangements, and a multidisciplinary approach may certainly lead to better outcomes. Although we described the general theme of hemodynamic disturbances, patients with acute cardiorenal syndrome may have certain unique and complex hemodynamic “phenotypes” that we did not discuss due to the limited scope of the paper. One such phenotype worth mentioning is decompensated right heart failure, as seen in patients with severe pulmonary hypertension. Acute cardiorenal syndrome due to renal congestion is often seen in these patients, but they also have certain other unique characteristics such as ventricular interdependence.2 Giving intravenous fluids to these patients not only will worsen renal function but can also cause catastrophic reduction in cardiac output and blood pressure due to worsening interventricular septal bowing. Certain treatments (eg, pulmonary vasodilators) are unique to this patient population, and these patients should hence be managed by experienced clinicians.

References
  1. Blehar DJ, Dickman E, Gaspari R. Identification of congestive heart failure via respiratory variation of inferior vena cava diameter. Am J Emerg Med 2009; 27(1):71–75. doi:10.1016/j.ajem.2008.01.002
  2. Piazza G, Goldhaber SZ. The acutely decompensated right ventricle: pathways for diagnosis and management. Chest 2005128(3):1836–1852. doi:10.1378/chest.128.3.1836
Article PDF
Author and Disclosure Information

Guramrinder S. Thind, MD
Western Michigan University School of Medicine, Kalamazoo

Mark Loehrke MD, FACP
Western Michigan University School of Medicine, Kalamazoo

Jeffrey L. Wilt, MD, FACP, FCCP
Western Michigan University School of Medicine, Kalamazoo

Issue
Cleveland Clinic Journal of Medicine - 85(5)
Publications
Topics
Page Number
360-361
Legacy Keywords
cardiorenal syndrome, heart failure, acute kidney injury, AKI, volume overload, serum creatinine, diuretics, Guramrinder Thind, Mark Loehrke, Jeffrey Wild
Sections
Author and Disclosure Information

Guramrinder S. Thind, MD
Western Michigan University School of Medicine, Kalamazoo

Mark Loehrke MD, FACP
Western Michigan University School of Medicine, Kalamazoo

Jeffrey L. Wilt, MD, FACP, FCCP
Western Michigan University School of Medicine, Kalamazoo

Author and Disclosure Information

Guramrinder S. Thind, MD
Western Michigan University School of Medicine, Kalamazoo

Mark Loehrke MD, FACP
Western Michigan University School of Medicine, Kalamazoo

Jeffrey L. Wilt, MD, FACP, FCCP
Western Michigan University School of Medicine, Kalamazoo

Article PDF
Article PDF
Related Articles

In Reply: We thank Dr. Freda for his remarks and observations. Certainly, the clinical importance of this entity and the challenge it poses to clinicians cannot be overemphasized. We concur with the overall message and reply to his specific comments:

We completely agree that clinical data-gathering is of paramount importance. This includes careful history-taking, physical examination, electronic medical record review, laboratory data review, and imaging. As discussed in our article, renal electrolytes will reveal a prerenal state in acute cardiorenal syndrome, and other causes of prerenal acute kidney injury (AKI) should be ruled out. The role of point-of-care ultrasonography (eg, to measure the size and respirophasic variation of the inferior vena cava) as a vital diagnostic tool has been well described, and we endorse it.1 Moreover, apart from snapshot values, trends are also very important. This is especially pertinent when the patient care is being transferred to a new service (eg, from hospitalist service to the critical care service). In this case, careful review of diuretic dosage, renal function trend, intake and output, and weight trend would help in the diagnosis.

Inadequate diuretic therapy is perhaps one of the most common errors made in the management of patients with acute cardiorenal syndrome. As mentioned in our article, diuretics should be correctly dosed based on the patient’s renal function. It is a common misconception that diuretics are nephrotoxic: in reality, there is no direct renal toxicity from the drug itself. Certainly, overdiuresis may lead to AKI, but this is not a valid concern in patients with acute cardiorenal syndrome, who are fluid-overloaded by definition.

Another challenging clinical scenario is when a patient is diagnosed with acute cardiorenal syndrome but renal function worsens with diuretic therapy. In our experience, this is a paradoxical situation and often stems from misinterpretation of clinical data. The most common example is diuretic underdosage leading to inadequate diuretic response. Renal function will continue to decline in these patients, as renal congestion has not yet been relieved. This reiterates the importance of paying close attention to urine output and intake-output data. When the diuretic regimen is strengthened and a robust diuretic response is achieved, renal function should improve as systemic congestion diminishes.

Acute cardiorenal syndrome stems from hemodynamic derangements, and a multidisciplinary approach may certainly lead to better outcomes. Although we described the general theme of hemodynamic disturbances, patients with acute cardiorenal syndrome may have certain unique and complex hemodynamic “phenotypes” that we did not discuss due to the limited scope of the paper. One such phenotype worth mentioning is decompensated right heart failure, as seen in patients with severe pulmonary hypertension. Acute cardiorenal syndrome due to renal congestion is often seen in these patients, but they also have certain other unique characteristics such as ventricular interdependence.2 Giving intravenous fluids to these patients not only will worsen renal function but can also cause catastrophic reduction in cardiac output and blood pressure due to worsening interventricular septal bowing. Certain treatments (eg, pulmonary vasodilators) are unique to this patient population, and these patients should hence be managed by experienced clinicians.

In Reply: We thank Dr. Freda for his remarks and observations. Certainly, the clinical importance of this entity and the challenge it poses to clinicians cannot be overemphasized. We concur with the overall message and reply to his specific comments:

We completely agree that clinical data-gathering is of paramount importance. This includes careful history-taking, physical examination, electronic medical record review, laboratory data review, and imaging. As discussed in our article, renal electrolytes will reveal a prerenal state in acute cardiorenal syndrome, and other causes of prerenal acute kidney injury (AKI) should be ruled out. The role of point-of-care ultrasonography (eg, to measure the size and respirophasic variation of the inferior vena cava) as a vital diagnostic tool has been well described, and we endorse it.1 Moreover, apart from snapshot values, trends are also very important. This is especially pertinent when the patient care is being transferred to a new service (eg, from hospitalist service to the critical care service). In this case, careful review of diuretic dosage, renal function trend, intake and output, and weight trend would help in the diagnosis.

Inadequate diuretic therapy is perhaps one of the most common errors made in the management of patients with acute cardiorenal syndrome. As mentioned in our article, diuretics should be correctly dosed based on the patient’s renal function. It is a common misconception that diuretics are nephrotoxic: in reality, there is no direct renal toxicity from the drug itself. Certainly, overdiuresis may lead to AKI, but this is not a valid concern in patients with acute cardiorenal syndrome, who are fluid-overloaded by definition.

Another challenging clinical scenario is when a patient is diagnosed with acute cardiorenal syndrome but renal function worsens with diuretic therapy. In our experience, this is a paradoxical situation and often stems from misinterpretation of clinical data. The most common example is diuretic underdosage leading to inadequate diuretic response. Renal function will continue to decline in these patients, as renal congestion has not yet been relieved. This reiterates the importance of paying close attention to urine output and intake-output data. When the diuretic regimen is strengthened and a robust diuretic response is achieved, renal function should improve as systemic congestion diminishes.

Acute cardiorenal syndrome stems from hemodynamic derangements, and a multidisciplinary approach may certainly lead to better outcomes. Although we described the general theme of hemodynamic disturbances, patients with acute cardiorenal syndrome may have certain unique and complex hemodynamic “phenotypes” that we did not discuss due to the limited scope of the paper. One such phenotype worth mentioning is decompensated right heart failure, as seen in patients with severe pulmonary hypertension. Acute cardiorenal syndrome due to renal congestion is often seen in these patients, but they also have certain other unique characteristics such as ventricular interdependence.2 Giving intravenous fluids to these patients not only will worsen renal function but can also cause catastrophic reduction in cardiac output and blood pressure due to worsening interventricular septal bowing. Certain treatments (eg, pulmonary vasodilators) are unique to this patient population, and these patients should hence be managed by experienced clinicians.

References
  1. Blehar DJ, Dickman E, Gaspari R. Identification of congestive heart failure via respiratory variation of inferior vena cava diameter. Am J Emerg Med 2009; 27(1):71–75. doi:10.1016/j.ajem.2008.01.002
  2. Piazza G, Goldhaber SZ. The acutely decompensated right ventricle: pathways for diagnosis and management. Chest 2005128(3):1836–1852. doi:10.1378/chest.128.3.1836
References
  1. Blehar DJ, Dickman E, Gaspari R. Identification of congestive heart failure via respiratory variation of inferior vena cava diameter. Am J Emerg Med 2009; 27(1):71–75. doi:10.1016/j.ajem.2008.01.002
  2. Piazza G, Goldhaber SZ. The acutely decompensated right ventricle: pathways for diagnosis and management. Chest 2005128(3):1836–1852. doi:10.1378/chest.128.3.1836
Issue
Cleveland Clinic Journal of Medicine - 85(5)
Issue
Cleveland Clinic Journal of Medicine - 85(5)
Page Number
360-361
Page Number
360-361
Publications
Publications
Topics
Article Type
Display Headline
In reply: Cardiorenal syndrome
Display Headline
In reply: Cardiorenal syndrome
Legacy Keywords
cardiorenal syndrome, heart failure, acute kidney injury, AKI, volume overload, serum creatinine, diuretics, Guramrinder Thind, Mark Loehrke, Jeffrey Wild
Legacy Keywords
cardiorenal syndrome, heart failure, acute kidney injury, AKI, volume overload, serum creatinine, diuretics, Guramrinder Thind, Mark Loehrke, Jeffrey Wild
Sections
Disallow All Ads
Content Gating
No Gating (article Unlocked/Free)
Alternative CME
Disqus Comments
Default
Gate On Date
Thu, 04/26/2018 - 10:15
Un-Gate On Date
Thu, 04/26/2018 - 10:15
Use ProPublica
CFC Schedule Remove Status
Thu, 04/26/2018 - 10:15
Article PDF Media

Rapid Development of Life-Threatening Emamectin Benzoate Poisoning

Article Type
Changed
Wed, 12/12/2018 - 21:10
A 75-year-old man presented to the ED after he intentionally ingested a 200-mL bottle of an agricultural insecticide.

Emamectin benzoate (EB) is a semisynthetic derivative of avermectin that has acaricidal, nematicidal, and insecticidal action. Avermectin analogs are natural products from soil fungi (Streptomyces avermitilis).1 Emamectin benzoate was initially developed to eradicate lepidopteran larvae, particularly armyworms, and is registered in the United States and Japan for use on vegetable crops.2-4 In addition to its agricultural use, EB also has antiparasitic effects on sea lice (Lepeophtheirus salmonis) that affect Atlantic salmon, and has been registered for use in several countries since 1999.5-7 Although a few studies have evaluated the toxic effects of avermectin on humans, there is a paucity of information regarding human toxicity associated with EB.7 This case report describes rapid deterioration of a patient following ingestion of EB.

Case

A 75-year-old man presented to the ED 20 minutes after intentionally ingesting an agricultural insecticide. Upon presentation, the patient stated that he drank a whole bottle (100 mL) of insecticide after consuming alcohol, but denied coingestion of other toxic substances or any medications. The patient provided the empty bottle upon presentation, and the ingested product was identified as Affirm, an insecticide containing 2.15% EB as the active ingredient.

The patient’s medical history was significant for major depressive disorder, for which he was on alprazolam, donepezil, paroxetine, and quetiapine. The patient stated that he also suffered from chronic back pain, noting that he only took analgesics intermittently as needed.

On examination, the patient was alert and oriented to time and place. Initially, he did not experience any physical discomfort. His vital signs were: blood pressure (BP), 126/74 mm Hg; pulse rate, 67 beats/minute; respiratory rate, mildly tachypneic at 23 breaths/minute; and temperature, 97.9°F. Oxygen saturation was 96% on room air.

Ocular examination revealed both pupils to be equally round, 3 mm in diameter, and reactive to light. Examination of the oropharynx was normal and without signs of mucosal injury. The lung sounds were clear bilaterally, and the heart was a regular rate and rhythm and without murmur. The patient’s abdomen was soft and nontender. No deficits, such as ataxia, dysarthria, or tremor were found on the neurological examination.

Prompt gastric lavage via a nasogastric tube was performed, and activated charcoal was administered. Laboratory evaluation was significant for the following: white blood cell count, 22.77 x 109/L with 78% neutrophils and 16% lymphocytes; sodium, 138 mEq/L; potassium, 3.1 mEq/L; chloride, 109 mEq/L; blood urea nitrogen, 19 mg/dL; and creatinine, 0.7 mg/dL. Arterial blood gas (ABG) results revealed a pH, 7.37; partial pressure of carbon dioxide, 25 mm Hg; partial pressure of oxygen, 93 mm Hg; bicarbonate, 14.5 mEq/L; base excess, –8.9 mEq/L; and an oxygen saturation, 97%. Serum creatine kinase (CK), CK-MB and troponin levels were both within normal range. Lactic acid, serum osmolality, and serum ethanol levels were not obtained. The patient’s electrocardiogram (ECG) and chest radiograph findings were normal.

Approximately 1 hour after presentation, the patient complained of an epigastric burning sensation and continued to exhibit mild tachypnea. A subsequent ABG test revealed progressive metabolic acidosis (Table). Although the patient was given a total of 800 mL of normal saline intravenously (IV) upon arrival at the ED, his total urinary output was less than 100 mL 7 hours afterward. Attempts to increase urinary output with IV furosemide were ineffective.

Table. Sequential Result of Arterial Blood Gas Analysis


Along with the progressive metabolic acidosis, the patient became hypotensive, and did not respond to IV fluid resuscitation. A norepinephrine infusion was started to improve BP, but this was likewise ineffective. Serial ECGs did not show any specific abnormalities such as dysrhythmia or ischemia.

The patient was admitted to the intensive care unit approximately 10.5 hours after presentation where he received continuous renal replacement therapy (CRRT) to correct the severe metabolic acidosis and poor circulation. Metabolic acidosis persisted despite CRRT, and the patient remained hypotensive even after receiving high-dose IV norepinephrine (Figure).
Figure. Chart detailing downward trend in the patient's systolic blood pressure (SBP) and mean arterial pressure (MAP) despite continuous renal replacement therapy and high-dose norepinephrine.


About 12.5 hours after his presentation to the ED, the patient began to vomit profusely and went into cardiac arrest. The cardiac monitor demonstrated pulseless ventricular tachycardia. Aggressive resuscitative efforts were initiated, but failed to restore spontaneous circulation.

Discussion

As an avermectin analog, EB interacts with γ-aminobutyric acid (GABA) receptors and enhances membrane chloride permeability.8 In mammals, GABA-containing neurons and receptors are found in the central nervous system (CNS), but not in the peripheral nervous system. In cases of high-dose avermectin ingestion in humans, CNS toxicity, including agitation and depressed mental status, have been reported, as well as death resulting from respiratory failure.9

 

 

With respect to human EB toxicity, there is only one other documented case in the literature by Yen and Lin.7 In their case, the authors report on a patient who ingested 100 mL of Proclaim, which contained 2.15% EB diluted with 400 mL of tap water.7 They note that the patient in their case presented with mild confusion and gastrointestinal (GI) symptoms of nausea, vomiting, and cramping discomfort. Following laboratory and radiological investigation, the patient was found to have aspiration pneumonitis and admitted to the inpatient hospital. On hospital day 2, the patient’s GI symptoms abated and he became alert and oriented. He was discharged 1 week from initial presentation and experienced no sequelae.

In our case, the patient ingested 100 mL of 2.15% EB without dilution. He also experienced GI symptoms, but did not have any CNS depression. The metabolic acidosis rapidly worsened, and could not be corrected, even with intensive therapy. This rapid life-threatening course has not previously been reported with avermectin or EB poisoning. In the avermectin poisoning cases in the literature, seven out of 19 patients (37%) exhibited severe effects, such as hypotension, coma, and aspiration with respiratory failure.9 Six of the seven patients experienced a full recovery; the remaining patient died 18 days after ingestion from multiple organ failure.

The reason for our patient’s rapid progression to metabolic acidosis and progressive deterioration (hypotension and hypoxemia) is not clear. One possible theory is that the solvents or other additives aside from EB in the ingested insecticide might make EB more toxic. In our case, the patient’s rapid deterioration alone or asphyxia by vomitus might have been the cause of the cardiac arrest. Future reports and studies about EB toxicity in humans are warranted to investigate the pathogenesis of toxicity and appropriate treatment.

Conclusion

This is the first report of a human death caused by EB poisoning; the patient experienced severe metabolic acidosis without CNS depression, ultimately leading to death. Emergency physicians should be aware of the possibility of rapid deterioration in patients who present after ingestion of EB and related substances.

References

1. Lasota JA, Dybas RA. Avermectins, a novel class of compounds: implications for use in arthropod pest control. Annu Rev Entomol. 1991;36:91-117. doi:10.1146/annurev.en.36.010191.000515.

2. Kuo JN, Buday C, van Aggelen G, Ikonomou MG, Pasternak J. Acute toxicity of emamectin benzoate and its desmethyl metabolite to Eohaustorius estuarius. Environ Toxicol Chem. 2010;29(8):1816-1820. doi:10.1002/etc.209.

3. Takai K, Soejima T, Suzuki T, et al. Development of a water-soluble preparation of emamectin benzoate and its preventative effect against the wilting of pot-grown pine trees inoculated with the pine wood nematode, Bursaphelenchus xylophilus. Pest Manag Sci. 2001;57(5):463-466. doi:10.1002/ps.30.

4. Chukwudebe AC, Beavers JB, Jaber M, Wislocki PG. Toxicity of emamectin benzoate to mallard duck and northern bobwhite quail. Environ Toxicol and Chem. 1998;17(6):1118-1123. doi:10.1002/etc.5620170619.

5. Armstrong R, MacPhee D, Katz T, Endris R. A field efficacy evaluation of emamectin benzoate for the control of sea lice on Atlantic salmon. Can Vet J. 2000;41(8):607-612.

6. Ramstad A, Colquhoun DJ, Nordmo R, Sutherland IH, Simmons R. Field trials in Norway with SLICE (0.2% emamectin benzoate) for the oral treatment of sea lice infestation in farmed Atlantic salmon Salmo salar. Dis Aquat Organ. 2002;21;50(1):29-33. doi:10.3354/dao050029.

7. Yen TH, Lin JL. Acute poisoning with emamectin benzoate. J Toxicol Clin Toxicol. 2004;42(5):657-661.

8. Campbell WC, Fisher MH, Stapley EO, Albers-Schönberg G, Jacob TA. Ivermectin: a potent new antiparasitic agent. Science. 1983; 221(4613):823-838.

9. Chung K, Yang CC, Wu ML, Deng JF, Tsai WJ. Agricultural avermectins: an uncommon but potentially fatal cause of pesticide poisoning. Ann Emerg Med. 1999;34(1):51-57.

Article PDF
Author and Disclosure Information

Dr Park is an associate professor, department of emergency medicine, Inje University Ilsan Paik Hospital, Gyeonggi-do, Korea

Author’s Disclosure Statement: The author reports no actual or potential conflict of interest in relation to this article.

Issue
Emergency Medicine - 50(4)
Publications
Topics
Page Number
81-84
Sections
Author and Disclosure Information

Dr Park is an associate professor, department of emergency medicine, Inje University Ilsan Paik Hospital, Gyeonggi-do, Korea

Author’s Disclosure Statement: The author reports no actual or potential conflict of interest in relation to this article.

Author and Disclosure Information

Dr Park is an associate professor, department of emergency medicine, Inje University Ilsan Paik Hospital, Gyeonggi-do, Korea

Author’s Disclosure Statement: The author reports no actual or potential conflict of interest in relation to this article.

Article PDF
Article PDF
A 75-year-old man presented to the ED after he intentionally ingested a 200-mL bottle of an agricultural insecticide.
A 75-year-old man presented to the ED after he intentionally ingested a 200-mL bottle of an agricultural insecticide.

Emamectin benzoate (EB) is a semisynthetic derivative of avermectin that has acaricidal, nematicidal, and insecticidal action. Avermectin analogs are natural products from soil fungi (Streptomyces avermitilis).1 Emamectin benzoate was initially developed to eradicate lepidopteran larvae, particularly armyworms, and is registered in the United States and Japan for use on vegetable crops.2-4 In addition to its agricultural use, EB also has antiparasitic effects on sea lice (Lepeophtheirus salmonis) that affect Atlantic salmon, and has been registered for use in several countries since 1999.5-7 Although a few studies have evaluated the toxic effects of avermectin on humans, there is a paucity of information regarding human toxicity associated with EB.7 This case report describes rapid deterioration of a patient following ingestion of EB.

Case

A 75-year-old man presented to the ED 20 minutes after intentionally ingesting an agricultural insecticide. Upon presentation, the patient stated that he drank a whole bottle (100 mL) of insecticide after consuming alcohol, but denied coingestion of other toxic substances or any medications. The patient provided the empty bottle upon presentation, and the ingested product was identified as Affirm, an insecticide containing 2.15% EB as the active ingredient.

The patient’s medical history was significant for major depressive disorder, for which he was on alprazolam, donepezil, paroxetine, and quetiapine. The patient stated that he also suffered from chronic back pain, noting that he only took analgesics intermittently as needed.

On examination, the patient was alert and oriented to time and place. Initially, he did not experience any physical discomfort. His vital signs were: blood pressure (BP), 126/74 mm Hg; pulse rate, 67 beats/minute; respiratory rate, mildly tachypneic at 23 breaths/minute; and temperature, 97.9°F. Oxygen saturation was 96% on room air.

Ocular examination revealed both pupils to be equally round, 3 mm in diameter, and reactive to light. Examination of the oropharynx was normal and without signs of mucosal injury. The lung sounds were clear bilaterally, and the heart was a regular rate and rhythm and without murmur. The patient’s abdomen was soft and nontender. No deficits, such as ataxia, dysarthria, or tremor were found on the neurological examination.

Prompt gastric lavage via a nasogastric tube was performed, and activated charcoal was administered. Laboratory evaluation was significant for the following: white blood cell count, 22.77 x 109/L with 78% neutrophils and 16% lymphocytes; sodium, 138 mEq/L; potassium, 3.1 mEq/L; chloride, 109 mEq/L; blood urea nitrogen, 19 mg/dL; and creatinine, 0.7 mg/dL. Arterial blood gas (ABG) results revealed a pH, 7.37; partial pressure of carbon dioxide, 25 mm Hg; partial pressure of oxygen, 93 mm Hg; bicarbonate, 14.5 mEq/L; base excess, –8.9 mEq/L; and an oxygen saturation, 97%. Serum creatine kinase (CK), CK-MB and troponin levels were both within normal range. Lactic acid, serum osmolality, and serum ethanol levels were not obtained. The patient’s electrocardiogram (ECG) and chest radiograph findings were normal.

Approximately 1 hour after presentation, the patient complained of an epigastric burning sensation and continued to exhibit mild tachypnea. A subsequent ABG test revealed progressive metabolic acidosis (Table). Although the patient was given a total of 800 mL of normal saline intravenously (IV) upon arrival at the ED, his total urinary output was less than 100 mL 7 hours afterward. Attempts to increase urinary output with IV furosemide were ineffective.

Table. Sequential Result of Arterial Blood Gas Analysis


Along with the progressive metabolic acidosis, the patient became hypotensive, and did not respond to IV fluid resuscitation. A norepinephrine infusion was started to improve BP, but this was likewise ineffective. Serial ECGs did not show any specific abnormalities such as dysrhythmia or ischemia.

The patient was admitted to the intensive care unit approximately 10.5 hours after presentation where he received continuous renal replacement therapy (CRRT) to correct the severe metabolic acidosis and poor circulation. Metabolic acidosis persisted despite CRRT, and the patient remained hypotensive even after receiving high-dose IV norepinephrine (Figure).
Figure. Chart detailing downward trend in the patient's systolic blood pressure (SBP) and mean arterial pressure (MAP) despite continuous renal replacement therapy and high-dose norepinephrine.


About 12.5 hours after his presentation to the ED, the patient began to vomit profusely and went into cardiac arrest. The cardiac monitor demonstrated pulseless ventricular tachycardia. Aggressive resuscitative efforts were initiated, but failed to restore spontaneous circulation.

Discussion

As an avermectin analog, EB interacts with γ-aminobutyric acid (GABA) receptors and enhances membrane chloride permeability.8 In mammals, GABA-containing neurons and receptors are found in the central nervous system (CNS), but not in the peripheral nervous system. In cases of high-dose avermectin ingestion in humans, CNS toxicity, including agitation and depressed mental status, have been reported, as well as death resulting from respiratory failure.9

 

 

With respect to human EB toxicity, there is only one other documented case in the literature by Yen and Lin.7 In their case, the authors report on a patient who ingested 100 mL of Proclaim, which contained 2.15% EB diluted with 400 mL of tap water.7 They note that the patient in their case presented with mild confusion and gastrointestinal (GI) symptoms of nausea, vomiting, and cramping discomfort. Following laboratory and radiological investigation, the patient was found to have aspiration pneumonitis and admitted to the inpatient hospital. On hospital day 2, the patient’s GI symptoms abated and he became alert and oriented. He was discharged 1 week from initial presentation and experienced no sequelae.

In our case, the patient ingested 100 mL of 2.15% EB without dilution. He also experienced GI symptoms, but did not have any CNS depression. The metabolic acidosis rapidly worsened, and could not be corrected, even with intensive therapy. This rapid life-threatening course has not previously been reported with avermectin or EB poisoning. In the avermectin poisoning cases in the literature, seven out of 19 patients (37%) exhibited severe effects, such as hypotension, coma, and aspiration with respiratory failure.9 Six of the seven patients experienced a full recovery; the remaining patient died 18 days after ingestion from multiple organ failure.

The reason for our patient’s rapid progression to metabolic acidosis and progressive deterioration (hypotension and hypoxemia) is not clear. One possible theory is that the solvents or other additives aside from EB in the ingested insecticide might make EB more toxic. In our case, the patient’s rapid deterioration alone or asphyxia by vomitus might have been the cause of the cardiac arrest. Future reports and studies about EB toxicity in humans are warranted to investigate the pathogenesis of toxicity and appropriate treatment.

Conclusion

This is the first report of a human death caused by EB poisoning; the patient experienced severe metabolic acidosis without CNS depression, ultimately leading to death. Emergency physicians should be aware of the possibility of rapid deterioration in patients who present after ingestion of EB and related substances.

Emamectin benzoate (EB) is a semisynthetic derivative of avermectin that has acaricidal, nematicidal, and insecticidal action. Avermectin analogs are natural products from soil fungi (Streptomyces avermitilis).1 Emamectin benzoate was initially developed to eradicate lepidopteran larvae, particularly armyworms, and is registered in the United States and Japan for use on vegetable crops.2-4 In addition to its agricultural use, EB also has antiparasitic effects on sea lice (Lepeophtheirus salmonis) that affect Atlantic salmon, and has been registered for use in several countries since 1999.5-7 Although a few studies have evaluated the toxic effects of avermectin on humans, there is a paucity of information regarding human toxicity associated with EB.7 This case report describes rapid deterioration of a patient following ingestion of EB.

Case

A 75-year-old man presented to the ED 20 minutes after intentionally ingesting an agricultural insecticide. Upon presentation, the patient stated that he drank a whole bottle (100 mL) of insecticide after consuming alcohol, but denied coingestion of other toxic substances or any medications. The patient provided the empty bottle upon presentation, and the ingested product was identified as Affirm, an insecticide containing 2.15% EB as the active ingredient.

The patient’s medical history was significant for major depressive disorder, for which he was on alprazolam, donepezil, paroxetine, and quetiapine. The patient stated that he also suffered from chronic back pain, noting that he only took analgesics intermittently as needed.

On examination, the patient was alert and oriented to time and place. Initially, he did not experience any physical discomfort. His vital signs were: blood pressure (BP), 126/74 mm Hg; pulse rate, 67 beats/minute; respiratory rate, mildly tachypneic at 23 breaths/minute; and temperature, 97.9°F. Oxygen saturation was 96% on room air.

Ocular examination revealed both pupils to be equally round, 3 mm in diameter, and reactive to light. Examination of the oropharynx was normal and without signs of mucosal injury. The lung sounds were clear bilaterally, and the heart was a regular rate and rhythm and without murmur. The patient’s abdomen was soft and nontender. No deficits, such as ataxia, dysarthria, or tremor were found on the neurological examination.

Prompt gastric lavage via a nasogastric tube was performed, and activated charcoal was administered. Laboratory evaluation was significant for the following: white blood cell count, 22.77 x 109/L with 78% neutrophils and 16% lymphocytes; sodium, 138 mEq/L; potassium, 3.1 mEq/L; chloride, 109 mEq/L; blood urea nitrogen, 19 mg/dL; and creatinine, 0.7 mg/dL. Arterial blood gas (ABG) results revealed a pH, 7.37; partial pressure of carbon dioxide, 25 mm Hg; partial pressure of oxygen, 93 mm Hg; bicarbonate, 14.5 mEq/L; base excess, –8.9 mEq/L; and an oxygen saturation, 97%. Serum creatine kinase (CK), CK-MB and troponin levels were both within normal range. Lactic acid, serum osmolality, and serum ethanol levels were not obtained. The patient’s electrocardiogram (ECG) and chest radiograph findings were normal.

Approximately 1 hour after presentation, the patient complained of an epigastric burning sensation and continued to exhibit mild tachypnea. A subsequent ABG test revealed progressive metabolic acidosis (Table). Although the patient was given a total of 800 mL of normal saline intravenously (IV) upon arrival at the ED, his total urinary output was less than 100 mL 7 hours afterward. Attempts to increase urinary output with IV furosemide were ineffective.

Table. Sequential Result of Arterial Blood Gas Analysis


Along with the progressive metabolic acidosis, the patient became hypotensive, and did not respond to IV fluid resuscitation. A norepinephrine infusion was started to improve BP, but this was likewise ineffective. Serial ECGs did not show any specific abnormalities such as dysrhythmia or ischemia.

The patient was admitted to the intensive care unit approximately 10.5 hours after presentation where he received continuous renal replacement therapy (CRRT) to correct the severe metabolic acidosis and poor circulation. Metabolic acidosis persisted despite CRRT, and the patient remained hypotensive even after receiving high-dose IV norepinephrine (Figure).
Figure. Chart detailing downward trend in the patient's systolic blood pressure (SBP) and mean arterial pressure (MAP) despite continuous renal replacement therapy and high-dose norepinephrine.


About 12.5 hours after his presentation to the ED, the patient began to vomit profusely and went into cardiac arrest. The cardiac monitor demonstrated pulseless ventricular tachycardia. Aggressive resuscitative efforts were initiated, but failed to restore spontaneous circulation.

Discussion

As an avermectin analog, EB interacts with γ-aminobutyric acid (GABA) receptors and enhances membrane chloride permeability.8 In mammals, GABA-containing neurons and receptors are found in the central nervous system (CNS), but not in the peripheral nervous system. In cases of high-dose avermectin ingestion in humans, CNS toxicity, including agitation and depressed mental status, have been reported, as well as death resulting from respiratory failure.9

 

 

With respect to human EB toxicity, there is only one other documented case in the literature by Yen and Lin.7 In their case, the authors report on a patient who ingested 100 mL of Proclaim, which contained 2.15% EB diluted with 400 mL of tap water.7 They note that the patient in their case presented with mild confusion and gastrointestinal (GI) symptoms of nausea, vomiting, and cramping discomfort. Following laboratory and radiological investigation, the patient was found to have aspiration pneumonitis and admitted to the inpatient hospital. On hospital day 2, the patient’s GI symptoms abated and he became alert and oriented. He was discharged 1 week from initial presentation and experienced no sequelae.

In our case, the patient ingested 100 mL of 2.15% EB without dilution. He also experienced GI symptoms, but did not have any CNS depression. The metabolic acidosis rapidly worsened, and could not be corrected, even with intensive therapy. This rapid life-threatening course has not previously been reported with avermectin or EB poisoning. In the avermectin poisoning cases in the literature, seven out of 19 patients (37%) exhibited severe effects, such as hypotension, coma, and aspiration with respiratory failure.9 Six of the seven patients experienced a full recovery; the remaining patient died 18 days after ingestion from multiple organ failure.

The reason for our patient’s rapid progression to metabolic acidosis and progressive deterioration (hypotension and hypoxemia) is not clear. One possible theory is that the solvents or other additives aside from EB in the ingested insecticide might make EB more toxic. In our case, the patient’s rapid deterioration alone or asphyxia by vomitus might have been the cause of the cardiac arrest. Future reports and studies about EB toxicity in humans are warranted to investigate the pathogenesis of toxicity and appropriate treatment.

Conclusion

This is the first report of a human death caused by EB poisoning; the patient experienced severe metabolic acidosis without CNS depression, ultimately leading to death. Emergency physicians should be aware of the possibility of rapid deterioration in patients who present after ingestion of EB and related substances.

References

1. Lasota JA, Dybas RA. Avermectins, a novel class of compounds: implications for use in arthropod pest control. Annu Rev Entomol. 1991;36:91-117. doi:10.1146/annurev.en.36.010191.000515.

2. Kuo JN, Buday C, van Aggelen G, Ikonomou MG, Pasternak J. Acute toxicity of emamectin benzoate and its desmethyl metabolite to Eohaustorius estuarius. Environ Toxicol Chem. 2010;29(8):1816-1820. doi:10.1002/etc.209.

3. Takai K, Soejima T, Suzuki T, et al. Development of a water-soluble preparation of emamectin benzoate and its preventative effect against the wilting of pot-grown pine trees inoculated with the pine wood nematode, Bursaphelenchus xylophilus. Pest Manag Sci. 2001;57(5):463-466. doi:10.1002/ps.30.

4. Chukwudebe AC, Beavers JB, Jaber M, Wislocki PG. Toxicity of emamectin benzoate to mallard duck and northern bobwhite quail. Environ Toxicol and Chem. 1998;17(6):1118-1123. doi:10.1002/etc.5620170619.

5. Armstrong R, MacPhee D, Katz T, Endris R. A field efficacy evaluation of emamectin benzoate for the control of sea lice on Atlantic salmon. Can Vet J. 2000;41(8):607-612.

6. Ramstad A, Colquhoun DJ, Nordmo R, Sutherland IH, Simmons R. Field trials in Norway with SLICE (0.2% emamectin benzoate) for the oral treatment of sea lice infestation in farmed Atlantic salmon Salmo salar. Dis Aquat Organ. 2002;21;50(1):29-33. doi:10.3354/dao050029.

7. Yen TH, Lin JL. Acute poisoning with emamectin benzoate. J Toxicol Clin Toxicol. 2004;42(5):657-661.

8. Campbell WC, Fisher MH, Stapley EO, Albers-Schönberg G, Jacob TA. Ivermectin: a potent new antiparasitic agent. Science. 1983; 221(4613):823-838.

9. Chung K, Yang CC, Wu ML, Deng JF, Tsai WJ. Agricultural avermectins: an uncommon but potentially fatal cause of pesticide poisoning. Ann Emerg Med. 1999;34(1):51-57.

References

1. Lasota JA, Dybas RA. Avermectins, a novel class of compounds: implications for use in arthropod pest control. Annu Rev Entomol. 1991;36:91-117. doi:10.1146/annurev.en.36.010191.000515.

2. Kuo JN, Buday C, van Aggelen G, Ikonomou MG, Pasternak J. Acute toxicity of emamectin benzoate and its desmethyl metabolite to Eohaustorius estuarius. Environ Toxicol Chem. 2010;29(8):1816-1820. doi:10.1002/etc.209.

3. Takai K, Soejima T, Suzuki T, et al. Development of a water-soluble preparation of emamectin benzoate and its preventative effect against the wilting of pot-grown pine trees inoculated with the pine wood nematode, Bursaphelenchus xylophilus. Pest Manag Sci. 2001;57(5):463-466. doi:10.1002/ps.30.

4. Chukwudebe AC, Beavers JB, Jaber M, Wislocki PG. Toxicity of emamectin benzoate to mallard duck and northern bobwhite quail. Environ Toxicol and Chem. 1998;17(6):1118-1123. doi:10.1002/etc.5620170619.

5. Armstrong R, MacPhee D, Katz T, Endris R. A field efficacy evaluation of emamectin benzoate for the control of sea lice on Atlantic salmon. Can Vet J. 2000;41(8):607-612.

6. Ramstad A, Colquhoun DJ, Nordmo R, Sutherland IH, Simmons R. Field trials in Norway with SLICE (0.2% emamectin benzoate) for the oral treatment of sea lice infestation in farmed Atlantic salmon Salmo salar. Dis Aquat Organ. 2002;21;50(1):29-33. doi:10.3354/dao050029.

7. Yen TH, Lin JL. Acute poisoning with emamectin benzoate. J Toxicol Clin Toxicol. 2004;42(5):657-661.

8. Campbell WC, Fisher MH, Stapley EO, Albers-Schönberg G, Jacob TA. Ivermectin: a potent new antiparasitic agent. Science. 1983; 221(4613):823-838.

9. Chung K, Yang CC, Wu ML, Deng JF, Tsai WJ. Agricultural avermectins: an uncommon but potentially fatal cause of pesticide poisoning. Ann Emerg Med. 1999;34(1):51-57.

Issue
Emergency Medicine - 50(4)
Issue
Emergency Medicine - 50(4)
Page Number
81-84
Page Number
81-84
Publications
Publications
Topics
Article Type
Sections
Disallow All Ads
Content Gating
No Gating (article Unlocked/Free)
Alternative CME
Disqus Comments
Default
Use ProPublica
Article PDF Media

From Local Bar to Police Car

Article Type
Changed
Mon, 07/09/2018 - 10:49
Display Headline
From Local Bar to Police Car

ANSWER

The radiograph demonstrates no acute injury to the wrist. There is, however, a subtle nondisplaced fracture of the distal fifth metacarpal joint. The patient was given a wrist splint for symptomatic relief, and orthopedic follow-up was coordinated.

Article PDF
Author and Disclosure Information

Nandan R. Hichkad, PA-C, MMSc, practices at the Georgia Neurosurgical Institute in Macon and is a clinical instructor at the Mercer University School of Medicine, Macon.

Issue
Clinician Reviews - 28(4)
Publications
Topics
Page Number
8,12
Sections
Author and Disclosure Information

Nandan R. Hichkad, PA-C, MMSc, practices at the Georgia Neurosurgical Institute in Macon and is a clinical instructor at the Mercer University School of Medicine, Macon.

Author and Disclosure Information

Nandan R. Hichkad, PA-C, MMSc, practices at the Georgia Neurosurgical Institute in Macon and is a clinical instructor at the Mercer University School of Medicine, Macon.

Article PDF
Article PDF

ANSWER

The radiograph demonstrates no acute injury to the wrist. There is, however, a subtle nondisplaced fracture of the distal fifth metacarpal joint. The patient was given a wrist splint for symptomatic relief, and orthopedic follow-up was coordinated.

ANSWER

The radiograph demonstrates no acute injury to the wrist. There is, however, a subtle nondisplaced fracture of the distal fifth metacarpal joint. The patient was given a wrist splint for symptomatic relief, and orthopedic follow-up was coordinated.

Issue
Clinician Reviews - 28(4)
Issue
Clinician Reviews - 28(4)
Page Number
8,12
Page Number
8,12
Publications
Publications
Topics
Article Type
Display Headline
From Local Bar to Police Car
Display Headline
From Local Bar to Police Car
Sections
Questionnaire Body

A 31-year-old man is brought to your facility by local law enforcement for evaluation of left wrist pain following an altercation. The patient was reportedly at a local bar; as he was leaving, he started arguing with some other patrons. A fight ensued.

The patient believes he was struck by something on his left wrist. He denies any other injuries or complaints. His medical history is unremarkable, and vital signs are stable.

Physical examination of his left wrist shows no obvious deformity. There is some mild swelling over the dorsal aspect of his wrist. Range of motion is painful and limited. Good capillary refill is noted in the fingers, and sensation is intact. Good pulses are present.

Radiograph of the left wrist is obtained. What is your impression?

Disallow All Ads
Content Gating
No Gating (article Unlocked/Free)
Alternative CME
Disqus Comments
Default
Article PDF Media

Are serum troponin levels elevated in conditions other than acute coronary syndrome?

Article Type
Changed
Mon, 04/02/2018 - 07:59
Display Headline
Are serum troponin levels elevated in conditions other than acute coronary syndrome?

Yes. Sepsis, stroke, chronic kidney disease, pulmonary disease, chemotherapy, heart failure, and stress cardiomyopathy can all raise serum troponin concentrations, and in some cases the elevations are prognostically important. Careful clinical assessment, serial monitoring of troponin levels, and other supportive tests are usually necessary to tell whether troponin elevations are due to acute coronary syndrome or to these other causes.

NOT ONLY A MARKER OF MYOCARDIAL DAMAGE

Troponin, an intracellular protein found in skeletal and cardiac muscle cells, is essential for muscle contraction. Troponin T and troponin I are clinically equivalent, and both are biomarkers of myocardial damage.

A troponin assay is ordered when patients present with sudden onset of symptoms of acute coronary syndrome such as chest pain, dyspnea, diaphoresis, and electrocardiographic abnormalities. The assay is positive when the manufacturer-specified threshold corresponding to a concentration above the 99th percentile is detected.

Serial testing of serum biomarkers of acute myocardial damage is essential to confirm the diagnosis of myocardial infarction. Because of their higher sensitivity and specificity compared with creatine kinase-MB and other markers, troponins are the preferred biomarker in diagnosing acute coronary syndrome.

In 1984, Piper et al1 reported that free cytosolic pools of cardiac enzymes could be released after reversible myocardial injury as a result of temporary disruption of the cell membrane. This upended the previous assumption that troponin was released only after irreversible myocardial necrosis, and it provided an explanation for troponin elevations observed in conditions with no evidence of epicardial coronary artery disease or permanent myocardial damage.1

SEPSIS

Studies of patients with sepsis, severe sepsis, and septic shock have shown troponin elevations with no evidence of acute coronary syndrome.2 In sepsis, troponin elevations are presumed to be caused by a combination of events. Renal dysfunction leads to decreased clearance of troponin fragments by the kidneys. The massive inflammatory response in septic shock results in cytokine-induced cardiac damage, and increased levels of endogen­ous and exogenous catecholamines damage cardiac myocytes.3

Studies of the prognostic value of these elevations have produced mixed and contradictory results. But a 2013 meta-analysis4 showed that patients with a troponin elevation at the time of diagnosis of sepsis had a risk of death almost twice that of patients without a troponin elevation (relative risk 1.91, 95% confidence interval [CI] 1.63–2.24).

STROKE

Acute ischemic stroke can trigger troponin elevations in several ways. Since the risk factors for acute ischemic stroke and coronary stenosis are similar, patients who have an ischemic stroke have a higher risk of coronary atherosclerosis and coronary stenosis than the general population.5

Stroke can cause a variety of cardiovascular and respiratory responses (eg, tachyarrhythmia, hypertensive crisis, respiratory failure) that increase the stress on the myocardium. In patients with stroke and concurrent coronary artery stenosis, the increased metabolic demand can exceed the oxygen supply capacity, resulting in myocardial ischemia, which can manifest as increased levels of serum troponin.5

Stroke can also cause troponin elevation through neurogenic myocardial damage. Ischemic stroke and intracranial hemorrhage can trigger alterations in autonomic control. Sometimes this results in increased sympathetic activity with concomitant catecholamine surge, leading to contraction band necrosis and other forms of myocardial damage and, as a result, troponin elevation.5,6 This may explain troponin elevation in patients with acute ischemic stroke in the absence of concomitant coronary artery disease. Recent evidence suggests that patients with acute ischemic stroke and elevated troponin had significantly less angiographic evidence of coronary artery disease than matched patients with non-ST-elevation myocardial infarction.7

 

 

CHRONIC KIDNEY DISEASE

Cardiac troponins may be elevated in chronic kidney disease. Explanations for this include the theory that troponin is broken down into fragments that are cleared by the kidney.8 Therefore, decreased renal function leads to an increase in troponin fragments measured with troponin assays. Other explanations are chronic volume overload, chronic elevation of proinflammatory cytokines, and associated comorbidities such as hypertension.

Troponin elevations can have prognostic significance in chronic kidney disease. In a meta-analysis of 98 studies of patients with chronic kidney disease and no symptoms of acute coronary syndrome, troponin elevation was associated with 2- to 4-fold higher rates of all-cause mortality, cardiovascular mortality, and major acute coronary events in both dialysis-dependent and nondialysis patients.8 Thus, troponin is a unique factor in risk-stratification in patients with chronic kidney disease and could affect how it is managed in the future.

To determine if an acute coronary syndrome is taking place when evaluating patients with chronic kidney disease and elevated troponins, physicians must use other evidence—for example, serial measurements of troponin levels showing continued troponin elevation, elevations in troponin from the patient’s baseline, elevated creatine kinase-MB levels, electrocardiographic changes, and clinical symptoms.

PULMONARY DISEASE

Troponin elevation can signify right heart strain in a variety of pulmonary diseases.

Pulmonary embolism. Troponin elevation is a marker of right ventricular dysfunction in patients with moderate to large pulmonary embolism.

In a study of normotensive patients with acute pulmonary embolism, 52% had elevated serum troponin, and they had a higher risk of an adverse outcome (death, recurrent pulmonary embolism, or major bleeding) within 30 days (odds ratio 4.97, 95% CI 1.71–14.43) and a lower probability of 6-month survival.9 Troponin elevation in pulmonary embolism is not helpful in confirming the diagnosis but is primarily useful in prognosis.

Pulmonary arterial hypertension. Cardiac troponin elevations can also indicate severe disease and poor outcomes in patients with pulmonary arterial hypertension. A study by Heresi et al10 confirmed this, even in patients with only slight elevations in troponin levels. Troponin was detected in 17 (25%) of 68 patients with pulmonary arterial hypertension diagnostic category 1. Further, patients with detectable troponin had more advanced functional class symptoms, a shorter 6-minute walk distance, more pericardial effusions, larger right atrial area, and higher B-type natriuretic peptide and C-reactive protein levels.10

Measuring troponins in the setting of pulmonary hypertension allows clinicians to identify high-risk patients and may help guide the management of these patients.

Chronic obstructive pulmonary disease. Elevation of serum troponins is also reported in patients with acute exacerbation of chronic obstructive pulmonary disease and has been correlated with increased all-cause mortality rates in these patients.11

CHEMOTHERAPY

Chemotherapy-induced cardiotoxicity may result in troponin elevations. Chemotherapy causes cardiac toxicity by several mechanisms, including production of oxygen free radicals, disturbance of mitochondrial energy metabolism, intracellular calcium overload, and increased lipid peroxidation. Chemotherapeutic agents associated with cardiotoxicity include anthracyclines, trastuzumab, chlormethine, and mitomycin.

Chemotherapy-induced left ventricular deterioration is often irreversible. Monitoring troponin levels can help identify problems before cardiac dysfunction becomes clinically evident during the weeks and months after the start of high-dose chemotherapy.

Cardinale et al12 found marked myocardial depression 7 months after the start of high-dose chemotherapy. They reported a close relationship between short-term troponin elevation and the greatest reduction in left-ventricular ejection fraction (r = −0.87; P < .0001). Normal troponin values after high-dose chemotherapy also seemed to identify patients at lower risk, with either no cardiac damage or only transient subclinical left-ventricular dysfunction.12

HEART FAILURE

Heart failure leads to release of cardiac troponins through myocardial strain and myocardial death. Volume and pressure overload of the ventricles causes excessive wall tension, resulting in myofibrillar damage. Measuring troponins is an effective way to detect cardiac myolysis in heart failure, independent of the presence of coronary artery disease.

In heart failure, elevated troponins correlate with adverse outcome in both hospitalized and stable patients. In addition, elevation of both troponins and B-type natriuretic peptide is associated with worse heart failure outcomes than elevation of either marker alone.

A prospective study13 of patients with New York Heart Association class III or IV heart failure showed that an increase in troponin concentration from normal baseline was associated with a risk of death, cardiac transplant, or hospitalization that was 3.4 to 5.09 times higher. Further elevations in B-type natriuretic peptide during the study period were associated with a poor outcome (hazard ratio 5.09; P < .001). Combined elevations of troponin and B-type natriuretic peptide defined the group at highest risk (hazard ratio 8.58; P < .001).

Increased myocardial wall stress may lead to decreased subendocardial perfusion, with resulting troponin elevation and decline in left ventricular systolic function. Further, in vitro experiments with myocytes established a link between myocardial wall stretch and programmed cell death, which may contribute to troponin elevations.14

 

 

STRESS CARDIOMYOPATHY

Profound reversible myocardial depression and troponin elevation are seen after sudden emotional stress, a condition called stress-induced or takotsubo cardiomyopathy. While the exact mechanism of stress-induced cardiomyopathy remains unclear, it is thought to be due to sudden supraphysiologic elevation of catecholamines and related neuropeptides. Although vasospasm in the epicardial and microvascular circulation has been suggested as the possible mechanism of left ventricular systolic dysfunction and troponin elevation, cardiac myocyte injury from catecholamine- induced cyclic AMP-mediated calcium overload and oxygen-derived free radicals appears to be a more likely mechanism.15

PSEUDOELEVATIONS OF TROPONIN

In rare cases, endogenous antibodies (eg, heterophilic antibodies) in the blood specimen can interfere with the processing of the troponin immunoassay in the laboratory, causing a false-positive assay. This can occur with samples from patients with a viral infection or autoimmune condition as well as with samples from patients treated with intravenous immunoglobulin (Ig). Heterophilic antibodies can bind to the Fc region of the test antibodies in certain troponin assays, leading to false-positive elevations.16 Macrotroponin, a molecule found in patients with autoantibodies against troponin I, is composed of troponin I fragments and IgG antibodies and can also cause a false-positive troponin immunoassay.16

In patients with seropositive rheumatoid arthritis, a false-positive troponin I assay was associated with a high concentration of IgM rheumatoid factor with the use of certain immunoassay techniques.17 In patients with acute skeletal muscle injury, the first-generation troponin T assay was found to be falsely positive due to the nonspecific binding of skeletal muscle troponin T to the walls of the test tube used for the assay. When the second-generation troponin T assay was used, troponin T levels were found to be slightly more positive than troponin I levels (1.7 vs 1.5 times the upper limit of normal), especially in patients with renal failure.18

Troponin may also be falsely elevated in hemolyzed blood samples. This has to be taken into consideration in interpreting the results of troponin testing in severely hemolyzed blood samples. However, Puelacher et al19 suggested that the presence of hemolysis did not appear to interfere with clinical value of the test.

References
  1. Piper HM, Schwartz P, Spahr R, Hütter J, Spieckermann P. Early enzyme release from myocardial cells is not due to irreversible cell damage. J Mol Cell Cardiol 1984; 16(4):385–388. doi:10.1016/S0022-2828(84)80609-4
  2. Ammann P, Fehr T, Minder EI, Günter C, Bertel O. Elevation of troponin I in sepsis and septic shock. Intensive Care Med 2001; 27(6):965–969.
  3. Landesberg G, Jaffe AS, Gilon D, et al. Troponin elevation in severe sepsis and septic shock. Crit Care Med 2014; 42(4):790–800. doi:10.1097/CCM.0000000000000107
  4. Bessière F, Khenifer S, Dubourg J, Durieu I, Lega JC. Prognostic value of troponins in sepsis: a meta-analysis. Intensive Care Med 2013; 39(7):1181–1189. doi:10.1007/s00134-013-2902-3
  5. Scheitz JF, Nolte CH, Laufs U, Endres M. Application and interpretation of high-sensitivity cardiac troponin assays in patients with acute ischemic stroke. Stroke 2015; 46(4):1132–1140. doi:10.1161/STROKEAHA.114.007858
  6. Naidech AM, Kreiter KT, Janjua N, et al. Cardiac troponin elevation, cardiovascular morbidity, and outcome after subarachnoid hemorrhage. Circulation 2005; 112(18):2851–2656. doi:10.1161/CIRCULATIONAHA.105.533620
  7. Mochmann HC, Scheitz JF, Petzold GC, et al; TRELAS Study Group. Coronary angiographic findings in acute ischemic stroke patients with elevated cardiac troponin: the Troponin Elevation in Acute Ischemic Stroke (TRELAS) Study. Circulation 2016; 133(13):1264–1271. doi:10.1161/CIRCULATIONAHA.115.018547
  8. Michos ED, Wilson LM, Yeh HC, et al. Prognostic value of cardiac troponin in patients with chronic kidney disease without suspected acute coronary syndrome. Ann Intern Med 2014; 161(7):491–501. doi:10.7326/M14-0743
  9. Lankeit M, Jiménez D, Kostrubiec M, et al. Predictive value of the high-sensitivity troponin T assay and the simplified pulmonary embolism severity index in hemodynamically stable patients with acute pulmonary embolism: a prospective validation study. Circulation 2011; 124(24):2716–2724. doi:10.1161/CIRCULATIONAHA.111.051177
  10. Heresi GA, Tang WH, Aytekin M, Hammel J, Hazen SL, Dweik RA. Sensitive cardiac troponin I predicts poor outcomes in pulmonary arterial hypertension. Eur Respir J 2012; 39(4)939–944. doi:10.1183/09031936.00067011
  11. Pavasini R, d’Ascenzo F, Campo G, et al. Cardiac troponin elevation predicts all-cause mortality in patients with acute exacerbation of chronic obstructive pulmonary disease: systematic review and meta-analysis. Int J Cardiol 2015; 191:187–193. doi:10.1016/j.ijcard.2015.05.006
  12. Cardinale D, Sandri MT, Martinoni A, et al. Left ventricular dysfunction predicted by early troponin I release after high-dose chemotherapy. J Am Coll Cardiol 2000; 36(2):517–522.
  13. Miller WL, Hartman KA, Burritt MF, et al. Serial biomarker measurements in ambulatory patients with chronic heart failure: the importance of change over time. Circulation 2007; 116(3):249–257. doi:10.1161/CIRCULATIONAHA.107.694562
  14. Logeart D, Beyne P, Cusson C, et al. Evidence of cardiac myolysis in severe nonischemic heart failure and the potential role of increased wall strain. Am Heart J 2001; 141(2):247–253. doi:10.1067/mhj.2001.111767
  15. Whittstein IS, Thiemann DR, Lima JA, et al. Neurohumoral features of myocardial stunning due to sudden emotional stress. N Engl J Med 2005; 352(6):539–548. doi:10.1056/NEJMoa043046
  16. McClennen S, Halamka JD, Horowitz GL, Kannam JP, Ho KK. Clinical prevalence and ramifications of false-positive cardiac troponin I elevations from the Abbott AxSYM Analyzer. Am J Cardiol 2003; 91(9):1125–1127.
  17. Bradham WS, Bian A, Oeser A, et al. High-sensitivity cardiac troponin-I is elevated in patients with rheumatoid arthritis, independent of cardiovascular risk factors and inflammation. PLoS One 2012; 7(6):e38930. doi:10.1371/journal.pone.0038930
  18. Li SF, Zapata J, Tillem E. The prevalence of false-positive cardiac troponin I in ED patients with rhabdomyolysis. Am J Emerg Med 2005; 23(7):860–863. doi:10.1016/j.ajem.2005.05.008
  19. Puelacher C, Twerenbold R, Mosimann T, et al. Effects of hemolysis on the diagnostic accuracy of cardiac troponin I for the diagnosis of myocardial infarction. Int J Cardiol 2015; 187:313–315. doi:10.1016/j.ijcard.2015.03.378
Article PDF
Author and Disclosure Information

Kiran Sebastian, MD
Division of Pulmonary, Critical Care, and Sleep Medicine, Wayne State University School of Medicine, Detroit, MI

Alexander Wester, BS
Division of Pulmonary, Critical Care, and Sleep Medicine, Wayne State University School of Medicine, Detroit, MI

Anupama Kottam, MD
Division of Cardiology, Wayne State University School of Medicine, Detroit, MI

Ayman O. Soubani, MD
Division of Pulmonary, Critical Care, and Sleep Medicine, Wayne State University School of Medicine; Professor of Medicine, Wayne State University School of Medicine; Medical Director, Medical ICU, Harper University Hospital; Service Chief, Pulmonary and Critical Care, and Medical Director, Critical Care Service, Karmanos Cancer Center, Detroit, MI

Address: Ayman O. Soubani, MD, Professor of Medicine, Wayne State University School of Medicine, 3990 John R-3 Hudson, Detroit, MI 48201; asoubani@med.wayne.edu

Issue
Cleveland Clinic Journal of Medicine - 85(4)
Publications
Topics
Page Number
274-277
Legacy Keywords
troponin, laboratory testing, acute coronary syndrome, myocardial infarction, MI, sepsis, stroke, chronic kidney disease, CKD, heart failure, stress cardiomyopathy, Kiran Sebastian, Alexander Wester, Anapama Kottam, Ayman Soubani
Sections
Author and Disclosure Information

Kiran Sebastian, MD
Division of Pulmonary, Critical Care, and Sleep Medicine, Wayne State University School of Medicine, Detroit, MI

Alexander Wester, BS
Division of Pulmonary, Critical Care, and Sleep Medicine, Wayne State University School of Medicine, Detroit, MI

Anupama Kottam, MD
Division of Cardiology, Wayne State University School of Medicine, Detroit, MI

Ayman O. Soubani, MD
Division of Pulmonary, Critical Care, and Sleep Medicine, Wayne State University School of Medicine; Professor of Medicine, Wayne State University School of Medicine; Medical Director, Medical ICU, Harper University Hospital; Service Chief, Pulmonary and Critical Care, and Medical Director, Critical Care Service, Karmanos Cancer Center, Detroit, MI

Address: Ayman O. Soubani, MD, Professor of Medicine, Wayne State University School of Medicine, 3990 John R-3 Hudson, Detroit, MI 48201; asoubani@med.wayne.edu

Author and Disclosure Information

Kiran Sebastian, MD
Division of Pulmonary, Critical Care, and Sleep Medicine, Wayne State University School of Medicine, Detroit, MI

Alexander Wester, BS
Division of Pulmonary, Critical Care, and Sleep Medicine, Wayne State University School of Medicine, Detroit, MI

Anupama Kottam, MD
Division of Cardiology, Wayne State University School of Medicine, Detroit, MI

Ayman O. Soubani, MD
Division of Pulmonary, Critical Care, and Sleep Medicine, Wayne State University School of Medicine; Professor of Medicine, Wayne State University School of Medicine; Medical Director, Medical ICU, Harper University Hospital; Service Chief, Pulmonary and Critical Care, and Medical Director, Critical Care Service, Karmanos Cancer Center, Detroit, MI

Address: Ayman O. Soubani, MD, Professor of Medicine, Wayne State University School of Medicine, 3990 John R-3 Hudson, Detroit, MI 48201; asoubani@med.wayne.edu

Article PDF
Article PDF
Related Articles

Yes. Sepsis, stroke, chronic kidney disease, pulmonary disease, chemotherapy, heart failure, and stress cardiomyopathy can all raise serum troponin concentrations, and in some cases the elevations are prognostically important. Careful clinical assessment, serial monitoring of troponin levels, and other supportive tests are usually necessary to tell whether troponin elevations are due to acute coronary syndrome or to these other causes.

NOT ONLY A MARKER OF MYOCARDIAL DAMAGE

Troponin, an intracellular protein found in skeletal and cardiac muscle cells, is essential for muscle contraction. Troponin T and troponin I are clinically equivalent, and both are biomarkers of myocardial damage.

A troponin assay is ordered when patients present with sudden onset of symptoms of acute coronary syndrome such as chest pain, dyspnea, diaphoresis, and electrocardiographic abnormalities. The assay is positive when the manufacturer-specified threshold corresponding to a concentration above the 99th percentile is detected.

Serial testing of serum biomarkers of acute myocardial damage is essential to confirm the diagnosis of myocardial infarction. Because of their higher sensitivity and specificity compared with creatine kinase-MB and other markers, troponins are the preferred biomarker in diagnosing acute coronary syndrome.

In 1984, Piper et al1 reported that free cytosolic pools of cardiac enzymes could be released after reversible myocardial injury as a result of temporary disruption of the cell membrane. This upended the previous assumption that troponin was released only after irreversible myocardial necrosis, and it provided an explanation for troponin elevations observed in conditions with no evidence of epicardial coronary artery disease or permanent myocardial damage.1

SEPSIS

Studies of patients with sepsis, severe sepsis, and septic shock have shown troponin elevations with no evidence of acute coronary syndrome.2 In sepsis, troponin elevations are presumed to be caused by a combination of events. Renal dysfunction leads to decreased clearance of troponin fragments by the kidneys. The massive inflammatory response in septic shock results in cytokine-induced cardiac damage, and increased levels of endogen­ous and exogenous catecholamines damage cardiac myocytes.3

Studies of the prognostic value of these elevations have produced mixed and contradictory results. But a 2013 meta-analysis4 showed that patients with a troponin elevation at the time of diagnosis of sepsis had a risk of death almost twice that of patients without a troponin elevation (relative risk 1.91, 95% confidence interval [CI] 1.63–2.24).

STROKE

Acute ischemic stroke can trigger troponin elevations in several ways. Since the risk factors for acute ischemic stroke and coronary stenosis are similar, patients who have an ischemic stroke have a higher risk of coronary atherosclerosis and coronary stenosis than the general population.5

Stroke can cause a variety of cardiovascular and respiratory responses (eg, tachyarrhythmia, hypertensive crisis, respiratory failure) that increase the stress on the myocardium. In patients with stroke and concurrent coronary artery stenosis, the increased metabolic demand can exceed the oxygen supply capacity, resulting in myocardial ischemia, which can manifest as increased levels of serum troponin.5

Stroke can also cause troponin elevation through neurogenic myocardial damage. Ischemic stroke and intracranial hemorrhage can trigger alterations in autonomic control. Sometimes this results in increased sympathetic activity with concomitant catecholamine surge, leading to contraction band necrosis and other forms of myocardial damage and, as a result, troponin elevation.5,6 This may explain troponin elevation in patients with acute ischemic stroke in the absence of concomitant coronary artery disease. Recent evidence suggests that patients with acute ischemic stroke and elevated troponin had significantly less angiographic evidence of coronary artery disease than matched patients with non-ST-elevation myocardial infarction.7

 

 

CHRONIC KIDNEY DISEASE

Cardiac troponins may be elevated in chronic kidney disease. Explanations for this include the theory that troponin is broken down into fragments that are cleared by the kidney.8 Therefore, decreased renal function leads to an increase in troponin fragments measured with troponin assays. Other explanations are chronic volume overload, chronic elevation of proinflammatory cytokines, and associated comorbidities such as hypertension.

Troponin elevations can have prognostic significance in chronic kidney disease. In a meta-analysis of 98 studies of patients with chronic kidney disease and no symptoms of acute coronary syndrome, troponin elevation was associated with 2- to 4-fold higher rates of all-cause mortality, cardiovascular mortality, and major acute coronary events in both dialysis-dependent and nondialysis patients.8 Thus, troponin is a unique factor in risk-stratification in patients with chronic kidney disease and could affect how it is managed in the future.

To determine if an acute coronary syndrome is taking place when evaluating patients with chronic kidney disease and elevated troponins, physicians must use other evidence—for example, serial measurements of troponin levels showing continued troponin elevation, elevations in troponin from the patient’s baseline, elevated creatine kinase-MB levels, electrocardiographic changes, and clinical symptoms.

PULMONARY DISEASE

Troponin elevation can signify right heart strain in a variety of pulmonary diseases.

Pulmonary embolism. Troponin elevation is a marker of right ventricular dysfunction in patients with moderate to large pulmonary embolism.

In a study of normotensive patients with acute pulmonary embolism, 52% had elevated serum troponin, and they had a higher risk of an adverse outcome (death, recurrent pulmonary embolism, or major bleeding) within 30 days (odds ratio 4.97, 95% CI 1.71–14.43) and a lower probability of 6-month survival.9 Troponin elevation in pulmonary embolism is not helpful in confirming the diagnosis but is primarily useful in prognosis.

Pulmonary arterial hypertension. Cardiac troponin elevations can also indicate severe disease and poor outcomes in patients with pulmonary arterial hypertension. A study by Heresi et al10 confirmed this, even in patients with only slight elevations in troponin levels. Troponin was detected in 17 (25%) of 68 patients with pulmonary arterial hypertension diagnostic category 1. Further, patients with detectable troponin had more advanced functional class symptoms, a shorter 6-minute walk distance, more pericardial effusions, larger right atrial area, and higher B-type natriuretic peptide and C-reactive protein levels.10

Measuring troponins in the setting of pulmonary hypertension allows clinicians to identify high-risk patients and may help guide the management of these patients.

Chronic obstructive pulmonary disease. Elevation of serum troponins is also reported in patients with acute exacerbation of chronic obstructive pulmonary disease and has been correlated with increased all-cause mortality rates in these patients.11

CHEMOTHERAPY

Chemotherapy-induced cardiotoxicity may result in troponin elevations. Chemotherapy causes cardiac toxicity by several mechanisms, including production of oxygen free radicals, disturbance of mitochondrial energy metabolism, intracellular calcium overload, and increased lipid peroxidation. Chemotherapeutic agents associated with cardiotoxicity include anthracyclines, trastuzumab, chlormethine, and mitomycin.

Chemotherapy-induced left ventricular deterioration is often irreversible. Monitoring troponin levels can help identify problems before cardiac dysfunction becomes clinically evident during the weeks and months after the start of high-dose chemotherapy.

Cardinale et al12 found marked myocardial depression 7 months after the start of high-dose chemotherapy. They reported a close relationship between short-term troponin elevation and the greatest reduction in left-ventricular ejection fraction (r = −0.87; P < .0001). Normal troponin values after high-dose chemotherapy also seemed to identify patients at lower risk, with either no cardiac damage or only transient subclinical left-ventricular dysfunction.12

HEART FAILURE

Heart failure leads to release of cardiac troponins through myocardial strain and myocardial death. Volume and pressure overload of the ventricles causes excessive wall tension, resulting in myofibrillar damage. Measuring troponins is an effective way to detect cardiac myolysis in heart failure, independent of the presence of coronary artery disease.

In heart failure, elevated troponins correlate with adverse outcome in both hospitalized and stable patients. In addition, elevation of both troponins and B-type natriuretic peptide is associated with worse heart failure outcomes than elevation of either marker alone.

A prospective study13 of patients with New York Heart Association class III or IV heart failure showed that an increase in troponin concentration from normal baseline was associated with a risk of death, cardiac transplant, or hospitalization that was 3.4 to 5.09 times higher. Further elevations in B-type natriuretic peptide during the study period were associated with a poor outcome (hazard ratio 5.09; P < .001). Combined elevations of troponin and B-type natriuretic peptide defined the group at highest risk (hazard ratio 8.58; P < .001).

Increased myocardial wall stress may lead to decreased subendocardial perfusion, with resulting troponin elevation and decline in left ventricular systolic function. Further, in vitro experiments with myocytes established a link between myocardial wall stretch and programmed cell death, which may contribute to troponin elevations.14

 

 

STRESS CARDIOMYOPATHY

Profound reversible myocardial depression and troponin elevation are seen after sudden emotional stress, a condition called stress-induced or takotsubo cardiomyopathy. While the exact mechanism of stress-induced cardiomyopathy remains unclear, it is thought to be due to sudden supraphysiologic elevation of catecholamines and related neuropeptides. Although vasospasm in the epicardial and microvascular circulation has been suggested as the possible mechanism of left ventricular systolic dysfunction and troponin elevation, cardiac myocyte injury from catecholamine- induced cyclic AMP-mediated calcium overload and oxygen-derived free radicals appears to be a more likely mechanism.15

PSEUDOELEVATIONS OF TROPONIN

In rare cases, endogenous antibodies (eg, heterophilic antibodies) in the blood specimen can interfere with the processing of the troponin immunoassay in the laboratory, causing a false-positive assay. This can occur with samples from patients with a viral infection or autoimmune condition as well as with samples from patients treated with intravenous immunoglobulin (Ig). Heterophilic antibodies can bind to the Fc region of the test antibodies in certain troponin assays, leading to false-positive elevations.16 Macrotroponin, a molecule found in patients with autoantibodies against troponin I, is composed of troponin I fragments and IgG antibodies and can also cause a false-positive troponin immunoassay.16

In patients with seropositive rheumatoid arthritis, a false-positive troponin I assay was associated with a high concentration of IgM rheumatoid factor with the use of certain immunoassay techniques.17 In patients with acute skeletal muscle injury, the first-generation troponin T assay was found to be falsely positive due to the nonspecific binding of skeletal muscle troponin T to the walls of the test tube used for the assay. When the second-generation troponin T assay was used, troponin T levels were found to be slightly more positive than troponin I levels (1.7 vs 1.5 times the upper limit of normal), especially in patients with renal failure.18

Troponin may also be falsely elevated in hemolyzed blood samples. This has to be taken into consideration in interpreting the results of troponin testing in severely hemolyzed blood samples. However, Puelacher et al19 suggested that the presence of hemolysis did not appear to interfere with clinical value of the test.

Yes. Sepsis, stroke, chronic kidney disease, pulmonary disease, chemotherapy, heart failure, and stress cardiomyopathy can all raise serum troponin concentrations, and in some cases the elevations are prognostically important. Careful clinical assessment, serial monitoring of troponin levels, and other supportive tests are usually necessary to tell whether troponin elevations are due to acute coronary syndrome or to these other causes.

NOT ONLY A MARKER OF MYOCARDIAL DAMAGE

Troponin, an intracellular protein found in skeletal and cardiac muscle cells, is essential for muscle contraction. Troponin T and troponin I are clinically equivalent, and both are biomarkers of myocardial damage.

A troponin assay is ordered when patients present with sudden onset of symptoms of acute coronary syndrome such as chest pain, dyspnea, diaphoresis, and electrocardiographic abnormalities. The assay is positive when the manufacturer-specified threshold corresponding to a concentration above the 99th percentile is detected.

Serial testing of serum biomarkers of acute myocardial damage is essential to confirm the diagnosis of myocardial infarction. Because of their higher sensitivity and specificity compared with creatine kinase-MB and other markers, troponins are the preferred biomarker in diagnosing acute coronary syndrome.

In 1984, Piper et al1 reported that free cytosolic pools of cardiac enzymes could be released after reversible myocardial injury as a result of temporary disruption of the cell membrane. This upended the previous assumption that troponin was released only after irreversible myocardial necrosis, and it provided an explanation for troponin elevations observed in conditions with no evidence of epicardial coronary artery disease or permanent myocardial damage.1

SEPSIS

Studies of patients with sepsis, severe sepsis, and septic shock have shown troponin elevations with no evidence of acute coronary syndrome.2 In sepsis, troponin elevations are presumed to be caused by a combination of events. Renal dysfunction leads to decreased clearance of troponin fragments by the kidneys. The massive inflammatory response in septic shock results in cytokine-induced cardiac damage, and increased levels of endogen­ous and exogenous catecholamines damage cardiac myocytes.3

Studies of the prognostic value of these elevations have produced mixed and contradictory results. But a 2013 meta-analysis4 showed that patients with a troponin elevation at the time of diagnosis of sepsis had a risk of death almost twice that of patients without a troponin elevation (relative risk 1.91, 95% confidence interval [CI] 1.63–2.24).

STROKE

Acute ischemic stroke can trigger troponin elevations in several ways. Since the risk factors for acute ischemic stroke and coronary stenosis are similar, patients who have an ischemic stroke have a higher risk of coronary atherosclerosis and coronary stenosis than the general population.5

Stroke can cause a variety of cardiovascular and respiratory responses (eg, tachyarrhythmia, hypertensive crisis, respiratory failure) that increase the stress on the myocardium. In patients with stroke and concurrent coronary artery stenosis, the increased metabolic demand can exceed the oxygen supply capacity, resulting in myocardial ischemia, which can manifest as increased levels of serum troponin.5

Stroke can also cause troponin elevation through neurogenic myocardial damage. Ischemic stroke and intracranial hemorrhage can trigger alterations in autonomic control. Sometimes this results in increased sympathetic activity with concomitant catecholamine surge, leading to contraction band necrosis and other forms of myocardial damage and, as a result, troponin elevation.5,6 This may explain troponin elevation in patients with acute ischemic stroke in the absence of concomitant coronary artery disease. Recent evidence suggests that patients with acute ischemic stroke and elevated troponin had significantly less angiographic evidence of coronary artery disease than matched patients with non-ST-elevation myocardial infarction.7

 

 

CHRONIC KIDNEY DISEASE

Cardiac troponins may be elevated in chronic kidney disease. Explanations for this include the theory that troponin is broken down into fragments that are cleared by the kidney.8 Therefore, decreased renal function leads to an increase in troponin fragments measured with troponin assays. Other explanations are chronic volume overload, chronic elevation of proinflammatory cytokines, and associated comorbidities such as hypertension.

Troponin elevations can have prognostic significance in chronic kidney disease. In a meta-analysis of 98 studies of patients with chronic kidney disease and no symptoms of acute coronary syndrome, troponin elevation was associated with 2- to 4-fold higher rates of all-cause mortality, cardiovascular mortality, and major acute coronary events in both dialysis-dependent and nondialysis patients.8 Thus, troponin is a unique factor in risk-stratification in patients with chronic kidney disease and could affect how it is managed in the future.

To determine if an acute coronary syndrome is taking place when evaluating patients with chronic kidney disease and elevated troponins, physicians must use other evidence—for example, serial measurements of troponin levels showing continued troponin elevation, elevations in troponin from the patient’s baseline, elevated creatine kinase-MB levels, electrocardiographic changes, and clinical symptoms.

PULMONARY DISEASE

Troponin elevation can signify right heart strain in a variety of pulmonary diseases.

Pulmonary embolism. Troponin elevation is a marker of right ventricular dysfunction in patients with moderate to large pulmonary embolism.

In a study of normotensive patients with acute pulmonary embolism, 52% had elevated serum troponin, and they had a higher risk of an adverse outcome (death, recurrent pulmonary embolism, or major bleeding) within 30 days (odds ratio 4.97, 95% CI 1.71–14.43) and a lower probability of 6-month survival.9 Troponin elevation in pulmonary embolism is not helpful in confirming the diagnosis but is primarily useful in prognosis.

Pulmonary arterial hypertension. Cardiac troponin elevations can also indicate severe disease and poor outcomes in patients with pulmonary arterial hypertension. A study by Heresi et al10 confirmed this, even in patients with only slight elevations in troponin levels. Troponin was detected in 17 (25%) of 68 patients with pulmonary arterial hypertension diagnostic category 1. Further, patients with detectable troponin had more advanced functional class symptoms, a shorter 6-minute walk distance, more pericardial effusions, larger right atrial area, and higher B-type natriuretic peptide and C-reactive protein levels.10

Measuring troponins in the setting of pulmonary hypertension allows clinicians to identify high-risk patients and may help guide the management of these patients.

Chronic obstructive pulmonary disease. Elevation of serum troponins is also reported in patients with acute exacerbation of chronic obstructive pulmonary disease and has been correlated with increased all-cause mortality rates in these patients.11

CHEMOTHERAPY

Chemotherapy-induced cardiotoxicity may result in troponin elevations. Chemotherapy causes cardiac toxicity by several mechanisms, including production of oxygen free radicals, disturbance of mitochondrial energy metabolism, intracellular calcium overload, and increased lipid peroxidation. Chemotherapeutic agents associated with cardiotoxicity include anthracyclines, trastuzumab, chlormethine, and mitomycin.

Chemotherapy-induced left ventricular deterioration is often irreversible. Monitoring troponin levels can help identify problems before cardiac dysfunction becomes clinically evident during the weeks and months after the start of high-dose chemotherapy.

Cardinale et al12 found marked myocardial depression 7 months after the start of high-dose chemotherapy. They reported a close relationship between short-term troponin elevation and the greatest reduction in left-ventricular ejection fraction (r = −0.87; P < .0001). Normal troponin values after high-dose chemotherapy also seemed to identify patients at lower risk, with either no cardiac damage or only transient subclinical left-ventricular dysfunction.12

HEART FAILURE

Heart failure leads to release of cardiac troponins through myocardial strain and myocardial death. Volume and pressure overload of the ventricles causes excessive wall tension, resulting in myofibrillar damage. Measuring troponins is an effective way to detect cardiac myolysis in heart failure, independent of the presence of coronary artery disease.

In heart failure, elevated troponins correlate with adverse outcome in both hospitalized and stable patients. In addition, elevation of both troponins and B-type natriuretic peptide is associated with worse heart failure outcomes than elevation of either marker alone.

A prospective study13 of patients with New York Heart Association class III or IV heart failure showed that an increase in troponin concentration from normal baseline was associated with a risk of death, cardiac transplant, or hospitalization that was 3.4 to 5.09 times higher. Further elevations in B-type natriuretic peptide during the study period were associated with a poor outcome (hazard ratio 5.09; P < .001). Combined elevations of troponin and B-type natriuretic peptide defined the group at highest risk (hazard ratio 8.58; P < .001).

Increased myocardial wall stress may lead to decreased subendocardial perfusion, with resulting troponin elevation and decline in left ventricular systolic function. Further, in vitro experiments with myocytes established a link between myocardial wall stretch and programmed cell death, which may contribute to troponin elevations.14

 

 

STRESS CARDIOMYOPATHY

Profound reversible myocardial depression and troponin elevation are seen after sudden emotional stress, a condition called stress-induced or takotsubo cardiomyopathy. While the exact mechanism of stress-induced cardiomyopathy remains unclear, it is thought to be due to sudden supraphysiologic elevation of catecholamines and related neuropeptides. Although vasospasm in the epicardial and microvascular circulation has been suggested as the possible mechanism of left ventricular systolic dysfunction and troponin elevation, cardiac myocyte injury from catecholamine- induced cyclic AMP-mediated calcium overload and oxygen-derived free radicals appears to be a more likely mechanism.15

PSEUDOELEVATIONS OF TROPONIN

In rare cases, endogenous antibodies (eg, heterophilic antibodies) in the blood specimen can interfere with the processing of the troponin immunoassay in the laboratory, causing a false-positive assay. This can occur with samples from patients with a viral infection or autoimmune condition as well as with samples from patients treated with intravenous immunoglobulin (Ig). Heterophilic antibodies can bind to the Fc region of the test antibodies in certain troponin assays, leading to false-positive elevations.16 Macrotroponin, a molecule found in patients with autoantibodies against troponin I, is composed of troponin I fragments and IgG antibodies and can also cause a false-positive troponin immunoassay.16

In patients with seropositive rheumatoid arthritis, a false-positive troponin I assay was associated with a high concentration of IgM rheumatoid factor with the use of certain immunoassay techniques.17 In patients with acute skeletal muscle injury, the first-generation troponin T assay was found to be falsely positive due to the nonspecific binding of skeletal muscle troponin T to the walls of the test tube used for the assay. When the second-generation troponin T assay was used, troponin T levels were found to be slightly more positive than troponin I levels (1.7 vs 1.5 times the upper limit of normal), especially in patients with renal failure.18

Troponin may also be falsely elevated in hemolyzed blood samples. This has to be taken into consideration in interpreting the results of troponin testing in severely hemolyzed blood samples. However, Puelacher et al19 suggested that the presence of hemolysis did not appear to interfere with clinical value of the test.

References
  1. Piper HM, Schwartz P, Spahr R, Hütter J, Spieckermann P. Early enzyme release from myocardial cells is not due to irreversible cell damage. J Mol Cell Cardiol 1984; 16(4):385–388. doi:10.1016/S0022-2828(84)80609-4
  2. Ammann P, Fehr T, Minder EI, Günter C, Bertel O. Elevation of troponin I in sepsis and septic shock. Intensive Care Med 2001; 27(6):965–969.
  3. Landesberg G, Jaffe AS, Gilon D, et al. Troponin elevation in severe sepsis and septic shock. Crit Care Med 2014; 42(4):790–800. doi:10.1097/CCM.0000000000000107
  4. Bessière F, Khenifer S, Dubourg J, Durieu I, Lega JC. Prognostic value of troponins in sepsis: a meta-analysis. Intensive Care Med 2013; 39(7):1181–1189. doi:10.1007/s00134-013-2902-3
  5. Scheitz JF, Nolte CH, Laufs U, Endres M. Application and interpretation of high-sensitivity cardiac troponin assays in patients with acute ischemic stroke. Stroke 2015; 46(4):1132–1140. doi:10.1161/STROKEAHA.114.007858
  6. Naidech AM, Kreiter KT, Janjua N, et al. Cardiac troponin elevation, cardiovascular morbidity, and outcome after subarachnoid hemorrhage. Circulation 2005; 112(18):2851–2656. doi:10.1161/CIRCULATIONAHA.105.533620
  7. Mochmann HC, Scheitz JF, Petzold GC, et al; TRELAS Study Group. Coronary angiographic findings in acute ischemic stroke patients with elevated cardiac troponin: the Troponin Elevation in Acute Ischemic Stroke (TRELAS) Study. Circulation 2016; 133(13):1264–1271. doi:10.1161/CIRCULATIONAHA.115.018547
  8. Michos ED, Wilson LM, Yeh HC, et al. Prognostic value of cardiac troponin in patients with chronic kidney disease without suspected acute coronary syndrome. Ann Intern Med 2014; 161(7):491–501. doi:10.7326/M14-0743
  9. Lankeit M, Jiménez D, Kostrubiec M, et al. Predictive value of the high-sensitivity troponin T assay and the simplified pulmonary embolism severity index in hemodynamically stable patients with acute pulmonary embolism: a prospective validation study. Circulation 2011; 124(24):2716–2724. doi:10.1161/CIRCULATIONAHA.111.051177
  10. Heresi GA, Tang WH, Aytekin M, Hammel J, Hazen SL, Dweik RA. Sensitive cardiac troponin I predicts poor outcomes in pulmonary arterial hypertension. Eur Respir J 2012; 39(4)939–944. doi:10.1183/09031936.00067011
  11. Pavasini R, d’Ascenzo F, Campo G, et al. Cardiac troponin elevation predicts all-cause mortality in patients with acute exacerbation of chronic obstructive pulmonary disease: systematic review and meta-analysis. Int J Cardiol 2015; 191:187–193. doi:10.1016/j.ijcard.2015.05.006
  12. Cardinale D, Sandri MT, Martinoni A, et al. Left ventricular dysfunction predicted by early troponin I release after high-dose chemotherapy. J Am Coll Cardiol 2000; 36(2):517–522.
  13. Miller WL, Hartman KA, Burritt MF, et al. Serial biomarker measurements in ambulatory patients with chronic heart failure: the importance of change over time. Circulation 2007; 116(3):249–257. doi:10.1161/CIRCULATIONAHA.107.694562
  14. Logeart D, Beyne P, Cusson C, et al. Evidence of cardiac myolysis in severe nonischemic heart failure and the potential role of increased wall strain. Am Heart J 2001; 141(2):247–253. doi:10.1067/mhj.2001.111767
  15. Whittstein IS, Thiemann DR, Lima JA, et al. Neurohumoral features of myocardial stunning due to sudden emotional stress. N Engl J Med 2005; 352(6):539–548. doi:10.1056/NEJMoa043046
  16. McClennen S, Halamka JD, Horowitz GL, Kannam JP, Ho KK. Clinical prevalence and ramifications of false-positive cardiac troponin I elevations from the Abbott AxSYM Analyzer. Am J Cardiol 2003; 91(9):1125–1127.
  17. Bradham WS, Bian A, Oeser A, et al. High-sensitivity cardiac troponin-I is elevated in patients with rheumatoid arthritis, independent of cardiovascular risk factors and inflammation. PLoS One 2012; 7(6):e38930. doi:10.1371/journal.pone.0038930
  18. Li SF, Zapata J, Tillem E. The prevalence of false-positive cardiac troponin I in ED patients with rhabdomyolysis. Am J Emerg Med 2005; 23(7):860–863. doi:10.1016/j.ajem.2005.05.008
  19. Puelacher C, Twerenbold R, Mosimann T, et al. Effects of hemolysis on the diagnostic accuracy of cardiac troponin I for the diagnosis of myocardial infarction. Int J Cardiol 2015; 187:313–315. doi:10.1016/j.ijcard.2015.03.378
References
  1. Piper HM, Schwartz P, Spahr R, Hütter J, Spieckermann P. Early enzyme release from myocardial cells is not due to irreversible cell damage. J Mol Cell Cardiol 1984; 16(4):385–388. doi:10.1016/S0022-2828(84)80609-4
  2. Ammann P, Fehr T, Minder EI, Günter C, Bertel O. Elevation of troponin I in sepsis and septic shock. Intensive Care Med 2001; 27(6):965–969.
  3. Landesberg G, Jaffe AS, Gilon D, et al. Troponin elevation in severe sepsis and septic shock. Crit Care Med 2014; 42(4):790–800. doi:10.1097/CCM.0000000000000107
  4. Bessière F, Khenifer S, Dubourg J, Durieu I, Lega JC. Prognostic value of troponins in sepsis: a meta-analysis. Intensive Care Med 2013; 39(7):1181–1189. doi:10.1007/s00134-013-2902-3
  5. Scheitz JF, Nolte CH, Laufs U, Endres M. Application and interpretation of high-sensitivity cardiac troponin assays in patients with acute ischemic stroke. Stroke 2015; 46(4):1132–1140. doi:10.1161/STROKEAHA.114.007858
  6. Naidech AM, Kreiter KT, Janjua N, et al. Cardiac troponin elevation, cardiovascular morbidity, and outcome after subarachnoid hemorrhage. Circulation 2005; 112(18):2851–2656. doi:10.1161/CIRCULATIONAHA.105.533620
  7. Mochmann HC, Scheitz JF, Petzold GC, et al; TRELAS Study Group. Coronary angiographic findings in acute ischemic stroke patients with elevated cardiac troponin: the Troponin Elevation in Acute Ischemic Stroke (TRELAS) Study. Circulation 2016; 133(13):1264–1271. doi:10.1161/CIRCULATIONAHA.115.018547
  8. Michos ED, Wilson LM, Yeh HC, et al. Prognostic value of cardiac troponin in patients with chronic kidney disease without suspected acute coronary syndrome. Ann Intern Med 2014; 161(7):491–501. doi:10.7326/M14-0743
  9. Lankeit M, Jiménez D, Kostrubiec M, et al. Predictive value of the high-sensitivity troponin T assay and the simplified pulmonary embolism severity index in hemodynamically stable patients with acute pulmonary embolism: a prospective validation study. Circulation 2011; 124(24):2716–2724. doi:10.1161/CIRCULATIONAHA.111.051177
  10. Heresi GA, Tang WH, Aytekin M, Hammel J, Hazen SL, Dweik RA. Sensitive cardiac troponin I predicts poor outcomes in pulmonary arterial hypertension. Eur Respir J 2012; 39(4)939–944. doi:10.1183/09031936.00067011
  11. Pavasini R, d’Ascenzo F, Campo G, et al. Cardiac troponin elevation predicts all-cause mortality in patients with acute exacerbation of chronic obstructive pulmonary disease: systematic review and meta-analysis. Int J Cardiol 2015; 191:187–193. doi:10.1016/j.ijcard.2015.05.006
  12. Cardinale D, Sandri MT, Martinoni A, et al. Left ventricular dysfunction predicted by early troponin I release after high-dose chemotherapy. J Am Coll Cardiol 2000; 36(2):517–522.
  13. Miller WL, Hartman KA, Burritt MF, et al. Serial biomarker measurements in ambulatory patients with chronic heart failure: the importance of change over time. Circulation 2007; 116(3):249–257. doi:10.1161/CIRCULATIONAHA.107.694562
  14. Logeart D, Beyne P, Cusson C, et al. Evidence of cardiac myolysis in severe nonischemic heart failure and the potential role of increased wall strain. Am Heart J 2001; 141(2):247–253. doi:10.1067/mhj.2001.111767
  15. Whittstein IS, Thiemann DR, Lima JA, et al. Neurohumoral features of myocardial stunning due to sudden emotional stress. N Engl J Med 2005; 352(6):539–548. doi:10.1056/NEJMoa043046
  16. McClennen S, Halamka JD, Horowitz GL, Kannam JP, Ho KK. Clinical prevalence and ramifications of false-positive cardiac troponin I elevations from the Abbott AxSYM Analyzer. Am J Cardiol 2003; 91(9):1125–1127.
  17. Bradham WS, Bian A, Oeser A, et al. High-sensitivity cardiac troponin-I is elevated in patients with rheumatoid arthritis, independent of cardiovascular risk factors and inflammation. PLoS One 2012; 7(6):e38930. doi:10.1371/journal.pone.0038930
  18. Li SF, Zapata J, Tillem E. The prevalence of false-positive cardiac troponin I in ED patients with rhabdomyolysis. Am J Emerg Med 2005; 23(7):860–863. doi:10.1016/j.ajem.2005.05.008
  19. Puelacher C, Twerenbold R, Mosimann T, et al. Effects of hemolysis on the diagnostic accuracy of cardiac troponin I for the diagnosis of myocardial infarction. Int J Cardiol 2015; 187:313–315. doi:10.1016/j.ijcard.2015.03.378
Issue
Cleveland Clinic Journal of Medicine - 85(4)
Issue
Cleveland Clinic Journal of Medicine - 85(4)
Page Number
274-277
Page Number
274-277
Publications
Publications
Topics
Article Type
Display Headline
Are serum troponin levels elevated in conditions other than acute coronary syndrome?
Display Headline
Are serum troponin levels elevated in conditions other than acute coronary syndrome?
Legacy Keywords
troponin, laboratory testing, acute coronary syndrome, myocardial infarction, MI, sepsis, stroke, chronic kidney disease, CKD, heart failure, stress cardiomyopathy, Kiran Sebastian, Alexander Wester, Anapama Kottam, Ayman Soubani
Legacy Keywords
troponin, laboratory testing, acute coronary syndrome, myocardial infarction, MI, sepsis, stroke, chronic kidney disease, CKD, heart failure, stress cardiomyopathy, Kiran Sebastian, Alexander Wester, Anapama Kottam, Ayman Soubani
Sections
Disallow All Ads
Content Gating
No Gating (article Unlocked/Free)
Alternative CME
Disqus Comments
Default
Gate On Date
Wed, 03/28/2018 - 11:30
Un-Gate On Date
Wed, 03/28/2018 - 11:30
Article PDF Media

A 71-year-old woman with shock and a high INR

Article Type
Changed
Mon, 04/01/2019 - 10:22
Display Headline
A 71-year-old woman with shock and a high INR

A 71-year-old woman is brought to the emergency department by her neighbor after complaining of fatigue and light-headedness for the last 8 hours. The patient lives alone and was feeling well when she woke up this morning, but then began to feel nauseated and vomited twice.

The patient appears drowsy and confused and cannot provide any further history. Her medical records show that she was seen in the cardiology clinic 6 months ago but has not kept her appointments since then.

Her medical history includes atrial fibrillation, hypertension, type 2 diabetes mellitus, and osteoarthritis. Her medications are daily warfarin, atenolol, aspirin, candesartan, and metformin, and she takes acetaminophen as needed. She is neither a smoker nor a drug user, but she drinks alcohol occasionally. Her family history is significant for her mother’s death from breast cancer at age 55.

The neighbor confirms that the patient appeared well this morning and has not had any recent illnesses except for a minor cold last week that improved over 5 days with acetaminophen only.

INITIAL EVALUATION AND MANAGEMENT

Physical examination

On physical examination, her blood pressure is 80/40 mm Hg, respiratory rate 25 breaths per minute, oral temperature 38.3°C (100.9°F), and heart rate 130 beats per minute and irregular.

Her neck veins are flat, and her chest is clear to auscultation with normal heart sounds. Abdominal palpation elicits discomfort in the middle segments, voluntary withdrawal, and abdominal wall rigidity. Her skin feels dry and cool, with decreased turgor.

Initial treatment

The patient is given 1 L of 0.9% saline intravenously over the first hour and then is transferred to the intensive care unit, where a norepinephrine drip is started to treat her ongoing hypotension. Normal saline is continued at a rate of 500 mL per hour for the next 4 hours.

Cardiac monitoring and 12-lead electrocardiography show atrial fibrillation with a rapid ventricular response of 138 beats per minute, but electrical cardioversion is not done.

Initial laboratory tests

Initial laboratory results
Results of basic laboratory tests in the emergency department are shown in Table 1.

Of note, her international normalized ratio (INR) is 6.13, while the therapeutic range for a patient taking warfarin because of atrial fibrillation is 2.0 to 3.0.

Her blood pH is 7.34 (reference range 7.35–7.45), and her bicarbonate level is 18 mmol/L (22–26); a low pH and low bicarbonate together indicate metabolic acidosis. Her sodium level is 128 mmol/L (135–145), her chloride level is 100 mmol/L (97–107), and, as mentioned, her bicarbonate level is 18 mmol/L; therefore, her anion gap is 128 – (100 + 18) = 10 mmol/L, which is normal (≤ 10).1

Her serum creatinine level is 1.3 mg/dL (0.5–1.1), and her blood urea nitrogen level is 35 mg/dL (7–20).

Her potassium level is 5.8 mmol/L, which is consistent with hyperkalemia (reference range 3.5–5.2).

DIFFERENTIAL DIAGNOSIS

1. Which of the following is the most likely cause of this patient’s symptoms?

  • Adrenal crisis
  • Cardiogenic shock due to decreased cardiac contractility
  • Intracranial hemorrhage
  • Acute abdomen due to small bowel obstruction
  • Septic shock due to bacterial toxin-induced loss of vascular tone

Our patient is presenting with shock. Given our inability to obtain a meaningful history, the differential diagnosis is broad and includes all of the above.

Adrenal crisis

The sudden onset and laboratory results that include hyperkalemia, hyponatremia, and normal anion gap metabolic acidosis raise suspicion of adrenal crisis resulting in acute mineralocorticoid and glucocorticoid insufficiency.1

The patient’s elevated serum creatinine and high blood urea nitrogen-to-creatinine ratio of 26.9 (reference range 10–20) also suggest intravascular volume contraction. Her low hemoglobin level and supratherapeutic INR, possibly due to an interaction between warfarin and acetaminophen combined with poor medical follow-up, raise suspicion of acute bilateral adrenal necrosis due to hemorrhage.

Clinical manifestations of adrenal insufficiency by organ system
Adrenal crisis is a medical emergency that can lead to rapid deterioration and death if not diagnosed and treated promptly. Some of its manifestations (Table 2) are nonspecific and are common to various other conditions.Thus, its diagnosis requires a high index of suspicion.

Bilateral adrenal hemorrhage is one cause of adrenal crisis resulting in bilateral adrenal necrosis. Risk factors for adrenal hemorrhage include anticoagulation therapy, underlying coagulopathy, postoperative states, and certain infections such as meningococcemia and Haemophilus influenzae infection.2–5 Nevertheless, in most cases the INR is in the therapeutic range and the patient has no bleeding elsewhere.4 Other causes of adrenal necrosis include emboli, sepsis, and blunt trauma.6,7

Other causes of adrenal crisis are listed in Table 3.

Cardiogenic shock

Major causes of adrenal crisis
Cardiogenic shock is caused by decreased myocardial contractility, making the heart unable to adequately pump the returning blood. However, the metabolic disturbances in our patient and the finding of flat neck veins make this cause of shock less likely.

 

 

Intracranial hemorrhage

Intracranial hemorrhage can present with a decreased level of consciousness, but it is less likely to cause hypotension, as the cranial space is limited. If massive intracranial hemorrhage would occur, the increase in intracranial pressure would more likely cause hypertension by the Cushing reflex than hypotension.

Acute abdomen

Abdominal pain and rigidity along with fever can be presenting symptoms of both adrenal insufficiency and an acute abdomen due to intestinal obstruction.4 However, intestinal obstruction typically causes a high anion gap metabolic acidosis due to lactic acidosis, instead of the normal anion gap metabolic acidosis present in this patient.8 Moreover, her deranged electrolytes, supratherapeutic INR, and absence of previous gastroenterologic conditions make adrenal crisis a more likely diagnosis.

Septic shock

Septic shock would also cause fever and hypotension as bacterial toxins induce a pyrexic response and vasodilation. However, at such an early stage of sepsis, the patient would be expected to be warm and hyperemic, whereas this patient’s skin is cool and dry due to volume depletion secondary to adrenal insufficiency.9 Sepsis would also cause a high anion gap metabolic acidosis due to lactic acidosis, as opposed to this patient’s normal anion gap metabolic acidosis. These findings, along with the metabolic derangements and the absence of a focus of infection, make sepsis a less likely possibility.

CASE CONTINUED: CARDIOMEGALY, PERSISTENT HYPOTENSION

Blood is drawn for cultures and measurement of troponins and lactic acid, and urine samples are taken for culture and biochemical analysis. Chest radiography shows mild cardiomegaly. The patient is started empirically on vancomycin and cefepime, and her warfarin is discontinued.

Five hours after presenting to the emergency department, her blood pressure remains at 80/40 mm Hg even after receiving 3 L of normal saline intravenously.

PROMPT MANAGEMENT OF ADRENAL CRISIS

2. Which of the following is the most appropriate next step in managing this patient?

  • Draw samples for serum cortisol and plasma adrenocorticotropic hormone (ACTH) levels, then give hydrocortisone 100 mg intravenously
  • Perform abdominal computed tomography (CT) without contrast
  • Perform transthoracic echocardiography
  • Increase the norepinephrine infusion
  • Immediately give fludrocortisone

First give fluids

The first step in managing a patient with suspected adrenal crisis is liberal intravenous fluid administration to replenish the depleted intravascular space. The amount and choice of fluid is empiric, but a recommendation is 1 L of normal saline or dextrose 5% in normal saline, infused quickly over the first hour and then titrated according to the patient’s fluid status.10

Measure cortisol and ACTH; start corticosteroids immediately

Immediate therapy with an appropriate stress dose of intravenous corticosteroids (eg, hydrocortisone 100 mg) is essential. However, this should be done after drawing blood for cortisol and ACTH measurements.10

Do not delay corticosteroid therapy while awaiting the results of the diagnostic tests.

Adrenal insufficiency
Figure 1. Adrenal insufficiency is classified according to whether the defect lies in the adrenal gland (primary adrenal insufficiency) or centrally, ie, in the pituitary gland (secondary adrenal insufficiency) or hypothalamus (tertiary adrenal insufficiency).
Cortisol and ACTH levels are useful in assessing adrenal function in critically ill patients.11 While inappropriately low serum cortisol usually indicates adrenal insufficiency, measuring plasma ACTH is important to distinguish whether the adrenal insufficiency is primary (ie, due to dysfunction of the adrenal gland itself) or central, ie, either secondary (due to dysfunction of the pituitary gland) or tertiary (due to dysfunction of the hypothalamus). Primary adrenal insufficiency is associated with elevated plasma ACTH, whereas central (secondary or tertiary) adrenal insufficiency is associated with low or inappropriately normal plasma ACTH levels (Figure 1).

In addition, in the early phase of evolving primary adrenal insufficiency, measurement of plasma renin and aldosterone levels may be beneficial, as mineralocorticoid deficiency may predominate.10,12,13

One of the most important aims of early corticosteroid supplementation is to prevent further hyponatremia by reducing a reactive increase in antidiuretic hormone secretion caused by cortisol deficiency. Corticosteroids also help to restore normal blood pressure by increasing vascular tone, as glucocorticoid receptor activation potentiates the vasoconstrictor actions of norepinephrine, angiotensin II, and other vasoconstrictors.14,15

Which corticosteroid to use?

Which corticosteroid to use in previously undiagnosed adrenal insufficiency is controversial. The Endocrine Society10 and Japan Endocrine Society16 clinical practice guidelines recommend hydrocortisone in a 100-mg intravenous bolus followed by 200 mg over 24 hours.

The choice of hydrocortisone is justified by its superior mineralocorticoid activity.10,16 Further, hydrocortisone is preferred over dexamethasone if the patient is known to have primary adrenal insufficiency, or if the serum potassium level is higher than 6.0 mmol/L.

Some clinicians, on the other hand, recommend dexamethasone, given as a 4-mg intravenous bolus followed by 4-mg boluses every 12 hours. Their rationale is that dexamethasone, unlike hydrocortisone, does not interfere with subsequent serum cortisol assays if the patient later undergoes ACTH stimulation testing.17 Dexamethasone may also be preferred to minimize unwanted mineralocorticoid effects, such as in neurosurgical patients at risk of brain edema.

If hydrocortisone is used, ACTH stimulation testing can be done after withholding hydrocortisone for 24 hours once the patient is stable. (It should be restarted after the test if the results are abnormal.)

 

 

Other possible steps

Abdominal CT should be done in our patient to address the possibility of bilateral adrenal hemorrhage. However, it is preferable to wait until the patient is stabilized.

Echocardiography. Our patient is likely to have an element of cardiac failure, given her hypertension and cardiomegaly. However, decompensated heart failure is probably not the cause of her presentation. Thus, the first priority is to treat her adrenal crisis, and echocardiography should be deferred.

Increasing the norepinephrine infusion is unlikely to improve her blood pressure very much, as she is significantly volume-depleted. Further, low cortisol decreases the vascular response to norepinephrine.15

Mineralocorticoids such as fludrocortisone are used to treat primary adrenal insufficiency. However, they are not required during acute management of adrenal crisis, as 40 mg of hydrocortisone offers mineralocorticoid activity equivalent to 100 µg of fludrocortisone. Thus, the high doses of hydrocortisone used to treat adrenal crisis provide adequate mineralocorticoid therapy.10,18

If dexamethasone is used, its effect along with normal saline supplementation would be sufficient to replete the intravenous space and bring the sodium level back up to normal in the acute setting.

CASE RESUMED: IMPROVEMENT WITH HYDROCORTISONE

The patient’s blood is drawn for serum cortisol and plasma ACTH measurements. A 100-mg intravenous bolus of hydrocortisone is given, followed by a 50-mg bolus every 6 hours until the patient stabilizes.

Twenty-four hours later, the patient states that she has more energy, and her appetite has improved. The norepinephrine infusion is stopped 48 hours after presentation, at which time her blood pressure is 120/70 mm Hg, heart rate 85 beats per minute and irregular, and temperature 36.7°C (98.1°F). Her current laboratory values include the following:

  • Serum sodium 137 mmolL
  • Serum potassium 4.3 mmol/L
  • Hemoglobin 9.3 g/dL
  • Serum cortisol (random) 7.2 μg/dL
  • Plasma ACTH 752 pg/mL (10–60 pg/mL).

ESTABLISHING THE DIAGNOSIS OF ADRENAL INSUFFICIENCY

3. Which of the following is the most appropriate test to establish the diagnosis of adrenal insufficiency?

  • 7 am total serum cortisol measurement
  • Random serum cortisol measurement
  • 7 am salivary cortisol measurement
  • 24-hour urinary free cortisol measurement
  • ACTH stimulation test for cortisol
  • Insulin tolerance test for cortisol

Laboratory findings in adrenal insufficiency
Adrenal insufficiency can present acutely with catastrophic outcomes, such as in adrenal crisis. Alternatively, it can present insidiously with multiple vague manifestations and nonspecific laboratory findings (Table 4). But even when the diagnosis of adrenal insufficiency is apparent, laboratory tests are required for confirmation.

These tests also help determine the type of adrenal insufficiency (primary, secondary, or tertiary) and guide further management. Secondary adrenal insufficiency is caused by inadequate pituitary ACTH secretion and subsequent inadequate cortisol production, whereas tertiary adrenal insufficiency is caused by inadequate hypothalamic corticotropin-releasing hormone secretion and subsequent inadequate ACTH and cortisol production. The diagnosis of adrenal insufficiency relies first on demonstrating inappropriately low total serum cortisol production. Subsequently, serum ACTH helps to differentiate between primary (high ACTH) and secondary or tertiary (low or inappropriately normal ACTH) adrenal insufficiency.

Each test listed above may demonstrate a low cortisol level. However, in a nonacute setting, safety concerns (especially regarding insulin tolerance testing), poor diagnostic value, feasibility (ie, the difficulty of 24-hour tests), and poor sensitivity of 7 am cortisol make the ACTH stimulation test the most appropriate test in clinical practice to establish the diagnosis of adrenal insufficiency.

7 am serum cortisol measurement

Measuring the serum cortisol level early in the morning in the nonacute setting could be of diagnostic value, as an extremely low value (< 3–5 μg/dL) is almost 100% specific for adrenal insufficiency in the absence of concurrent exogenous steroid intake. However, the very low cutoff for this test causes poor sensitivity (about 33%), as many patients have partial adrenal insufficiency and hence have higher serum cortisol levels that may even be in the normal physiologic range.19–22

Random serum cortisol measurements

Random serum cortisol measurements are not very useful in a nonacute setting, since cortisol levels are affected by factors such as stress and hydration status. Moreover, they fluctuate during the day in a circadian rhythm.

On the other hand, random serum cortisol is a very good test to evaluate for adrenal insufficiency in the acute setting. A random value higher than 15 to 18 μg/dL is almost always associated with adequate adrenal function and generally rules out adrenal insufficiency.11,23,24

 

 

7 am salivary cortisol measurement

The same principle applies to early morning salivary cortisol. Only extremely low values (< 2.65 ng/mL) may distinguish patients with adrenal insufficiency from healthy individuals, with 97.1% sensitivity and 93.3% specificity.25

Of note, early morning salivary cortisol is not routinely measured in most clinical practices for evaluation of adrenal function. Hence, morning serum and morning salivary cortisol are useful screening tools and have meaningful results when their values are in the extremes of the spectrum, but they are not reliable as a single test, as they may overlook patients with partial adrenal insufficiency.

Urinary cortisol measurement

Urinary cortisol measurement is not used to diagnose adrenal insufficiency, as values can be normal in patients with partial adrenal insufficiency.

The ACTH stimulation test

The ACTH stimulation test involves an intramuscular or intravenous injection of cosyntropin (a synthetic analogue of ACTH fragment 1–24 that has the full activity of native ACTH) and measuring total serum cortisol at baseline, 30 minutes, and 60 minutes to assess the response of the adrenal glands.

The test can be done using a high or low dose of cosyntropin. The Endocrine Society’s 2016 guidelines recommend the high dose (250 μg) for most patients.10 The standard high-dose stimulation test can be done at any time during the day.26 If the cosyntropin is injected intravenously, any value higher than 18 to 20 μg/dL indicates normal adrenal function and excludes adrenal insufficiency.27,28 If intramuscular injection is used, any value higher than 16 to 18 μg/dL at 30 minutes post-consyntropin excludes adrenal insufficiency.29

The ACTH stimulation test may not exclude acute secondary or tertiary adrenal insufficiency.

Insulin tolerance testing

Insulin tolerance testing remains the gold standard for diagnosing adrenal insufficiency and assessing the integrity of the pituitary-adrenal axis. However, given its difficulty to perform, safety concerns, and the availability of other reliable tests, its use in clinical practice is limited. It is nonetheless useful in assessing patients with recent onset of ACTH deficiency.30,31

CASE RESUMED: PATIENT DISCHARGED, LOST TO FOLLOW-UP

Abdominal CT without contrast is done and demonstrates bilateral adrenal hemorrhage. Thus, the patient is diagnosed with primary acute adrenal insufficiency due to adrenal necrosis.

She is started on oral hydrocortisone and fludrocortisone after intravenous hydrocortisone is discontinued. She is counseled about adhering to medications, wearing a medical alert bracelet, giving herself emergency cortisol injections, taking higher doses of hydrocortisone if she is ill, and monitoring her INR. She is discharged home after her symptoms resolve.

The patient does not keep her scheduled appointment and is lost to follow-up. She returns 2 years later complaining of fatigue and feeling unwell. She admits that she stopped taking hydrocortisone 1 year ago after reading an online article about corticosteroid side effects. She has continued to take fludrocortisone.

MINERALOCORTICOID VS CORTICOSTEROID DEFICIENCY

Clinical features of primary vs central adrenal insufficiency
Our patient has primary adrenal insufficiency. The presentations of primary and central (secondary or tertiary) adrenal insufficiency are similar, but there are critical differences (Table 5). Further, she has been taking her mineralocorticoid (fludrocortisone) replacement but has stopped taking her corticosteroid (hydrocortisone).

4. Which of the following is least likely to be present in this patient at this time?

  • Intravascular volume depletion
  • Hyponatremia
  • Skin hyperpigmentation
  • Normokalemia
  • Elevated serum ACTH level

Intravascular volume depletion

Intravascular volume depletion is the least likely to be present. This is because intravascular volume depletion is mainly secondary to mineralocorticoid deficiency rather than corticosteroid deficiency, which is not present in this patient, as she is compliant with her mineralocorticoid replacement therapy.32,33 However, even with sufficient mineralocorticoid replacement, mild hypotension may be present in this patient due to corticosteroid deficiency-induced loss of vascular tone.

Hyponatremia

Hyponatremia in adrenal insufficiency is not due only to mineralocorticoid deficiency. Patients with secondary or tertiary adrenal insufficiency may also exhibit hyponatremia.34 ACTH deficiency in such patients is not expected to cause mineralocorticoid deficiency, as ACTH has only a minor role in aldosterone production.

It has been proposed that hyponatremia in secondary adrenal insufficiency is due to cortisol deficiency resulting in an increase of antidiuretic hormone secretion.35,36 The mechanisms for increased antidiuretic hormone include cortisol deficiency resulting in an increased corticotropin-releasing hormone level, which acts as an antidiuretic hormone secretagogue,37,38 and cortisol directly suppressing antidiuretic hormone secretion.39

In our patient, volume expansion and hyponatremia are expected due to increased antidiuretic hormone secretion as a result of corticosteroid insufficiency.

 

 

Hyperpigmentation

Hyperpigmentation of the skin is present only in long-standing primary adrenal insufficiency. This is due to chronic cortisol deficiency causing an increased secretion of pro-opiomelanocortin, a prohormone that is cleaved into ACTH, melanocyte-stimulating hormone, and other hormones. Melanocyte-stimulating hormone causes skin hyperpigmentation due to increased melanin synthesis.40 The hyperpigmentation is seen in sun-exposed areas, pressure areas, palmar creases, nipples, and mucous membranes.

This patient has long-standing corticosteroid deficiency due to noncompliance and primary adrenal insufficiency, and as a result she is expected to have elevated serum ACTH and hyperpigmentation.

Normokalemia

Mineralocorticoid deficiency results in hyperkalemia and metabolic acidosis by impairing renal excretion of potassium and acid.41 This patient is compliant with her mineralocorticoid replacement regimen; thus, potassium levels and pH are expected to be normal.

TAKE-HOME POINTS

  • Suspect adrenal crisis in any patient who presents with shock.
  • Acute abdomen or unexplained fever could be among the manifestations.
  • Initial management requires liberal normal saline intravenous fluid administration to replete the intravascular space.
  • Draw blood samples for serum chemistry, cortisol, and ACTH, followed immediately by intravenous hydrocortisone supplementation.
  • In critically ill patients, evaluate adrenal function with random serum cortisol; in a nonacute setting use the ACTH stimulation test.
  • Chronic management of primary adrenal insufficiency requires corticosteroid and mineralocorticoid therapy.
References
  1. Mani S, Rutecki GW. A patient with altered mental status and an acid-base disturbance. Cleve Clin J Med 2017; 84(1):27–34. doi:10.3949/ccjm.84a.16042
  2. Almiani M, Gorthi J, Subbiah S, Firoz M. Quiz page November 2012: an unusual case of acute hyponatremia and normal anion gap metabolic acidosis. Am J Kidney Dis 2012; 60(5):xxxiii–xxxvi. doi:10.1053/j.ajkd.2012.05.026
  3. Migeon CJ, Kenny FM, Hung W, Voorhess ML. Study of adrenal function in children with meningitis. Pediatrics 1967; 40(2):163–183.
  4. Rao RH, Vagnucci AH, Amico JA. Bilateral massive adrenal hemorrhage: early recognition and treatment. Ann Intern Med 1989; 110(3):227–235.
  5. Shimizu S, Tahara Y, Atsumi T, et al. Waterhouse-Friderichsen syndrome caused by invasive Haemophilus influenzae type B infection in a previously healthy young man. Anaesth Intensive Care 2010; 38(1):214–215.
  6. Castaldo ET, Guillamondegui OD, Greco JA 3rd, Feurer ID, Miller RS, Morris JA Jr. Are adrenal injuries predictive of adrenal insufficiency in patients sustaining blunt trauma? Am Surg 2008; 74(3):262–266.
  7. Xarli VP, Steele AA, Davis PJ, Buescher ES, Rios CN, Garcia-Bunuel R. Adrenal hemorrhage in the adult. Medicine (Baltimore) 1978; 57(3):211–221.
  8. Takeuchi K, Tsuzuki Y, Ando T, et al. Clinical studies of strangulating small bowel obstruction. Am Surg 2004; 70(1):40–44.
  9. MacKenzie IM. The haemodynamics of human septic shock. Anaesthesia 2001; 56(2):130–144.
  10. Bornstein SR, Allolio B, Arlt W, et al. Diagnosis and treatment of primary adrenal insufficiency: an Endocrine Society clinical practice guideline. J Clin Endocrinol Metab 2016; 101(2):364–389. doi:10.1210/jc.2015-1710
  11. Hamrahian AH, Fleseriu M; AACE Adrenal Scientific Committee. Evaluation and management of adrenal insufficiency in critically ill patients: disease state review. Endocr Pract 2017; 23(6):716–725. doi:10.4158/EP161720.RA
  12. Saenger P, Levine LS, Irvine WJ, et al. Progressive adrenal failure in polyglandular autoimmune disease. J Clin Endocrinol Metab 1982; 54(4):863–867.
  13. Coco G, Dal Pra C, Presotto F, et al. Estimated risk for developing autoimmune Addison's disease in patients with adrenal cortex autoantibodies. J Clin Endocrinol Metab 2006; 91(5):1637–1645. doi:10.1210/jc.2005-0860
  14. Ullian ME. The role of corticosteroids in the regulation of vascular tone. Cardiovasc Res 1999; 41(1):55–64.
  15. Yang S, Zhang L. Glucocorticoids and vascular reactivity. Curr Vasc Pharmacol 2004; 2(1):1–12.
  16. Yanase T, Tajima T, Katabami T, et al. Diagnosis and treatment of adrenal insufficiency including adrenal crisis: a Japan Endocrine Society clinical practice guideline [Opinion]. Endocr J 2016; 63(9):765–784. doi:10.1507/endocrj.EJ16-0242
  17. Taylor RL, Grebe SK, Singh RJ. Quantitative, highly sensitive liquid chromatography-tandem mass spectrometry method for detection of synthetic corticosteroids. Clin Chem 2004; 50(10):2345–2352. doi:10.1373/clinchem.2004.033605
  18. Goldfien A, Laidlaw JC, Haydar NA, Renold AE, Thorn GW. Fluorohydrocortisone and chlorohydrocortisone, highly potent derivatives of compound F. N Engl J Med 1955; 252(11):415–421. doi:10.1056/NEJM195503172521101
  19. Jenkins D, Forsham PH, Laidlaw JC, Reddy WJ, Thorn GW. Use of ACTH in the diagnosis of adrenal cortical insufficiency. Am J Med 1955; 18(1):3–14.
  20. Hägg E, Asplund K, Lithner F. Value of basal plasma cortisol assays in the assessment of pituitary-adrenal insufficiency. Clin Endocrinol (Oxf) 1987; 26(2):221–226.
  21. Deutschbein T, Unger N, Mann K, Petersenn S. Diagnosis of secondary adrenal insufficiency: unstimulated early morning cortisol in saliva and serum in comparison with the insulin tolerance test. Horm Metab Res 2009; 41(4):834–839. doi:10.1055/s-0029-1225630
  22. Erturk E, Jaffe CA, Barkan AL. Evaluation of the integrity of the hypothalamic-pituitary-adrenal axis by insulin hypoglycemia test. J Clin Endocrinol Metab 1998; 83(7):2350–2354.
  23. Cooper MS, Stewart PM. Corticosteroid insufficiency in acutely ill patients. N Engl J Med 2003; 348(8):727–734. doi:10.1056/NEJMra020529
  24. Dellinger RP, Levy MM, Rhodes A, et al; Surviving Sepsis Campaign Guidelines Committee including the Pediatric Subgroup. Surviving Sepsis Campaign: international guidelines for management of severe sepsis and septic shock, 2012. Intensive Care Med 2013; 39(2):165–228. doi:10.1007/s00134-012-2769-8
  25. Ceccato F, Barbot M, Zilio M, et al. Performance of salivary cortisol in the diagnosis of Cushing's syndrome, adrenal incidentaloma, and adrenal insufficiency. Eur J Endocrinol 2013; 169(1):31–36. doi:10.1530/EJE-13-0159
  26. Dickstein G, Shechner C, Nicholson WE, et al. Adrenocorticotropin stimulation test: effects of basal cortisol level, time of day, and suggested new sensitive low dose test. J Clin Endocrinol Metab 1991; 72(4):773–778. doi:10.1210/jcem-72-4-773
  27. May ME, Carey RM. Rapid adrenocorticotropic hormone test in practice. Retrospective review. Am J Med 1985; 79(6):679–884.
  28. Speckart PF, Nicoloff JT, Bethune JE. Screening for adrenocortical insufficiency with cosyntropin (synthetic ACTH). Arch Intern Med 1971; 128(5):761–763.
  29. Peechakara S, Bena J, Clarke NJ, et al. Total and free cortisol levels during 1 μg, 25 μg, and 250 μg cosyntropin stimulation tests compared to insulin tolerance test: results of a randomized, prospective, pilot study. Endocrine 2017; 57(3):388–393. doi:10.1007/s12020-017-1371-9
  30. Finucane FM, Liew A, Thornton E, Rogers B, Tormey W, Agha A. Clinical insights into the safety and utility of the insulin tolerance test (ITT) in the assessment of the hypothalamo-pituitary-adrenal axis. Clin Endocrinol (Oxf) 2008; 69(4):603–607. doi:10.1111/j.1365-2265.2008.03240.x
  31. Lindholm J, Kehlet H. Re-evaluation of the clinical value of the 30 min ACTH test in assessing the hypothalamic-pituitary-adrenocortical function. Clin Endocrinol (Oxf) 1987; 26(1):53–59.
  32. Charmandari E, Nicolaides NC, Chrousos GP. Adrenal insufficiency. Lancet 2014; 383(9935):2152–2167. doi:10.1016/S0140-6736(13)61684-0
  33. Burke CW. Adrenocortical insufficiency. Clin Endocrinol Metab 1985; 14(4):947–976.
  34. Jessani N, Jehangir W, Behman D, Yousif A, Spiler IJ. Secondary adrenal insufficiency: an overlooked cause of hyponatremia. J Clin Med Res 2015; 7(4):286–288. doi:10.14740/jocmr2041w
  35. Oelkers W. Hyponatremia and inappropriate secretion of vasopressin (antidiuretic hormone) in patients with hypopituitarism. N Engl J Med 1989; 321(8):492–496. doi:10.1056/NEJM198908243210802
  36. Ishikawa S, Schrier RW. Effect of arginine vasopressin antagonist on renal water excretion in glucocorticoid and mineralocorticoid deficient rats. Kidney Int 1982; 22(6):587–593.
  37. Wolfson B, Manning RW, Davis LG, Arentzen R, Baldino F Jr. Co-localization of corticotropin releasing factor and vasopressin mRNA in neurones after adrenalectomy. Nature 1985; 315(6014):59–61.
  38. Kalogeras KT, Nieman LK, Friedman TC, et al. Inferior petrosal sinus sampling in healthy subjects reveals a unilateral corticotropin-releasing hormone-induced arginine vasopressin release associated with ipsilateral adrenocorticotropin secretion. J Clin Invest 1996; 97:2045–2050.
  39. Kovacs KJ, Foldes A, Sawchenko PE. Glucocorticoid negative feedback selectively targets vasopressin transcription in parvocellular neurosecretory neurons. J Neurosci 2000; 20:3843–3852.
  40. Sarkar SB, Sarkar S, Ghosh S, Bandyopadhyay S. Addison's disease. Contemp Clin Dent 2012; 3(4):484–486. doi:10.4103/0976-237X.107450
  41. Szylman P, Better OS, Chaimowitz C, Rosler A. Role of hyperkalemia in the metabolic acidosis of isolated hypoaldosteronism. N Engl J Med 1976; 294(7):361–365. doi:10.1056/NEJM197602122940703
Article PDF
Author and Disclosure Information

Raja Y. Zaghlol, MD
Georgetown University/Washington Hospital Center, Department of Internal Medicine, Washington, DC

Michael E. Tierney, MD, BMedSc
Sydney University Orange Health Service, New South Wales, Australia

Louay Y. Zaghlol
School of Medicine, The University of Jordan, Amman, Jordan

Ayman A. Zayed, MD, MSc, FACE, FACP
Chief, Division of Endocrinology, Diabetes, and Metabolism, Department of Internal Medicine, School of Medicine, The University of Jordan, Jordan University Hospital, Amman, Jordan

Address: Ayman A. Zayed, MD, MSc, FACE, FACP, PO Box 13046, Amman 11942, Jordan; baraaayman@gmail.com

Issue
Cleveland Clinic Journal of Medicine - 85(4)
Publications
Topics
Page Number
303-312
Legacy Keywords
adrenal crisis, adrenal failure, shock, hypotension, atrial fibrillation, anticoagulation, warfarin, acetaminophen, interaction, prothrombin time, INR, international normalized ratio, adrenal hemorrhage, cortisone, ACTH, adrenocoticotropic hormone, ACTH stimulation test, Raja Zaghlol, Michael Tierney, Louay Zaghlol, Ayman Zayed
Sections
Author and Disclosure Information

Raja Y. Zaghlol, MD
Georgetown University/Washington Hospital Center, Department of Internal Medicine, Washington, DC

Michael E. Tierney, MD, BMedSc
Sydney University Orange Health Service, New South Wales, Australia

Louay Y. Zaghlol
School of Medicine, The University of Jordan, Amman, Jordan

Ayman A. Zayed, MD, MSc, FACE, FACP
Chief, Division of Endocrinology, Diabetes, and Metabolism, Department of Internal Medicine, School of Medicine, The University of Jordan, Jordan University Hospital, Amman, Jordan

Address: Ayman A. Zayed, MD, MSc, FACE, FACP, PO Box 13046, Amman 11942, Jordan; baraaayman@gmail.com

Author and Disclosure Information

Raja Y. Zaghlol, MD
Georgetown University/Washington Hospital Center, Department of Internal Medicine, Washington, DC

Michael E. Tierney, MD, BMedSc
Sydney University Orange Health Service, New South Wales, Australia

Louay Y. Zaghlol
School of Medicine, The University of Jordan, Amman, Jordan

Ayman A. Zayed, MD, MSc, FACE, FACP
Chief, Division of Endocrinology, Diabetes, and Metabolism, Department of Internal Medicine, School of Medicine, The University of Jordan, Jordan University Hospital, Amman, Jordan

Address: Ayman A. Zayed, MD, MSc, FACE, FACP, PO Box 13046, Amman 11942, Jordan; baraaayman@gmail.com

Article PDF
Article PDF
Related Articles

A 71-year-old woman is brought to the emergency department by her neighbor after complaining of fatigue and light-headedness for the last 8 hours. The patient lives alone and was feeling well when she woke up this morning, but then began to feel nauseated and vomited twice.

The patient appears drowsy and confused and cannot provide any further history. Her medical records show that she was seen in the cardiology clinic 6 months ago but has not kept her appointments since then.

Her medical history includes atrial fibrillation, hypertension, type 2 diabetes mellitus, and osteoarthritis. Her medications are daily warfarin, atenolol, aspirin, candesartan, and metformin, and she takes acetaminophen as needed. She is neither a smoker nor a drug user, but she drinks alcohol occasionally. Her family history is significant for her mother’s death from breast cancer at age 55.

The neighbor confirms that the patient appeared well this morning and has not had any recent illnesses except for a minor cold last week that improved over 5 days with acetaminophen only.

INITIAL EVALUATION AND MANAGEMENT

Physical examination

On physical examination, her blood pressure is 80/40 mm Hg, respiratory rate 25 breaths per minute, oral temperature 38.3°C (100.9°F), and heart rate 130 beats per minute and irregular.

Her neck veins are flat, and her chest is clear to auscultation with normal heart sounds. Abdominal palpation elicits discomfort in the middle segments, voluntary withdrawal, and abdominal wall rigidity. Her skin feels dry and cool, with decreased turgor.

Initial treatment

The patient is given 1 L of 0.9% saline intravenously over the first hour and then is transferred to the intensive care unit, where a norepinephrine drip is started to treat her ongoing hypotension. Normal saline is continued at a rate of 500 mL per hour for the next 4 hours.

Cardiac monitoring and 12-lead electrocardiography show atrial fibrillation with a rapid ventricular response of 138 beats per minute, but electrical cardioversion is not done.

Initial laboratory tests

Initial laboratory results
Results of basic laboratory tests in the emergency department are shown in Table 1.

Of note, her international normalized ratio (INR) is 6.13, while the therapeutic range for a patient taking warfarin because of atrial fibrillation is 2.0 to 3.0.

Her blood pH is 7.34 (reference range 7.35–7.45), and her bicarbonate level is 18 mmol/L (22–26); a low pH and low bicarbonate together indicate metabolic acidosis. Her sodium level is 128 mmol/L (135–145), her chloride level is 100 mmol/L (97–107), and, as mentioned, her bicarbonate level is 18 mmol/L; therefore, her anion gap is 128 – (100 + 18) = 10 mmol/L, which is normal (≤ 10).1

Her serum creatinine level is 1.3 mg/dL (0.5–1.1), and her blood urea nitrogen level is 35 mg/dL (7–20).

Her potassium level is 5.8 mmol/L, which is consistent with hyperkalemia (reference range 3.5–5.2).

DIFFERENTIAL DIAGNOSIS

1. Which of the following is the most likely cause of this patient’s symptoms?

  • Adrenal crisis
  • Cardiogenic shock due to decreased cardiac contractility
  • Intracranial hemorrhage
  • Acute abdomen due to small bowel obstruction
  • Septic shock due to bacterial toxin-induced loss of vascular tone

Our patient is presenting with shock. Given our inability to obtain a meaningful history, the differential diagnosis is broad and includes all of the above.

Adrenal crisis

The sudden onset and laboratory results that include hyperkalemia, hyponatremia, and normal anion gap metabolic acidosis raise suspicion of adrenal crisis resulting in acute mineralocorticoid and glucocorticoid insufficiency.1

The patient’s elevated serum creatinine and high blood urea nitrogen-to-creatinine ratio of 26.9 (reference range 10–20) also suggest intravascular volume contraction. Her low hemoglobin level and supratherapeutic INR, possibly due to an interaction between warfarin and acetaminophen combined with poor medical follow-up, raise suspicion of acute bilateral adrenal necrosis due to hemorrhage.

Clinical manifestations of adrenal insufficiency by organ system
Adrenal crisis is a medical emergency that can lead to rapid deterioration and death if not diagnosed and treated promptly. Some of its manifestations (Table 2) are nonspecific and are common to various other conditions.Thus, its diagnosis requires a high index of suspicion.

Bilateral adrenal hemorrhage is one cause of adrenal crisis resulting in bilateral adrenal necrosis. Risk factors for adrenal hemorrhage include anticoagulation therapy, underlying coagulopathy, postoperative states, and certain infections such as meningococcemia and Haemophilus influenzae infection.2–5 Nevertheless, in most cases the INR is in the therapeutic range and the patient has no bleeding elsewhere.4 Other causes of adrenal necrosis include emboli, sepsis, and blunt trauma.6,7

Other causes of adrenal crisis are listed in Table 3.

Cardiogenic shock

Major causes of adrenal crisis
Cardiogenic shock is caused by decreased myocardial contractility, making the heart unable to adequately pump the returning blood. However, the metabolic disturbances in our patient and the finding of flat neck veins make this cause of shock less likely.

 

 

Intracranial hemorrhage

Intracranial hemorrhage can present with a decreased level of consciousness, but it is less likely to cause hypotension, as the cranial space is limited. If massive intracranial hemorrhage would occur, the increase in intracranial pressure would more likely cause hypertension by the Cushing reflex than hypotension.

Acute abdomen

Abdominal pain and rigidity along with fever can be presenting symptoms of both adrenal insufficiency and an acute abdomen due to intestinal obstruction.4 However, intestinal obstruction typically causes a high anion gap metabolic acidosis due to lactic acidosis, instead of the normal anion gap metabolic acidosis present in this patient.8 Moreover, her deranged electrolytes, supratherapeutic INR, and absence of previous gastroenterologic conditions make adrenal crisis a more likely diagnosis.

Septic shock

Septic shock would also cause fever and hypotension as bacterial toxins induce a pyrexic response and vasodilation. However, at such an early stage of sepsis, the patient would be expected to be warm and hyperemic, whereas this patient’s skin is cool and dry due to volume depletion secondary to adrenal insufficiency.9 Sepsis would also cause a high anion gap metabolic acidosis due to lactic acidosis, as opposed to this patient’s normal anion gap metabolic acidosis. These findings, along with the metabolic derangements and the absence of a focus of infection, make sepsis a less likely possibility.

CASE CONTINUED: CARDIOMEGALY, PERSISTENT HYPOTENSION

Blood is drawn for cultures and measurement of troponins and lactic acid, and urine samples are taken for culture and biochemical analysis. Chest radiography shows mild cardiomegaly. The patient is started empirically on vancomycin and cefepime, and her warfarin is discontinued.

Five hours after presenting to the emergency department, her blood pressure remains at 80/40 mm Hg even after receiving 3 L of normal saline intravenously.

PROMPT MANAGEMENT OF ADRENAL CRISIS

2. Which of the following is the most appropriate next step in managing this patient?

  • Draw samples for serum cortisol and plasma adrenocorticotropic hormone (ACTH) levels, then give hydrocortisone 100 mg intravenously
  • Perform abdominal computed tomography (CT) without contrast
  • Perform transthoracic echocardiography
  • Increase the norepinephrine infusion
  • Immediately give fludrocortisone

First give fluids

The first step in managing a patient with suspected adrenal crisis is liberal intravenous fluid administration to replenish the depleted intravascular space. The amount and choice of fluid is empiric, but a recommendation is 1 L of normal saline or dextrose 5% in normal saline, infused quickly over the first hour and then titrated according to the patient’s fluid status.10

Measure cortisol and ACTH; start corticosteroids immediately

Immediate therapy with an appropriate stress dose of intravenous corticosteroids (eg, hydrocortisone 100 mg) is essential. However, this should be done after drawing blood for cortisol and ACTH measurements.10

Do not delay corticosteroid therapy while awaiting the results of the diagnostic tests.

Adrenal insufficiency
Figure 1. Adrenal insufficiency is classified according to whether the defect lies in the adrenal gland (primary adrenal insufficiency) or centrally, ie, in the pituitary gland (secondary adrenal insufficiency) or hypothalamus (tertiary adrenal insufficiency).
Cortisol and ACTH levels are useful in assessing adrenal function in critically ill patients.11 While inappropriately low serum cortisol usually indicates adrenal insufficiency, measuring plasma ACTH is important to distinguish whether the adrenal insufficiency is primary (ie, due to dysfunction of the adrenal gland itself) or central, ie, either secondary (due to dysfunction of the pituitary gland) or tertiary (due to dysfunction of the hypothalamus). Primary adrenal insufficiency is associated with elevated plasma ACTH, whereas central (secondary or tertiary) adrenal insufficiency is associated with low or inappropriately normal plasma ACTH levels (Figure 1).

In addition, in the early phase of evolving primary adrenal insufficiency, measurement of plasma renin and aldosterone levels may be beneficial, as mineralocorticoid deficiency may predominate.10,12,13

One of the most important aims of early corticosteroid supplementation is to prevent further hyponatremia by reducing a reactive increase in antidiuretic hormone secretion caused by cortisol deficiency. Corticosteroids also help to restore normal blood pressure by increasing vascular tone, as glucocorticoid receptor activation potentiates the vasoconstrictor actions of norepinephrine, angiotensin II, and other vasoconstrictors.14,15

Which corticosteroid to use?

Which corticosteroid to use in previously undiagnosed adrenal insufficiency is controversial. The Endocrine Society10 and Japan Endocrine Society16 clinical practice guidelines recommend hydrocortisone in a 100-mg intravenous bolus followed by 200 mg over 24 hours.

The choice of hydrocortisone is justified by its superior mineralocorticoid activity.10,16 Further, hydrocortisone is preferred over dexamethasone if the patient is known to have primary adrenal insufficiency, or if the serum potassium level is higher than 6.0 mmol/L.

Some clinicians, on the other hand, recommend dexamethasone, given as a 4-mg intravenous bolus followed by 4-mg boluses every 12 hours. Their rationale is that dexamethasone, unlike hydrocortisone, does not interfere with subsequent serum cortisol assays if the patient later undergoes ACTH stimulation testing.17 Dexamethasone may also be preferred to minimize unwanted mineralocorticoid effects, such as in neurosurgical patients at risk of brain edema.

If hydrocortisone is used, ACTH stimulation testing can be done after withholding hydrocortisone for 24 hours once the patient is stable. (It should be restarted after the test if the results are abnormal.)

 

 

Other possible steps

Abdominal CT should be done in our patient to address the possibility of bilateral adrenal hemorrhage. However, it is preferable to wait until the patient is stabilized.

Echocardiography. Our patient is likely to have an element of cardiac failure, given her hypertension and cardiomegaly. However, decompensated heart failure is probably not the cause of her presentation. Thus, the first priority is to treat her adrenal crisis, and echocardiography should be deferred.

Increasing the norepinephrine infusion is unlikely to improve her blood pressure very much, as she is significantly volume-depleted. Further, low cortisol decreases the vascular response to norepinephrine.15

Mineralocorticoids such as fludrocortisone are used to treat primary adrenal insufficiency. However, they are not required during acute management of adrenal crisis, as 40 mg of hydrocortisone offers mineralocorticoid activity equivalent to 100 µg of fludrocortisone. Thus, the high doses of hydrocortisone used to treat adrenal crisis provide adequate mineralocorticoid therapy.10,18

If dexamethasone is used, its effect along with normal saline supplementation would be sufficient to replete the intravenous space and bring the sodium level back up to normal in the acute setting.

CASE RESUMED: IMPROVEMENT WITH HYDROCORTISONE

The patient’s blood is drawn for serum cortisol and plasma ACTH measurements. A 100-mg intravenous bolus of hydrocortisone is given, followed by a 50-mg bolus every 6 hours until the patient stabilizes.

Twenty-four hours later, the patient states that she has more energy, and her appetite has improved. The norepinephrine infusion is stopped 48 hours after presentation, at which time her blood pressure is 120/70 mm Hg, heart rate 85 beats per minute and irregular, and temperature 36.7°C (98.1°F). Her current laboratory values include the following:

  • Serum sodium 137 mmolL
  • Serum potassium 4.3 mmol/L
  • Hemoglobin 9.3 g/dL
  • Serum cortisol (random) 7.2 μg/dL
  • Plasma ACTH 752 pg/mL (10–60 pg/mL).

ESTABLISHING THE DIAGNOSIS OF ADRENAL INSUFFICIENCY

3. Which of the following is the most appropriate test to establish the diagnosis of adrenal insufficiency?

  • 7 am total serum cortisol measurement
  • Random serum cortisol measurement
  • 7 am salivary cortisol measurement
  • 24-hour urinary free cortisol measurement
  • ACTH stimulation test for cortisol
  • Insulin tolerance test for cortisol

Laboratory findings in adrenal insufficiency
Adrenal insufficiency can present acutely with catastrophic outcomes, such as in adrenal crisis. Alternatively, it can present insidiously with multiple vague manifestations and nonspecific laboratory findings (Table 4). But even when the diagnosis of adrenal insufficiency is apparent, laboratory tests are required for confirmation.

These tests also help determine the type of adrenal insufficiency (primary, secondary, or tertiary) and guide further management. Secondary adrenal insufficiency is caused by inadequate pituitary ACTH secretion and subsequent inadequate cortisol production, whereas tertiary adrenal insufficiency is caused by inadequate hypothalamic corticotropin-releasing hormone secretion and subsequent inadequate ACTH and cortisol production. The diagnosis of adrenal insufficiency relies first on demonstrating inappropriately low total serum cortisol production. Subsequently, serum ACTH helps to differentiate between primary (high ACTH) and secondary or tertiary (low or inappropriately normal ACTH) adrenal insufficiency.

Each test listed above may demonstrate a low cortisol level. However, in a nonacute setting, safety concerns (especially regarding insulin tolerance testing), poor diagnostic value, feasibility (ie, the difficulty of 24-hour tests), and poor sensitivity of 7 am cortisol make the ACTH stimulation test the most appropriate test in clinical practice to establish the diagnosis of adrenal insufficiency.

7 am serum cortisol measurement

Measuring the serum cortisol level early in the morning in the nonacute setting could be of diagnostic value, as an extremely low value (< 3–5 μg/dL) is almost 100% specific for adrenal insufficiency in the absence of concurrent exogenous steroid intake. However, the very low cutoff for this test causes poor sensitivity (about 33%), as many patients have partial adrenal insufficiency and hence have higher serum cortisol levels that may even be in the normal physiologic range.19–22

Random serum cortisol measurements

Random serum cortisol measurements are not very useful in a nonacute setting, since cortisol levels are affected by factors such as stress and hydration status. Moreover, they fluctuate during the day in a circadian rhythm.

On the other hand, random serum cortisol is a very good test to evaluate for adrenal insufficiency in the acute setting. A random value higher than 15 to 18 μg/dL is almost always associated with adequate adrenal function and generally rules out adrenal insufficiency.11,23,24

 

 

7 am salivary cortisol measurement

The same principle applies to early morning salivary cortisol. Only extremely low values (< 2.65 ng/mL) may distinguish patients with adrenal insufficiency from healthy individuals, with 97.1% sensitivity and 93.3% specificity.25

Of note, early morning salivary cortisol is not routinely measured in most clinical practices for evaluation of adrenal function. Hence, morning serum and morning salivary cortisol are useful screening tools and have meaningful results when their values are in the extremes of the spectrum, but they are not reliable as a single test, as they may overlook patients with partial adrenal insufficiency.

Urinary cortisol measurement

Urinary cortisol measurement is not used to diagnose adrenal insufficiency, as values can be normal in patients with partial adrenal insufficiency.

The ACTH stimulation test

The ACTH stimulation test involves an intramuscular or intravenous injection of cosyntropin (a synthetic analogue of ACTH fragment 1–24 that has the full activity of native ACTH) and measuring total serum cortisol at baseline, 30 minutes, and 60 minutes to assess the response of the adrenal glands.

The test can be done using a high or low dose of cosyntropin. The Endocrine Society’s 2016 guidelines recommend the high dose (250 μg) for most patients.10 The standard high-dose stimulation test can be done at any time during the day.26 If the cosyntropin is injected intravenously, any value higher than 18 to 20 μg/dL indicates normal adrenal function and excludes adrenal insufficiency.27,28 If intramuscular injection is used, any value higher than 16 to 18 μg/dL at 30 minutes post-consyntropin excludes adrenal insufficiency.29

The ACTH stimulation test may not exclude acute secondary or tertiary adrenal insufficiency.

Insulin tolerance testing

Insulin tolerance testing remains the gold standard for diagnosing adrenal insufficiency and assessing the integrity of the pituitary-adrenal axis. However, given its difficulty to perform, safety concerns, and the availability of other reliable tests, its use in clinical practice is limited. It is nonetheless useful in assessing patients with recent onset of ACTH deficiency.30,31

CASE RESUMED: PATIENT DISCHARGED, LOST TO FOLLOW-UP

Abdominal CT without contrast is done and demonstrates bilateral adrenal hemorrhage. Thus, the patient is diagnosed with primary acute adrenal insufficiency due to adrenal necrosis.

She is started on oral hydrocortisone and fludrocortisone after intravenous hydrocortisone is discontinued. She is counseled about adhering to medications, wearing a medical alert bracelet, giving herself emergency cortisol injections, taking higher doses of hydrocortisone if she is ill, and monitoring her INR. She is discharged home after her symptoms resolve.

The patient does not keep her scheduled appointment and is lost to follow-up. She returns 2 years later complaining of fatigue and feeling unwell. She admits that she stopped taking hydrocortisone 1 year ago after reading an online article about corticosteroid side effects. She has continued to take fludrocortisone.

MINERALOCORTICOID VS CORTICOSTEROID DEFICIENCY

Clinical features of primary vs central adrenal insufficiency
Our patient has primary adrenal insufficiency. The presentations of primary and central (secondary or tertiary) adrenal insufficiency are similar, but there are critical differences (Table 5). Further, she has been taking her mineralocorticoid (fludrocortisone) replacement but has stopped taking her corticosteroid (hydrocortisone).

4. Which of the following is least likely to be present in this patient at this time?

  • Intravascular volume depletion
  • Hyponatremia
  • Skin hyperpigmentation
  • Normokalemia
  • Elevated serum ACTH level

Intravascular volume depletion

Intravascular volume depletion is the least likely to be present. This is because intravascular volume depletion is mainly secondary to mineralocorticoid deficiency rather than corticosteroid deficiency, which is not present in this patient, as she is compliant with her mineralocorticoid replacement therapy.32,33 However, even with sufficient mineralocorticoid replacement, mild hypotension may be present in this patient due to corticosteroid deficiency-induced loss of vascular tone.

Hyponatremia

Hyponatremia in adrenal insufficiency is not due only to mineralocorticoid deficiency. Patients with secondary or tertiary adrenal insufficiency may also exhibit hyponatremia.34 ACTH deficiency in such patients is not expected to cause mineralocorticoid deficiency, as ACTH has only a minor role in aldosterone production.

It has been proposed that hyponatremia in secondary adrenal insufficiency is due to cortisol deficiency resulting in an increase of antidiuretic hormone secretion.35,36 The mechanisms for increased antidiuretic hormone include cortisol deficiency resulting in an increased corticotropin-releasing hormone level, which acts as an antidiuretic hormone secretagogue,37,38 and cortisol directly suppressing antidiuretic hormone secretion.39

In our patient, volume expansion and hyponatremia are expected due to increased antidiuretic hormone secretion as a result of corticosteroid insufficiency.

 

 

Hyperpigmentation

Hyperpigmentation of the skin is present only in long-standing primary adrenal insufficiency. This is due to chronic cortisol deficiency causing an increased secretion of pro-opiomelanocortin, a prohormone that is cleaved into ACTH, melanocyte-stimulating hormone, and other hormones. Melanocyte-stimulating hormone causes skin hyperpigmentation due to increased melanin synthesis.40 The hyperpigmentation is seen in sun-exposed areas, pressure areas, palmar creases, nipples, and mucous membranes.

This patient has long-standing corticosteroid deficiency due to noncompliance and primary adrenal insufficiency, and as a result she is expected to have elevated serum ACTH and hyperpigmentation.

Normokalemia

Mineralocorticoid deficiency results in hyperkalemia and metabolic acidosis by impairing renal excretion of potassium and acid.41 This patient is compliant with her mineralocorticoid replacement regimen; thus, potassium levels and pH are expected to be normal.

TAKE-HOME POINTS

  • Suspect adrenal crisis in any patient who presents with shock.
  • Acute abdomen or unexplained fever could be among the manifestations.
  • Initial management requires liberal normal saline intravenous fluid administration to replete the intravascular space.
  • Draw blood samples for serum chemistry, cortisol, and ACTH, followed immediately by intravenous hydrocortisone supplementation.
  • In critically ill patients, evaluate adrenal function with random serum cortisol; in a nonacute setting use the ACTH stimulation test.
  • Chronic management of primary adrenal insufficiency requires corticosteroid and mineralocorticoid therapy.

A 71-year-old woman is brought to the emergency department by her neighbor after complaining of fatigue and light-headedness for the last 8 hours. The patient lives alone and was feeling well when she woke up this morning, but then began to feel nauseated and vomited twice.

The patient appears drowsy and confused and cannot provide any further history. Her medical records show that she was seen in the cardiology clinic 6 months ago but has not kept her appointments since then.

Her medical history includes atrial fibrillation, hypertension, type 2 diabetes mellitus, and osteoarthritis. Her medications are daily warfarin, atenolol, aspirin, candesartan, and metformin, and she takes acetaminophen as needed. She is neither a smoker nor a drug user, but she drinks alcohol occasionally. Her family history is significant for her mother’s death from breast cancer at age 55.

The neighbor confirms that the patient appeared well this morning and has not had any recent illnesses except for a minor cold last week that improved over 5 days with acetaminophen only.

INITIAL EVALUATION AND MANAGEMENT

Physical examination

On physical examination, her blood pressure is 80/40 mm Hg, respiratory rate 25 breaths per minute, oral temperature 38.3°C (100.9°F), and heart rate 130 beats per minute and irregular.

Her neck veins are flat, and her chest is clear to auscultation with normal heart sounds. Abdominal palpation elicits discomfort in the middle segments, voluntary withdrawal, and abdominal wall rigidity. Her skin feels dry and cool, with decreased turgor.

Initial treatment

The patient is given 1 L of 0.9% saline intravenously over the first hour and then is transferred to the intensive care unit, where a norepinephrine drip is started to treat her ongoing hypotension. Normal saline is continued at a rate of 500 mL per hour for the next 4 hours.

Cardiac monitoring and 12-lead electrocardiography show atrial fibrillation with a rapid ventricular response of 138 beats per minute, but electrical cardioversion is not done.

Initial laboratory tests

Initial laboratory results
Results of basic laboratory tests in the emergency department are shown in Table 1.

Of note, her international normalized ratio (INR) is 6.13, while the therapeutic range for a patient taking warfarin because of atrial fibrillation is 2.0 to 3.0.

Her blood pH is 7.34 (reference range 7.35–7.45), and her bicarbonate level is 18 mmol/L (22–26); a low pH and low bicarbonate together indicate metabolic acidosis. Her sodium level is 128 mmol/L (135–145), her chloride level is 100 mmol/L (97–107), and, as mentioned, her bicarbonate level is 18 mmol/L; therefore, her anion gap is 128 – (100 + 18) = 10 mmol/L, which is normal (≤ 10).1

Her serum creatinine level is 1.3 mg/dL (0.5–1.1), and her blood urea nitrogen level is 35 mg/dL (7–20).

Her potassium level is 5.8 mmol/L, which is consistent with hyperkalemia (reference range 3.5–5.2).

DIFFERENTIAL DIAGNOSIS

1. Which of the following is the most likely cause of this patient’s symptoms?

  • Adrenal crisis
  • Cardiogenic shock due to decreased cardiac contractility
  • Intracranial hemorrhage
  • Acute abdomen due to small bowel obstruction
  • Septic shock due to bacterial toxin-induced loss of vascular tone

Our patient is presenting with shock. Given our inability to obtain a meaningful history, the differential diagnosis is broad and includes all of the above.

Adrenal crisis

The sudden onset and laboratory results that include hyperkalemia, hyponatremia, and normal anion gap metabolic acidosis raise suspicion of adrenal crisis resulting in acute mineralocorticoid and glucocorticoid insufficiency.1

The patient’s elevated serum creatinine and high blood urea nitrogen-to-creatinine ratio of 26.9 (reference range 10–20) also suggest intravascular volume contraction. Her low hemoglobin level and supratherapeutic INR, possibly due to an interaction between warfarin and acetaminophen combined with poor medical follow-up, raise suspicion of acute bilateral adrenal necrosis due to hemorrhage.

Clinical manifestations of adrenal insufficiency by organ system
Adrenal crisis is a medical emergency that can lead to rapid deterioration and death if not diagnosed and treated promptly. Some of its manifestations (Table 2) are nonspecific and are common to various other conditions.Thus, its diagnosis requires a high index of suspicion.

Bilateral adrenal hemorrhage is one cause of adrenal crisis resulting in bilateral adrenal necrosis. Risk factors for adrenal hemorrhage include anticoagulation therapy, underlying coagulopathy, postoperative states, and certain infections such as meningococcemia and Haemophilus influenzae infection.2–5 Nevertheless, in most cases the INR is in the therapeutic range and the patient has no bleeding elsewhere.4 Other causes of adrenal necrosis include emboli, sepsis, and blunt trauma.6,7

Other causes of adrenal crisis are listed in Table 3.

Cardiogenic shock

Major causes of adrenal crisis
Cardiogenic shock is caused by decreased myocardial contractility, making the heart unable to adequately pump the returning blood. However, the metabolic disturbances in our patient and the finding of flat neck veins make this cause of shock less likely.

 

 

Intracranial hemorrhage

Intracranial hemorrhage can present with a decreased level of consciousness, but it is less likely to cause hypotension, as the cranial space is limited. If massive intracranial hemorrhage would occur, the increase in intracranial pressure would more likely cause hypertension by the Cushing reflex than hypotension.

Acute abdomen

Abdominal pain and rigidity along with fever can be presenting symptoms of both adrenal insufficiency and an acute abdomen due to intestinal obstruction.4 However, intestinal obstruction typically causes a high anion gap metabolic acidosis due to lactic acidosis, instead of the normal anion gap metabolic acidosis present in this patient.8 Moreover, her deranged electrolytes, supratherapeutic INR, and absence of previous gastroenterologic conditions make adrenal crisis a more likely diagnosis.

Septic shock

Septic shock would also cause fever and hypotension as bacterial toxins induce a pyrexic response and vasodilation. However, at such an early stage of sepsis, the patient would be expected to be warm and hyperemic, whereas this patient’s skin is cool and dry due to volume depletion secondary to adrenal insufficiency.9 Sepsis would also cause a high anion gap metabolic acidosis due to lactic acidosis, as opposed to this patient’s normal anion gap metabolic acidosis. These findings, along with the metabolic derangements and the absence of a focus of infection, make sepsis a less likely possibility.

CASE CONTINUED: CARDIOMEGALY, PERSISTENT HYPOTENSION

Blood is drawn for cultures and measurement of troponins and lactic acid, and urine samples are taken for culture and biochemical analysis. Chest radiography shows mild cardiomegaly. The patient is started empirically on vancomycin and cefepime, and her warfarin is discontinued.

Five hours after presenting to the emergency department, her blood pressure remains at 80/40 mm Hg even after receiving 3 L of normal saline intravenously.

PROMPT MANAGEMENT OF ADRENAL CRISIS

2. Which of the following is the most appropriate next step in managing this patient?

  • Draw samples for serum cortisol and plasma adrenocorticotropic hormone (ACTH) levels, then give hydrocortisone 100 mg intravenously
  • Perform abdominal computed tomography (CT) without contrast
  • Perform transthoracic echocardiography
  • Increase the norepinephrine infusion
  • Immediately give fludrocortisone

First give fluids

The first step in managing a patient with suspected adrenal crisis is liberal intravenous fluid administration to replenish the depleted intravascular space. The amount and choice of fluid is empiric, but a recommendation is 1 L of normal saline or dextrose 5% in normal saline, infused quickly over the first hour and then titrated according to the patient’s fluid status.10

Measure cortisol and ACTH; start corticosteroids immediately

Immediate therapy with an appropriate stress dose of intravenous corticosteroids (eg, hydrocortisone 100 mg) is essential. However, this should be done after drawing blood for cortisol and ACTH measurements.10

Do not delay corticosteroid therapy while awaiting the results of the diagnostic tests.

Adrenal insufficiency
Figure 1. Adrenal insufficiency is classified according to whether the defect lies in the adrenal gland (primary adrenal insufficiency) or centrally, ie, in the pituitary gland (secondary adrenal insufficiency) or hypothalamus (tertiary adrenal insufficiency).
Cortisol and ACTH levels are useful in assessing adrenal function in critically ill patients.11 While inappropriately low serum cortisol usually indicates adrenal insufficiency, measuring plasma ACTH is important to distinguish whether the adrenal insufficiency is primary (ie, due to dysfunction of the adrenal gland itself) or central, ie, either secondary (due to dysfunction of the pituitary gland) or tertiary (due to dysfunction of the hypothalamus). Primary adrenal insufficiency is associated with elevated plasma ACTH, whereas central (secondary or tertiary) adrenal insufficiency is associated with low or inappropriately normal plasma ACTH levels (Figure 1).

In addition, in the early phase of evolving primary adrenal insufficiency, measurement of plasma renin and aldosterone levels may be beneficial, as mineralocorticoid deficiency may predominate.10,12,13

One of the most important aims of early corticosteroid supplementation is to prevent further hyponatremia by reducing a reactive increase in antidiuretic hormone secretion caused by cortisol deficiency. Corticosteroids also help to restore normal blood pressure by increasing vascular tone, as glucocorticoid receptor activation potentiates the vasoconstrictor actions of norepinephrine, angiotensin II, and other vasoconstrictors.14,15

Which corticosteroid to use?

Which corticosteroid to use in previously undiagnosed adrenal insufficiency is controversial. The Endocrine Society10 and Japan Endocrine Society16 clinical practice guidelines recommend hydrocortisone in a 100-mg intravenous bolus followed by 200 mg over 24 hours.

The choice of hydrocortisone is justified by its superior mineralocorticoid activity.10,16 Further, hydrocortisone is preferred over dexamethasone if the patient is known to have primary adrenal insufficiency, or if the serum potassium level is higher than 6.0 mmol/L.

Some clinicians, on the other hand, recommend dexamethasone, given as a 4-mg intravenous bolus followed by 4-mg boluses every 12 hours. Their rationale is that dexamethasone, unlike hydrocortisone, does not interfere with subsequent serum cortisol assays if the patient later undergoes ACTH stimulation testing.17 Dexamethasone may also be preferred to minimize unwanted mineralocorticoid effects, such as in neurosurgical patients at risk of brain edema.

If hydrocortisone is used, ACTH stimulation testing can be done after withholding hydrocortisone for 24 hours once the patient is stable. (It should be restarted after the test if the results are abnormal.)

 

 

Other possible steps

Abdominal CT should be done in our patient to address the possibility of bilateral adrenal hemorrhage. However, it is preferable to wait until the patient is stabilized.

Echocardiography. Our patient is likely to have an element of cardiac failure, given her hypertension and cardiomegaly. However, decompensated heart failure is probably not the cause of her presentation. Thus, the first priority is to treat her adrenal crisis, and echocardiography should be deferred.

Increasing the norepinephrine infusion is unlikely to improve her blood pressure very much, as she is significantly volume-depleted. Further, low cortisol decreases the vascular response to norepinephrine.15

Mineralocorticoids such as fludrocortisone are used to treat primary adrenal insufficiency. However, they are not required during acute management of adrenal crisis, as 40 mg of hydrocortisone offers mineralocorticoid activity equivalent to 100 µg of fludrocortisone. Thus, the high doses of hydrocortisone used to treat adrenal crisis provide adequate mineralocorticoid therapy.10,18

If dexamethasone is used, its effect along with normal saline supplementation would be sufficient to replete the intravenous space and bring the sodium level back up to normal in the acute setting.

CASE RESUMED: IMPROVEMENT WITH HYDROCORTISONE

The patient’s blood is drawn for serum cortisol and plasma ACTH measurements. A 100-mg intravenous bolus of hydrocortisone is given, followed by a 50-mg bolus every 6 hours until the patient stabilizes.

Twenty-four hours later, the patient states that she has more energy, and her appetite has improved. The norepinephrine infusion is stopped 48 hours after presentation, at which time her blood pressure is 120/70 mm Hg, heart rate 85 beats per minute and irregular, and temperature 36.7°C (98.1°F). Her current laboratory values include the following:

  • Serum sodium 137 mmolL
  • Serum potassium 4.3 mmol/L
  • Hemoglobin 9.3 g/dL
  • Serum cortisol (random) 7.2 μg/dL
  • Plasma ACTH 752 pg/mL (10–60 pg/mL).

ESTABLISHING THE DIAGNOSIS OF ADRENAL INSUFFICIENCY

3. Which of the following is the most appropriate test to establish the diagnosis of adrenal insufficiency?

  • 7 am total serum cortisol measurement
  • Random serum cortisol measurement
  • 7 am salivary cortisol measurement
  • 24-hour urinary free cortisol measurement
  • ACTH stimulation test for cortisol
  • Insulin tolerance test for cortisol

Laboratory findings in adrenal insufficiency
Adrenal insufficiency can present acutely with catastrophic outcomes, such as in adrenal crisis. Alternatively, it can present insidiously with multiple vague manifestations and nonspecific laboratory findings (Table 4). But even when the diagnosis of adrenal insufficiency is apparent, laboratory tests are required for confirmation.

These tests also help determine the type of adrenal insufficiency (primary, secondary, or tertiary) and guide further management. Secondary adrenal insufficiency is caused by inadequate pituitary ACTH secretion and subsequent inadequate cortisol production, whereas tertiary adrenal insufficiency is caused by inadequate hypothalamic corticotropin-releasing hormone secretion and subsequent inadequate ACTH and cortisol production. The diagnosis of adrenal insufficiency relies first on demonstrating inappropriately low total serum cortisol production. Subsequently, serum ACTH helps to differentiate between primary (high ACTH) and secondary or tertiary (low or inappropriately normal ACTH) adrenal insufficiency.

Each test listed above may demonstrate a low cortisol level. However, in a nonacute setting, safety concerns (especially regarding insulin tolerance testing), poor diagnostic value, feasibility (ie, the difficulty of 24-hour tests), and poor sensitivity of 7 am cortisol make the ACTH stimulation test the most appropriate test in clinical practice to establish the diagnosis of adrenal insufficiency.

7 am serum cortisol measurement

Measuring the serum cortisol level early in the morning in the nonacute setting could be of diagnostic value, as an extremely low value (< 3–5 μg/dL) is almost 100% specific for adrenal insufficiency in the absence of concurrent exogenous steroid intake. However, the very low cutoff for this test causes poor sensitivity (about 33%), as many patients have partial adrenal insufficiency and hence have higher serum cortisol levels that may even be in the normal physiologic range.19–22

Random serum cortisol measurements

Random serum cortisol measurements are not very useful in a nonacute setting, since cortisol levels are affected by factors such as stress and hydration status. Moreover, they fluctuate during the day in a circadian rhythm.

On the other hand, random serum cortisol is a very good test to evaluate for adrenal insufficiency in the acute setting. A random value higher than 15 to 18 μg/dL is almost always associated with adequate adrenal function and generally rules out adrenal insufficiency.11,23,24

 

 

7 am salivary cortisol measurement

The same principle applies to early morning salivary cortisol. Only extremely low values (< 2.65 ng/mL) may distinguish patients with adrenal insufficiency from healthy individuals, with 97.1% sensitivity and 93.3% specificity.25

Of note, early morning salivary cortisol is not routinely measured in most clinical practices for evaluation of adrenal function. Hence, morning serum and morning salivary cortisol are useful screening tools and have meaningful results when their values are in the extremes of the spectrum, but they are not reliable as a single test, as they may overlook patients with partial adrenal insufficiency.

Urinary cortisol measurement

Urinary cortisol measurement is not used to diagnose adrenal insufficiency, as values can be normal in patients with partial adrenal insufficiency.

The ACTH stimulation test

The ACTH stimulation test involves an intramuscular or intravenous injection of cosyntropin (a synthetic analogue of ACTH fragment 1–24 that has the full activity of native ACTH) and measuring total serum cortisol at baseline, 30 minutes, and 60 minutes to assess the response of the adrenal glands.

The test can be done using a high or low dose of cosyntropin. The Endocrine Society’s 2016 guidelines recommend the high dose (250 μg) for most patients.10 The standard high-dose stimulation test can be done at any time during the day.26 If the cosyntropin is injected intravenously, any value higher than 18 to 20 μg/dL indicates normal adrenal function and excludes adrenal insufficiency.27,28 If intramuscular injection is used, any value higher than 16 to 18 μg/dL at 30 minutes post-consyntropin excludes adrenal insufficiency.29

The ACTH stimulation test may not exclude acute secondary or tertiary adrenal insufficiency.

Insulin tolerance testing

Insulin tolerance testing remains the gold standard for diagnosing adrenal insufficiency and assessing the integrity of the pituitary-adrenal axis. However, given its difficulty to perform, safety concerns, and the availability of other reliable tests, its use in clinical practice is limited. It is nonetheless useful in assessing patients with recent onset of ACTH deficiency.30,31

CASE RESUMED: PATIENT DISCHARGED, LOST TO FOLLOW-UP

Abdominal CT without contrast is done and demonstrates bilateral adrenal hemorrhage. Thus, the patient is diagnosed with primary acute adrenal insufficiency due to adrenal necrosis.

She is started on oral hydrocortisone and fludrocortisone after intravenous hydrocortisone is discontinued. She is counseled about adhering to medications, wearing a medical alert bracelet, giving herself emergency cortisol injections, taking higher doses of hydrocortisone if she is ill, and monitoring her INR. She is discharged home after her symptoms resolve.

The patient does not keep her scheduled appointment and is lost to follow-up. She returns 2 years later complaining of fatigue and feeling unwell. She admits that she stopped taking hydrocortisone 1 year ago after reading an online article about corticosteroid side effects. She has continued to take fludrocortisone.

MINERALOCORTICOID VS CORTICOSTEROID DEFICIENCY

Clinical features of primary vs central adrenal insufficiency
Our patient has primary adrenal insufficiency. The presentations of primary and central (secondary or tertiary) adrenal insufficiency are similar, but there are critical differences (Table 5). Further, she has been taking her mineralocorticoid (fludrocortisone) replacement but has stopped taking her corticosteroid (hydrocortisone).

4. Which of the following is least likely to be present in this patient at this time?

  • Intravascular volume depletion
  • Hyponatremia
  • Skin hyperpigmentation
  • Normokalemia
  • Elevated serum ACTH level

Intravascular volume depletion

Intravascular volume depletion is the least likely to be present. This is because intravascular volume depletion is mainly secondary to mineralocorticoid deficiency rather than corticosteroid deficiency, which is not present in this patient, as she is compliant with her mineralocorticoid replacement therapy.32,33 However, even with sufficient mineralocorticoid replacement, mild hypotension may be present in this patient due to corticosteroid deficiency-induced loss of vascular tone.

Hyponatremia

Hyponatremia in adrenal insufficiency is not due only to mineralocorticoid deficiency. Patients with secondary or tertiary adrenal insufficiency may also exhibit hyponatremia.34 ACTH deficiency in such patients is not expected to cause mineralocorticoid deficiency, as ACTH has only a minor role in aldosterone production.

It has been proposed that hyponatremia in secondary adrenal insufficiency is due to cortisol deficiency resulting in an increase of antidiuretic hormone secretion.35,36 The mechanisms for increased antidiuretic hormone include cortisol deficiency resulting in an increased corticotropin-releasing hormone level, which acts as an antidiuretic hormone secretagogue,37,38 and cortisol directly suppressing antidiuretic hormone secretion.39

In our patient, volume expansion and hyponatremia are expected due to increased antidiuretic hormone secretion as a result of corticosteroid insufficiency.

 

 

Hyperpigmentation

Hyperpigmentation of the skin is present only in long-standing primary adrenal insufficiency. This is due to chronic cortisol deficiency causing an increased secretion of pro-opiomelanocortin, a prohormone that is cleaved into ACTH, melanocyte-stimulating hormone, and other hormones. Melanocyte-stimulating hormone causes skin hyperpigmentation due to increased melanin synthesis.40 The hyperpigmentation is seen in sun-exposed areas, pressure areas, palmar creases, nipples, and mucous membranes.

This patient has long-standing corticosteroid deficiency due to noncompliance and primary adrenal insufficiency, and as a result she is expected to have elevated serum ACTH and hyperpigmentation.

Normokalemia

Mineralocorticoid deficiency results in hyperkalemia and metabolic acidosis by impairing renal excretion of potassium and acid.41 This patient is compliant with her mineralocorticoid replacement regimen; thus, potassium levels and pH are expected to be normal.

TAKE-HOME POINTS

  • Suspect adrenal crisis in any patient who presents with shock.
  • Acute abdomen or unexplained fever could be among the manifestations.
  • Initial management requires liberal normal saline intravenous fluid administration to replete the intravascular space.
  • Draw blood samples for serum chemistry, cortisol, and ACTH, followed immediately by intravenous hydrocortisone supplementation.
  • In critically ill patients, evaluate adrenal function with random serum cortisol; in a nonacute setting use the ACTH stimulation test.
  • Chronic management of primary adrenal insufficiency requires corticosteroid and mineralocorticoid therapy.
References
  1. Mani S, Rutecki GW. A patient with altered mental status and an acid-base disturbance. Cleve Clin J Med 2017; 84(1):27–34. doi:10.3949/ccjm.84a.16042
  2. Almiani M, Gorthi J, Subbiah S, Firoz M. Quiz page November 2012: an unusual case of acute hyponatremia and normal anion gap metabolic acidosis. Am J Kidney Dis 2012; 60(5):xxxiii–xxxvi. doi:10.1053/j.ajkd.2012.05.026
  3. Migeon CJ, Kenny FM, Hung W, Voorhess ML. Study of adrenal function in children with meningitis. Pediatrics 1967; 40(2):163–183.
  4. Rao RH, Vagnucci AH, Amico JA. Bilateral massive adrenal hemorrhage: early recognition and treatment. Ann Intern Med 1989; 110(3):227–235.
  5. Shimizu S, Tahara Y, Atsumi T, et al. Waterhouse-Friderichsen syndrome caused by invasive Haemophilus influenzae type B infection in a previously healthy young man. Anaesth Intensive Care 2010; 38(1):214–215.
  6. Castaldo ET, Guillamondegui OD, Greco JA 3rd, Feurer ID, Miller RS, Morris JA Jr. Are adrenal injuries predictive of adrenal insufficiency in patients sustaining blunt trauma? Am Surg 2008; 74(3):262–266.
  7. Xarli VP, Steele AA, Davis PJ, Buescher ES, Rios CN, Garcia-Bunuel R. Adrenal hemorrhage in the adult. Medicine (Baltimore) 1978; 57(3):211–221.
  8. Takeuchi K, Tsuzuki Y, Ando T, et al. Clinical studies of strangulating small bowel obstruction. Am Surg 2004; 70(1):40–44.
  9. MacKenzie IM. The haemodynamics of human septic shock. Anaesthesia 2001; 56(2):130–144.
  10. Bornstein SR, Allolio B, Arlt W, et al. Diagnosis and treatment of primary adrenal insufficiency: an Endocrine Society clinical practice guideline. J Clin Endocrinol Metab 2016; 101(2):364–389. doi:10.1210/jc.2015-1710
  11. Hamrahian AH, Fleseriu M; AACE Adrenal Scientific Committee. Evaluation and management of adrenal insufficiency in critically ill patients: disease state review. Endocr Pract 2017; 23(6):716–725. doi:10.4158/EP161720.RA
  12. Saenger P, Levine LS, Irvine WJ, et al. Progressive adrenal failure in polyglandular autoimmune disease. J Clin Endocrinol Metab 1982; 54(4):863–867.
  13. Coco G, Dal Pra C, Presotto F, et al. Estimated risk for developing autoimmune Addison's disease in patients with adrenal cortex autoantibodies. J Clin Endocrinol Metab 2006; 91(5):1637–1645. doi:10.1210/jc.2005-0860
  14. Ullian ME. The role of corticosteroids in the regulation of vascular tone. Cardiovasc Res 1999; 41(1):55–64.
  15. Yang S, Zhang L. Glucocorticoids and vascular reactivity. Curr Vasc Pharmacol 2004; 2(1):1–12.
  16. Yanase T, Tajima T, Katabami T, et al. Diagnosis and treatment of adrenal insufficiency including adrenal crisis: a Japan Endocrine Society clinical practice guideline [Opinion]. Endocr J 2016; 63(9):765–784. doi:10.1507/endocrj.EJ16-0242
  17. Taylor RL, Grebe SK, Singh RJ. Quantitative, highly sensitive liquid chromatography-tandem mass spectrometry method for detection of synthetic corticosteroids. Clin Chem 2004; 50(10):2345–2352. doi:10.1373/clinchem.2004.033605
  18. Goldfien A, Laidlaw JC, Haydar NA, Renold AE, Thorn GW. Fluorohydrocortisone and chlorohydrocortisone, highly potent derivatives of compound F. N Engl J Med 1955; 252(11):415–421. doi:10.1056/NEJM195503172521101
  19. Jenkins D, Forsham PH, Laidlaw JC, Reddy WJ, Thorn GW. Use of ACTH in the diagnosis of adrenal cortical insufficiency. Am J Med 1955; 18(1):3–14.
  20. Hägg E, Asplund K, Lithner F. Value of basal plasma cortisol assays in the assessment of pituitary-adrenal insufficiency. Clin Endocrinol (Oxf) 1987; 26(2):221–226.
  21. Deutschbein T, Unger N, Mann K, Petersenn S. Diagnosis of secondary adrenal insufficiency: unstimulated early morning cortisol in saliva and serum in comparison with the insulin tolerance test. Horm Metab Res 2009; 41(4):834–839. doi:10.1055/s-0029-1225630
  22. Erturk E, Jaffe CA, Barkan AL. Evaluation of the integrity of the hypothalamic-pituitary-adrenal axis by insulin hypoglycemia test. J Clin Endocrinol Metab 1998; 83(7):2350–2354.
  23. Cooper MS, Stewart PM. Corticosteroid insufficiency in acutely ill patients. N Engl J Med 2003; 348(8):727–734. doi:10.1056/NEJMra020529
  24. Dellinger RP, Levy MM, Rhodes A, et al; Surviving Sepsis Campaign Guidelines Committee including the Pediatric Subgroup. Surviving Sepsis Campaign: international guidelines for management of severe sepsis and septic shock, 2012. Intensive Care Med 2013; 39(2):165–228. doi:10.1007/s00134-012-2769-8
  25. Ceccato F, Barbot M, Zilio M, et al. Performance of salivary cortisol in the diagnosis of Cushing's syndrome, adrenal incidentaloma, and adrenal insufficiency. Eur J Endocrinol 2013; 169(1):31–36. doi:10.1530/EJE-13-0159
  26. Dickstein G, Shechner C, Nicholson WE, et al. Adrenocorticotropin stimulation test: effects of basal cortisol level, time of day, and suggested new sensitive low dose test. J Clin Endocrinol Metab 1991; 72(4):773–778. doi:10.1210/jcem-72-4-773
  27. May ME, Carey RM. Rapid adrenocorticotropic hormone test in practice. Retrospective review. Am J Med 1985; 79(6):679–884.
  28. Speckart PF, Nicoloff JT, Bethune JE. Screening for adrenocortical insufficiency with cosyntropin (synthetic ACTH). Arch Intern Med 1971; 128(5):761–763.
  29. Peechakara S, Bena J, Clarke NJ, et al. Total and free cortisol levels during 1 μg, 25 μg, and 250 μg cosyntropin stimulation tests compared to insulin tolerance test: results of a randomized, prospective, pilot study. Endocrine 2017; 57(3):388–393. doi:10.1007/s12020-017-1371-9
  30. Finucane FM, Liew A, Thornton E, Rogers B, Tormey W, Agha A. Clinical insights into the safety and utility of the insulin tolerance test (ITT) in the assessment of the hypothalamo-pituitary-adrenal axis. Clin Endocrinol (Oxf) 2008; 69(4):603–607. doi:10.1111/j.1365-2265.2008.03240.x
  31. Lindholm J, Kehlet H. Re-evaluation of the clinical value of the 30 min ACTH test in assessing the hypothalamic-pituitary-adrenocortical function. Clin Endocrinol (Oxf) 1987; 26(1):53–59.
  32. Charmandari E, Nicolaides NC, Chrousos GP. Adrenal insufficiency. Lancet 2014; 383(9935):2152–2167. doi:10.1016/S0140-6736(13)61684-0
  33. Burke CW. Adrenocortical insufficiency. Clin Endocrinol Metab 1985; 14(4):947–976.
  34. Jessani N, Jehangir W, Behman D, Yousif A, Spiler IJ. Secondary adrenal insufficiency: an overlooked cause of hyponatremia. J Clin Med Res 2015; 7(4):286–288. doi:10.14740/jocmr2041w
  35. Oelkers W. Hyponatremia and inappropriate secretion of vasopressin (antidiuretic hormone) in patients with hypopituitarism. N Engl J Med 1989; 321(8):492–496. doi:10.1056/NEJM198908243210802
  36. Ishikawa S, Schrier RW. Effect of arginine vasopressin antagonist on renal water excretion in glucocorticoid and mineralocorticoid deficient rats. Kidney Int 1982; 22(6):587–593.
  37. Wolfson B, Manning RW, Davis LG, Arentzen R, Baldino F Jr. Co-localization of corticotropin releasing factor and vasopressin mRNA in neurones after adrenalectomy. Nature 1985; 315(6014):59–61.
  38. Kalogeras KT, Nieman LK, Friedman TC, et al. Inferior petrosal sinus sampling in healthy subjects reveals a unilateral corticotropin-releasing hormone-induced arginine vasopressin release associated with ipsilateral adrenocorticotropin secretion. J Clin Invest 1996; 97:2045–2050.
  39. Kovacs KJ, Foldes A, Sawchenko PE. Glucocorticoid negative feedback selectively targets vasopressin transcription in parvocellular neurosecretory neurons. J Neurosci 2000; 20:3843–3852.
  40. Sarkar SB, Sarkar S, Ghosh S, Bandyopadhyay S. Addison's disease. Contemp Clin Dent 2012; 3(4):484–486. doi:10.4103/0976-237X.107450
  41. Szylman P, Better OS, Chaimowitz C, Rosler A. Role of hyperkalemia in the metabolic acidosis of isolated hypoaldosteronism. N Engl J Med 1976; 294(7):361–365. doi:10.1056/NEJM197602122940703
References
  1. Mani S, Rutecki GW. A patient with altered mental status and an acid-base disturbance. Cleve Clin J Med 2017; 84(1):27–34. doi:10.3949/ccjm.84a.16042
  2. Almiani M, Gorthi J, Subbiah S, Firoz M. Quiz page November 2012: an unusual case of acute hyponatremia and normal anion gap metabolic acidosis. Am J Kidney Dis 2012; 60(5):xxxiii–xxxvi. doi:10.1053/j.ajkd.2012.05.026
  3. Migeon CJ, Kenny FM, Hung W, Voorhess ML. Study of adrenal function in children with meningitis. Pediatrics 1967; 40(2):163–183.
  4. Rao RH, Vagnucci AH, Amico JA. Bilateral massive adrenal hemorrhage: early recognition and treatment. Ann Intern Med 1989; 110(3):227–235.
  5. Shimizu S, Tahara Y, Atsumi T, et al. Waterhouse-Friderichsen syndrome caused by invasive Haemophilus influenzae type B infection in a previously healthy young man. Anaesth Intensive Care 2010; 38(1):214–215.
  6. Castaldo ET, Guillamondegui OD, Greco JA 3rd, Feurer ID, Miller RS, Morris JA Jr. Are adrenal injuries predictive of adrenal insufficiency in patients sustaining blunt trauma? Am Surg 2008; 74(3):262–266.
  7. Xarli VP, Steele AA, Davis PJ, Buescher ES, Rios CN, Garcia-Bunuel R. Adrenal hemorrhage in the adult. Medicine (Baltimore) 1978; 57(3):211–221.
  8. Takeuchi K, Tsuzuki Y, Ando T, et al. Clinical studies of strangulating small bowel obstruction. Am Surg 2004; 70(1):40–44.
  9. MacKenzie IM. The haemodynamics of human septic shock. Anaesthesia 2001; 56(2):130–144.
  10. Bornstein SR, Allolio B, Arlt W, et al. Diagnosis and treatment of primary adrenal insufficiency: an Endocrine Society clinical practice guideline. J Clin Endocrinol Metab 2016; 101(2):364–389. doi:10.1210/jc.2015-1710
  11. Hamrahian AH, Fleseriu M; AACE Adrenal Scientific Committee. Evaluation and management of adrenal insufficiency in critically ill patients: disease state review. Endocr Pract 2017; 23(6):716–725. doi:10.4158/EP161720.RA
  12. Saenger P, Levine LS, Irvine WJ, et al. Progressive adrenal failure in polyglandular autoimmune disease. J Clin Endocrinol Metab 1982; 54(4):863–867.
  13. Coco G, Dal Pra C, Presotto F, et al. Estimated risk for developing autoimmune Addison's disease in patients with adrenal cortex autoantibodies. J Clin Endocrinol Metab 2006; 91(5):1637–1645. doi:10.1210/jc.2005-0860
  14. Ullian ME. The role of corticosteroids in the regulation of vascular tone. Cardiovasc Res 1999; 41(1):55–64.
  15. Yang S, Zhang L. Glucocorticoids and vascular reactivity. Curr Vasc Pharmacol 2004; 2(1):1–12.
  16. Yanase T, Tajima T, Katabami T, et al. Diagnosis and treatment of adrenal insufficiency including adrenal crisis: a Japan Endocrine Society clinical practice guideline [Opinion]. Endocr J 2016; 63(9):765–784. doi:10.1507/endocrj.EJ16-0242
  17. Taylor RL, Grebe SK, Singh RJ. Quantitative, highly sensitive liquid chromatography-tandem mass spectrometry method for detection of synthetic corticosteroids. Clin Chem 2004; 50(10):2345–2352. doi:10.1373/clinchem.2004.033605
  18. Goldfien A, Laidlaw JC, Haydar NA, Renold AE, Thorn GW. Fluorohydrocortisone and chlorohydrocortisone, highly potent derivatives of compound F. N Engl J Med 1955; 252(11):415–421. doi:10.1056/NEJM195503172521101
  19. Jenkins D, Forsham PH, Laidlaw JC, Reddy WJ, Thorn GW. Use of ACTH in the diagnosis of adrenal cortical insufficiency. Am J Med 1955; 18(1):3–14.
  20. Hägg E, Asplund K, Lithner F. Value of basal plasma cortisol assays in the assessment of pituitary-adrenal insufficiency. Clin Endocrinol (Oxf) 1987; 26(2):221–226.
  21. Deutschbein T, Unger N, Mann K, Petersenn S. Diagnosis of secondary adrenal insufficiency: unstimulated early morning cortisol in saliva and serum in comparison with the insulin tolerance test. Horm Metab Res 2009; 41(4):834–839. doi:10.1055/s-0029-1225630
  22. Erturk E, Jaffe CA, Barkan AL. Evaluation of the integrity of the hypothalamic-pituitary-adrenal axis by insulin hypoglycemia test. J Clin Endocrinol Metab 1998; 83(7):2350–2354.
  23. Cooper MS, Stewart PM. Corticosteroid insufficiency in acutely ill patients. N Engl J Med 2003; 348(8):727–734. doi:10.1056/NEJMra020529
  24. Dellinger RP, Levy MM, Rhodes A, et al; Surviving Sepsis Campaign Guidelines Committee including the Pediatric Subgroup. Surviving Sepsis Campaign: international guidelines for management of severe sepsis and septic shock, 2012. Intensive Care Med 2013; 39(2):165–228. doi:10.1007/s00134-012-2769-8
  25. Ceccato F, Barbot M, Zilio M, et al. Performance of salivary cortisol in the diagnosis of Cushing's syndrome, adrenal incidentaloma, and adrenal insufficiency. Eur J Endocrinol 2013; 169(1):31–36. doi:10.1530/EJE-13-0159
  26. Dickstein G, Shechner C, Nicholson WE, et al. Adrenocorticotropin stimulation test: effects of basal cortisol level, time of day, and suggested new sensitive low dose test. J Clin Endocrinol Metab 1991; 72(4):773–778. doi:10.1210/jcem-72-4-773
  27. May ME, Carey RM. Rapid adrenocorticotropic hormone test in practice. Retrospective review. Am J Med 1985; 79(6):679–884.
  28. Speckart PF, Nicoloff JT, Bethune JE. Screening for adrenocortical insufficiency with cosyntropin (synthetic ACTH). Arch Intern Med 1971; 128(5):761–763.
  29. Peechakara S, Bena J, Clarke NJ, et al. Total and free cortisol levels during 1 μg, 25 μg, and 250 μg cosyntropin stimulation tests compared to insulin tolerance test: results of a randomized, prospective, pilot study. Endocrine 2017; 57(3):388–393. doi:10.1007/s12020-017-1371-9
  30. Finucane FM, Liew A, Thornton E, Rogers B, Tormey W, Agha A. Clinical insights into the safety and utility of the insulin tolerance test (ITT) in the assessment of the hypothalamo-pituitary-adrenal axis. Clin Endocrinol (Oxf) 2008; 69(4):603–607. doi:10.1111/j.1365-2265.2008.03240.x
  31. Lindholm J, Kehlet H. Re-evaluation of the clinical value of the 30 min ACTH test in assessing the hypothalamic-pituitary-adrenocortical function. Clin Endocrinol (Oxf) 1987; 26(1):53–59.
  32. Charmandari E, Nicolaides NC, Chrousos GP. Adrenal insufficiency. Lancet 2014; 383(9935):2152–2167. doi:10.1016/S0140-6736(13)61684-0
  33. Burke CW. Adrenocortical insufficiency. Clin Endocrinol Metab 1985; 14(4):947–976.
  34. Jessani N, Jehangir W, Behman D, Yousif A, Spiler IJ. Secondary adrenal insufficiency: an overlooked cause of hyponatremia. J Clin Med Res 2015; 7(4):286–288. doi:10.14740/jocmr2041w
  35. Oelkers W. Hyponatremia and inappropriate secretion of vasopressin (antidiuretic hormone) in patients with hypopituitarism. N Engl J Med 1989; 321(8):492–496. doi:10.1056/NEJM198908243210802
  36. Ishikawa S, Schrier RW. Effect of arginine vasopressin antagonist on renal water excretion in glucocorticoid and mineralocorticoid deficient rats. Kidney Int 1982; 22(6):587–593.
  37. Wolfson B, Manning RW, Davis LG, Arentzen R, Baldino F Jr. Co-localization of corticotropin releasing factor and vasopressin mRNA in neurones after adrenalectomy. Nature 1985; 315(6014):59–61.
  38. Kalogeras KT, Nieman LK, Friedman TC, et al. Inferior petrosal sinus sampling in healthy subjects reveals a unilateral corticotropin-releasing hormone-induced arginine vasopressin release associated with ipsilateral adrenocorticotropin secretion. J Clin Invest 1996; 97:2045–2050.
  39. Kovacs KJ, Foldes A, Sawchenko PE. Glucocorticoid negative feedback selectively targets vasopressin transcription in parvocellular neurosecretory neurons. J Neurosci 2000; 20:3843–3852.
  40. Sarkar SB, Sarkar S, Ghosh S, Bandyopadhyay S. Addison's disease. Contemp Clin Dent 2012; 3(4):484–486. doi:10.4103/0976-237X.107450
  41. Szylman P, Better OS, Chaimowitz C, Rosler A. Role of hyperkalemia in the metabolic acidosis of isolated hypoaldosteronism. N Engl J Med 1976; 294(7):361–365. doi:10.1056/NEJM197602122940703
Issue
Cleveland Clinic Journal of Medicine - 85(4)
Issue
Cleveland Clinic Journal of Medicine - 85(4)
Page Number
303-312
Page Number
303-312
Publications
Publications
Topics
Article Type
Display Headline
A 71-year-old woman with shock and a high INR
Display Headline
A 71-year-old woman with shock and a high INR
Legacy Keywords
adrenal crisis, adrenal failure, shock, hypotension, atrial fibrillation, anticoagulation, warfarin, acetaminophen, interaction, prothrombin time, INR, international normalized ratio, adrenal hemorrhage, cortisone, ACTH, adrenocoticotropic hormone, ACTH stimulation test, Raja Zaghlol, Michael Tierney, Louay Zaghlol, Ayman Zayed
Legacy Keywords
adrenal crisis, adrenal failure, shock, hypotension, atrial fibrillation, anticoagulation, warfarin, acetaminophen, interaction, prothrombin time, INR, international normalized ratio, adrenal hemorrhage, cortisone, ACTH, adrenocoticotropic hormone, ACTH stimulation test, Raja Zaghlol, Michael Tierney, Louay Zaghlol, Ayman Zayed
Sections
Disallow All Ads
Content Gating
No Gating (article Unlocked/Free)
Alternative CME
Disqus Comments
Default
Gate On Date
Thu, 03/29/2018 - 08:15
Un-Gate On Date
Thu, 03/29/2018 - 08:15
Use ProPublica
Hide sidebar & use full width
render the right sidebar.
Article PDF Media

Single-Dose Niacin-Induced Hepatitis

Article Type
Changed
Wed, 12/12/2018 - 21:09
A 22-year-old man with an unremarkable medical history presented with a 2-week history of intermittent periumbilical abdominal pain.

Niacin, also known as vitamin B3, is an important cofactor in many metabolic processes necessary to life. Over the past 15 to 20 years, niacin has been prescribed to patients with hyperlipidemia to increase high-density lipoprotein and lower low-density lipoprotein.1 As a naturally occurring vitamin, niacin is also available over-the-counter (OTC) as a dietary supplement, and is also a common ingredient in energy drinks and multivitamins.2

In addition to treating hyperlipidemia and as a nutritional supplement, some anecdotal reports amongst lay-persons suggests that niacin offers other health benefits, such as promoting weight loss and expediting the elimination of alcohol and illicit drugs from one’s system (eg, marijuana).3,4 The increased use of niacin supplementation in the general population for all of the aforementioned reasons has resulted in an increased incidence of niacin toxicity.

Formulations

Niacin is available in three formulations: extended-release (ER, also referred to as intermediate-release), immediate-release (IR), and sustained-release (SR).

The ER formulations of niacin are typically prescribed to treat hyperlipidemia. Patients are usually started on ER niacin at an initial dose of 250 mg once daily. The dose is gradually increased, as tolerated or necessary, to 2 g per day, taken in three doses. It is not uncommon for patients with hyperlipidemia to take more than 1 g of niacin per day after titration by their primary physicians.

Side Effects

Since niacin increases the release of arachidonic acid from cell membranes that metabolizes into prostaglandins, specifically prostaglandins E2 and D2, many patients taking niacin experience uncomfortable flushing and itching.5 Nonsteroidal anti-inflammatory drugs (NSAIDs) prevent this side effect by inhibiting the metabolism of arachidonic acid into those vasodilatory prostaglandins. The newer ER and SR formulations of niacin, which are approved for OTC use as a dietary supplement, are less likely to cause flushing.5

Extended-release niacin, however, is associated with a higher incidence of hepatotoxicity than the other prescription formulations of niacin.6 Toxicity has been well recognized in patients taking niacin chronically for hyperlipidemia, with reports of such cases dating back to the 1980s.7,8 We report a unique case of niacin toxicity following a single-dose ingestion in a young man.

Case

A 22-year-old man presented to the ED for evaluation of a 2-week history of intermittent periumbilical abdominal pain. This visit represented the patient’s second visit to the ED over the past week for the same complaint.

Upon presentation the patient’s vital signs were: blood pressure (BP), 113/64 mm Hg; heart rate, 82 beats/min; respiratory rate, 16 breaths/min; and temperature 36.6°C. Oxygen saturation was 100% on room air. The patient was otherwise healthy and had no significant recent or remote medical history. He denied taking any medications prior to his initial presentation, and reported only occasional alcohol use.

At the patient’s initial presentation 1 week earlier, he was diagnosed with acute gastroenteritis and treated with famotidine and ondansetron in the ED. The patient appeared well clinically at this visit, and laboratory values were within normal limits, including normal blood glucose and urinalysis.

The patient was discharged home from this first visit with prescriptions of famotidine and ondansetron, and was advised to follow up with his primary care physician in 1 week. Throughout the week after discharge from the ED, the patient experienced worsening abdominal pain, and he developed frequent nonbloody emesis, prompting his second presentation to the ED. At this second visit, the patient stated that he had taken one dose of ondansetron at home, without effect. He also noted subjective fevers, but had no diarrhea or melena.

Vital signs remained within normal limits with BPs ranging from 115 to 130 mm Hg systolic and 50 to 89 mm Hg diastolic. The patient was never tachycardic, tachypneic, febrile, or hypoxic. Physical examination was remarkable for periumbilical tenderness. The patient had no jaundice. A more thorough laboratory evaluation revealed elevated anion gap and blood urea nitrogen/creatinine values, and leukocytosis. The patient’s hepatic enzymes were also elevated, with aspartate aminotransferase (AST) over 2,000 U/L and alanine aminotransferase (ALT) of 1,698 U/L. Lipase, bilirubin, and alkaline phosphatase were all within normal limits. The patient’s prothrombin time (PT) was elevated at 14 seconds, and the international normalized ratio (INR) was elevated at 1.28. Laboratory analysis for acetaminophen and alcohol was negative.

A computed tomography (CT) scan of the abdomen/pelvis with intravenous (IV) contrast was unremarkable, demonstrating a liver devoid of any masses, portal or biliary dilation, or cirrhotic changes.

The patient received IV famotidine and ondansetron, and morphine for pain control, and was admitted to the general medical floor for hepatitis of uncertain etiology. A viral hepatitis panel was negative.

On the recommendation of the toxicology service, the patient was given N-acetylcysteine (NAC), and his hepatic enzymes trended down to an AST of 642 U/L and an ALT of 456 U/L by hospital day 2. (The patient essentially completed a positive dechallenge test).9

A gastroenterology consult was ordered, during which additional history-taking and chart review noted that the patient admitted to taking one or two tablets of OTC niacin as a dietary supplement the day before his initial presentation. Although the patient could not recall the exact dosage, he stated that he had been taking supplemental niacin approximately once a month over the past several years without any issues. Since OTC niacin is most commonly available in 500-mg tablets, this suggested the patient’s recent one-time ingested dose was approximately 500 to 1,000 mg.

Based on the patient’s admission to niacin use, additional studies were ordered, including an abdominal ultrasound and a urine drug screen. Ultrasound findings were unremarkable for portal venous thrombosis. The urine drug screen, however, was positive for marijuana and opiates. While the patient denied any history of opioid use, the positive opiate assay could have been attributed to the morphine given in the ED.

Throughout the patient’s hospital course, he remained normotensive and had no change in mental status. His liver enzymes, PT, and INR continued to normalize, and he was discharged home after 3 days, with instructions to follow up with the gastrointestinal clinic within 11 days. An appointment was made for him, which he did not attend.

Given the patient’s negative autoimmune and viral workup, and rapid resolution of symptoms after discontinuing niacin use, it is believed that he had an acute drug-induced hepatitis due to niacin ingestion. Regarding any coingestants that could have contributed to the hepatitis, the patient denied taking other common coingestants such as alcohol and acetaminophen; this assertion was supported by laboratory results.

Since we were unable to attain a qualitative measurement of the patient’s niacin concentration, our diagnosis was primarily based on the patient’s reported history.10 It is possible the patient had been taking more niacin than that to which he admitted, or that he was taking another hepatotoxic substance not detected on our toxicology workup. As previously noted, there are many medications and/or dietary supplements that could cause or contribute to a synergistic effect of drug-induced hepatitis for which the patient was not tested at his initial presentation. The patient could have co-ingested this large dose of niacin with acetaminophen and/or other supplements, energy drinks, or alcohol. A combination such as this could have contributed to his hepatitis, and the metabolites of these other substances would have been eliminated by the time of his second ED presentation.

 

 

Discussion

There are over 900 different drugs, toxins, and supplements known to cause hepatic injury.11,12 Clinical manifestations of toxicity range from asymptomatic incidental elevations in transaminases to fulminant liver failure causing mortality. Ingestion of commonly used medications such as statins (although not in overdose quantities) can cause transient asymptomatic transaminitis.13 These elevations are usually mild—ie, less than twice the upper limit of normal. Patients who experience such elevations can usually continue to take the medications with frequent and vigilant monitoring of hepatic function.

Signs and Symptoms

Acute Liver Injury. Acute liver injury is diagnosed when AST and ALT levels are greater than twice the upper limit of normal. Patients also typically have mild-to-moderate abdominal findings, such as pain, nausea, and vomiting—as was experienced by our patient. Along with niacin, angiotensin-converting enzyme inhibitors, NSAIDs, and antifungal medications are examples of other medications that can cause this degree of drug-induced hepatitis.

Severe Liver Injury. Severe liver injury features elevations in not only AST and ALT, but also alkaline phosphate and bilirubin. Patients with severe hepatic injury appear clinically ill and may exhibit altered mental status and jaundice. This type of subfulminant hepatic failure commonly results from acetaminophen toxicity, anesthetic gases, iron toxicity, phosphorus toxicity, and cocaine toxicity. Examples of drugs that result in massive liver necrosis and fulminant hepatitis are acetaminophen, isoniazid, phenelzine, phenytoin, propylthiouracil, and sertraline. Patients with massive hepatic necrosis and hepatitis may require liver transplantation.

Etiology

Identifying the etiology of liver injury is made largely through the patient’s history because there are simply too many possible hepatotoxic agents to test for them all. Diagnostic suspicion of hepatic toxicity should be increased with signs of more serious disease; however, drug-induced liver injury should be included in the differential diagnosis for all cases of abdominal pain.

With respect to the patient in our case, obtaining a more complete history involving supplement and vitamin use would have allowed us to make the diagnosis in the ED. Unfortunately, these subtle aspects of a patient’s history are often overlooked in the emergent care setting.

Treatment

The treatment of niacin-induced liver injury is similar to the guidelines for treating most other drug-induced pathology.14 Removal of the offending agent and providing supportive care is the primary treatment modality.15 In addition, it is important that the clinician exclude and rule-out other causes of hepatitis such as those of viral, autoimmune, or ischemic etiology.

N-acetylcysteine. A medication classically used in patients with acetaminophen overdose, NAC is a safe and effective treatment for non-acetaminophen-induced liver injury, and was given to treat our patient.16L-carnitine. L-carnitine has been shown to be effective in cases of chronic steatosis from hepatitis C and in valproic acid induced hepatitis.17Since L-carnitine is not included on our hospital’s formulary, it was not a treatment option for our patient.Glucocorticoid Therapy. Although glucocorticoids are occasionally given to patients with systemic symptoms of drug reactions, its effectiveness has not been adequately studied.18

Prognosis

The prognosis of patients with acute drug-induced hepatitis is generally good, and most patients fully recover once the offending agent is removed. Poor prognostic factors include the presence of jaundice, requirement for dialysis, underlying chronic liver conditions, or elevated serum creatinine. While most patients will experience a complete recovery, approximately 5% to 10% will develop chronic hepatitis and/or cirrhosis.

Conclusion

Niacin is now available as prescription and OTC formulations and is a potentially hepatotoxic medication and dietary supplement. Niacin can cause an acute hepatitis, especially when taken in conjunction with other hepatotoxic substances. Drug-induced liver injury from niacin ingestion will improve quickly following removal, and the prognosis in otherwise healthy individuals is good.

Patients, especially young, healthy patients who present with symptoms concerning for hepatitis, should be asked specifically about any nutritional, herbal, or other supplement usage. During the history intake, many patients do not consider vitamins or other nutritional or herbal supplements as “medication” or as being significant, and only report prescription and OTC medications.

References

1. Berglund L, Brunzell JD, Goldberg AC, et al. Evaluation and treatment of hypertriglyceridemia: an Endocrine Society clinical practice guideline. J Clin Endocrinol Metab. 2012;97(9):2969-2989. doi:10.1210/jc.2011-3213.

2. Vivekanandarajah A, Ni S, Waked A. Acute hepatitis in a woman following excessive ingestion of an energy drink: a case report. J Med Case Rep. 2011;5:227. doi:10.1186/1752-1947-5-227.

3. Niacin. U.S. National Library of Medicine. https://medlineplus.gov/druginfo/meds/a682518.html. Updated July 15, 2017. Accessed February 21, 2018.

4. Addiction Resource. Niacin flush for drug detox. https://addictionresource.com/drug-testing/niacin-drug-test/. Accessed February 20, 2018.

5. Kamanna VS, Ganji SH, Kashyap ML. The mechanism and mitigation of niacin-induced flushing. Int J Clin Pract. 2009;63(9):1369-1377. doi:10.1111/j.1742-1241.2009.02099.x.

6. Etchason JA, Miller TD, Squires RW, et al. Niacin-induced hepatitis: a potential side effect with low-dose time-release niacin. Mayo Clin Proc. 1991;66(1):23-28.

7. Patterson DJ, Dew EW, Gyorkey F, Graham DY. Niacin hepatitis. South Med J. 1983;76(2):239-241.

8. Ferenchick G, Rovner D. Hepatitis and hematemesis complicating nicotinic acid use. Am J Med Sci. 1989;298(3):191-193.

9. Henkin Y, Johnson KC, Segrest JP. Rechallenge with crystalline niacin after drug-induced hepatitis from sustained-release niacin. JAMA. 1990;264(2):241-243.

10. Barritt AS 4th, Lee J, Hayashi PH. Detective work in drug-induced liver injury: sometimes it is all about interviewing the right witness. Clin Gastroenterol Hepatol. 2010;8(7):635-637. doi:10.1016/j.cgh.2010.03.020.

11. Chitturi S, Teoh NC, Farrell GC. Hepatic drug metabolism and liver disease caused by drugs. In: Feldman M, Friedman LS, Brandt LJ, eds. Sleisenger and Fordtran’s Gastrointestinal and Liver Disease. 10th ed. Philadelphia, PA: Elsevier Saunders; 2016:1442-1477.

12. National Institutes of Health. LiverTox: Clinical and Research Information on Drug-Induced Liver Injury. Niacin. https://livertox.nih.gov/Niacin.htm. Updated January 18, 2018. Accessed February 21, 2018.

13. Chalasani N. Statins and hepatotoxicity: focus on patients with fatty liver. Hepatology. 2005;41(4):690-695. 

14. Chalasani NP, Hayashi PH, Bonkovsky HL, et al. ACG Clinical Guideline: the diagnosis and management of idiosyncratic drug-induced liver injury. Am J Gastroenterol. 2014;109(7):950-966. doi:10.1038/ajg.2014.131.

15. Larson AM. Hepatotoxicity due to herbal medication and dietary supplements. UpToDate Web site. https://www.uptodate.com/contents/hepatotoxicity-due-to-herbal-medications-and-dietary-supplements?source=search_result&search=niacin%20induced%20hepatitis&selectedTitle=3~150. Updated December 21, 2017. Accessed February 21, 2018.

16. Mumtaz K, Azam Z, Hamid S, et al. Role of N-acetylcysteine in adults with non-acetaminophen-induced acute liver failure in a center without the facility of liver transplantation. Hepatol Int. 2009;3(4):563-570. doi:10.1007/s12072-009-9151-0.

17. Perrott J, Murphy NG, Zed PJ. L-carnitine for acute valproic acid overdose: a systematic review of published cases. Ann Pharmacother. 2010;44(7-8):1287-1293. doi:10.1345/aph.1P135.

18. O’Grady JG, Alexander GJ, Hayllar KM, Williams R. Early indicators of prognosis in fulminant hepatic failure. Gastroenterology. 1989;97(2):439-445.

Article PDF
Author and Disclosure Information

Authors’ Disclosure Statement: The authors report no actual or potential conflict of interest in relation to this article.

Dr Reines is a resident, department of emergency medicine, Detroit Receiving Hospital, Detroit Medical Center, Wayne State University School of Medicine, Michigan. Dr Lagina is a clinical assistant professor, department of emergency medicine, Wayne State University School of Medicine, Detroit, Michigan.

Issue
Emergency Medicine - 50(2)
Publications
Topics
Page Number
60-64
Sections
Author and Disclosure Information

Authors’ Disclosure Statement: The authors report no actual or potential conflict of interest in relation to this article.

Dr Reines is a resident, department of emergency medicine, Detroit Receiving Hospital, Detroit Medical Center, Wayne State University School of Medicine, Michigan. Dr Lagina is a clinical assistant professor, department of emergency medicine, Wayne State University School of Medicine, Detroit, Michigan.

Author and Disclosure Information

Authors’ Disclosure Statement: The authors report no actual or potential conflict of interest in relation to this article.

Dr Reines is a resident, department of emergency medicine, Detroit Receiving Hospital, Detroit Medical Center, Wayne State University School of Medicine, Michigan. Dr Lagina is a clinical assistant professor, department of emergency medicine, Wayne State University School of Medicine, Detroit, Michigan.

Article PDF
Article PDF
A 22-year-old man with an unremarkable medical history presented with a 2-week history of intermittent periumbilical abdominal pain.
A 22-year-old man with an unremarkable medical history presented with a 2-week history of intermittent periumbilical abdominal pain.

Niacin, also known as vitamin B3, is an important cofactor in many metabolic processes necessary to life. Over the past 15 to 20 years, niacin has been prescribed to patients with hyperlipidemia to increase high-density lipoprotein and lower low-density lipoprotein.1 As a naturally occurring vitamin, niacin is also available over-the-counter (OTC) as a dietary supplement, and is also a common ingredient in energy drinks and multivitamins.2

In addition to treating hyperlipidemia and as a nutritional supplement, some anecdotal reports amongst lay-persons suggests that niacin offers other health benefits, such as promoting weight loss and expediting the elimination of alcohol and illicit drugs from one’s system (eg, marijuana).3,4 The increased use of niacin supplementation in the general population for all of the aforementioned reasons has resulted in an increased incidence of niacin toxicity.

Formulations

Niacin is available in three formulations: extended-release (ER, also referred to as intermediate-release), immediate-release (IR), and sustained-release (SR).

The ER formulations of niacin are typically prescribed to treat hyperlipidemia. Patients are usually started on ER niacin at an initial dose of 250 mg once daily. The dose is gradually increased, as tolerated or necessary, to 2 g per day, taken in three doses. It is not uncommon for patients with hyperlipidemia to take more than 1 g of niacin per day after titration by their primary physicians.

Side Effects

Since niacin increases the release of arachidonic acid from cell membranes that metabolizes into prostaglandins, specifically prostaglandins E2 and D2, many patients taking niacin experience uncomfortable flushing and itching.5 Nonsteroidal anti-inflammatory drugs (NSAIDs) prevent this side effect by inhibiting the metabolism of arachidonic acid into those vasodilatory prostaglandins. The newer ER and SR formulations of niacin, which are approved for OTC use as a dietary supplement, are less likely to cause flushing.5

Extended-release niacin, however, is associated with a higher incidence of hepatotoxicity than the other prescription formulations of niacin.6 Toxicity has been well recognized in patients taking niacin chronically for hyperlipidemia, with reports of such cases dating back to the 1980s.7,8 We report a unique case of niacin toxicity following a single-dose ingestion in a young man.

Case

A 22-year-old man presented to the ED for evaluation of a 2-week history of intermittent periumbilical abdominal pain. This visit represented the patient’s second visit to the ED over the past week for the same complaint.

Upon presentation the patient’s vital signs were: blood pressure (BP), 113/64 mm Hg; heart rate, 82 beats/min; respiratory rate, 16 breaths/min; and temperature 36.6°C. Oxygen saturation was 100% on room air. The patient was otherwise healthy and had no significant recent or remote medical history. He denied taking any medications prior to his initial presentation, and reported only occasional alcohol use.

At the patient’s initial presentation 1 week earlier, he was diagnosed with acute gastroenteritis and treated with famotidine and ondansetron in the ED. The patient appeared well clinically at this visit, and laboratory values were within normal limits, including normal blood glucose and urinalysis.

The patient was discharged home from this first visit with prescriptions of famotidine and ondansetron, and was advised to follow up with his primary care physician in 1 week. Throughout the week after discharge from the ED, the patient experienced worsening abdominal pain, and he developed frequent nonbloody emesis, prompting his second presentation to the ED. At this second visit, the patient stated that he had taken one dose of ondansetron at home, without effect. He also noted subjective fevers, but had no diarrhea or melena.

Vital signs remained within normal limits with BPs ranging from 115 to 130 mm Hg systolic and 50 to 89 mm Hg diastolic. The patient was never tachycardic, tachypneic, febrile, or hypoxic. Physical examination was remarkable for periumbilical tenderness. The patient had no jaundice. A more thorough laboratory evaluation revealed elevated anion gap and blood urea nitrogen/creatinine values, and leukocytosis. The patient’s hepatic enzymes were also elevated, with aspartate aminotransferase (AST) over 2,000 U/L and alanine aminotransferase (ALT) of 1,698 U/L. Lipase, bilirubin, and alkaline phosphatase were all within normal limits. The patient’s prothrombin time (PT) was elevated at 14 seconds, and the international normalized ratio (INR) was elevated at 1.28. Laboratory analysis for acetaminophen and alcohol was negative.

A computed tomography (CT) scan of the abdomen/pelvis with intravenous (IV) contrast was unremarkable, demonstrating a liver devoid of any masses, portal or biliary dilation, or cirrhotic changes.

The patient received IV famotidine and ondansetron, and morphine for pain control, and was admitted to the general medical floor for hepatitis of uncertain etiology. A viral hepatitis panel was negative.

On the recommendation of the toxicology service, the patient was given N-acetylcysteine (NAC), and his hepatic enzymes trended down to an AST of 642 U/L and an ALT of 456 U/L by hospital day 2. (The patient essentially completed a positive dechallenge test).9

A gastroenterology consult was ordered, during which additional history-taking and chart review noted that the patient admitted to taking one or two tablets of OTC niacin as a dietary supplement the day before his initial presentation. Although the patient could not recall the exact dosage, he stated that he had been taking supplemental niacin approximately once a month over the past several years without any issues. Since OTC niacin is most commonly available in 500-mg tablets, this suggested the patient’s recent one-time ingested dose was approximately 500 to 1,000 mg.

Based on the patient’s admission to niacin use, additional studies were ordered, including an abdominal ultrasound and a urine drug screen. Ultrasound findings were unremarkable for portal venous thrombosis. The urine drug screen, however, was positive for marijuana and opiates. While the patient denied any history of opioid use, the positive opiate assay could have been attributed to the morphine given in the ED.

Throughout the patient’s hospital course, he remained normotensive and had no change in mental status. His liver enzymes, PT, and INR continued to normalize, and he was discharged home after 3 days, with instructions to follow up with the gastrointestinal clinic within 11 days. An appointment was made for him, which he did not attend.

Given the patient’s negative autoimmune and viral workup, and rapid resolution of symptoms after discontinuing niacin use, it is believed that he had an acute drug-induced hepatitis due to niacin ingestion. Regarding any coingestants that could have contributed to the hepatitis, the patient denied taking other common coingestants such as alcohol and acetaminophen; this assertion was supported by laboratory results.

Since we were unable to attain a qualitative measurement of the patient’s niacin concentration, our diagnosis was primarily based on the patient’s reported history.10 It is possible the patient had been taking more niacin than that to which he admitted, or that he was taking another hepatotoxic substance not detected on our toxicology workup. As previously noted, there are many medications and/or dietary supplements that could cause or contribute to a synergistic effect of drug-induced hepatitis for which the patient was not tested at his initial presentation. The patient could have co-ingested this large dose of niacin with acetaminophen and/or other supplements, energy drinks, or alcohol. A combination such as this could have contributed to his hepatitis, and the metabolites of these other substances would have been eliminated by the time of his second ED presentation.

 

 

Discussion

There are over 900 different drugs, toxins, and supplements known to cause hepatic injury.11,12 Clinical manifestations of toxicity range from asymptomatic incidental elevations in transaminases to fulminant liver failure causing mortality. Ingestion of commonly used medications such as statins (although not in overdose quantities) can cause transient asymptomatic transaminitis.13 These elevations are usually mild—ie, less than twice the upper limit of normal. Patients who experience such elevations can usually continue to take the medications with frequent and vigilant monitoring of hepatic function.

Signs and Symptoms

Acute Liver Injury. Acute liver injury is diagnosed when AST and ALT levels are greater than twice the upper limit of normal. Patients also typically have mild-to-moderate abdominal findings, such as pain, nausea, and vomiting—as was experienced by our patient. Along with niacin, angiotensin-converting enzyme inhibitors, NSAIDs, and antifungal medications are examples of other medications that can cause this degree of drug-induced hepatitis.

Severe Liver Injury. Severe liver injury features elevations in not only AST and ALT, but also alkaline phosphate and bilirubin. Patients with severe hepatic injury appear clinically ill and may exhibit altered mental status and jaundice. This type of subfulminant hepatic failure commonly results from acetaminophen toxicity, anesthetic gases, iron toxicity, phosphorus toxicity, and cocaine toxicity. Examples of drugs that result in massive liver necrosis and fulminant hepatitis are acetaminophen, isoniazid, phenelzine, phenytoin, propylthiouracil, and sertraline. Patients with massive hepatic necrosis and hepatitis may require liver transplantation.

Etiology

Identifying the etiology of liver injury is made largely through the patient’s history because there are simply too many possible hepatotoxic agents to test for them all. Diagnostic suspicion of hepatic toxicity should be increased with signs of more serious disease; however, drug-induced liver injury should be included in the differential diagnosis for all cases of abdominal pain.

With respect to the patient in our case, obtaining a more complete history involving supplement and vitamin use would have allowed us to make the diagnosis in the ED. Unfortunately, these subtle aspects of a patient’s history are often overlooked in the emergent care setting.

Treatment

The treatment of niacin-induced liver injury is similar to the guidelines for treating most other drug-induced pathology.14 Removal of the offending agent and providing supportive care is the primary treatment modality.15 In addition, it is important that the clinician exclude and rule-out other causes of hepatitis such as those of viral, autoimmune, or ischemic etiology.

N-acetylcysteine. A medication classically used in patients with acetaminophen overdose, NAC is a safe and effective treatment for non-acetaminophen-induced liver injury, and was given to treat our patient.16L-carnitine. L-carnitine has been shown to be effective in cases of chronic steatosis from hepatitis C and in valproic acid induced hepatitis.17Since L-carnitine is not included on our hospital’s formulary, it was not a treatment option for our patient.Glucocorticoid Therapy. Although glucocorticoids are occasionally given to patients with systemic symptoms of drug reactions, its effectiveness has not been adequately studied.18

Prognosis

The prognosis of patients with acute drug-induced hepatitis is generally good, and most patients fully recover once the offending agent is removed. Poor prognostic factors include the presence of jaundice, requirement for dialysis, underlying chronic liver conditions, or elevated serum creatinine. While most patients will experience a complete recovery, approximately 5% to 10% will develop chronic hepatitis and/or cirrhosis.

Conclusion

Niacin is now available as prescription and OTC formulations and is a potentially hepatotoxic medication and dietary supplement. Niacin can cause an acute hepatitis, especially when taken in conjunction with other hepatotoxic substances. Drug-induced liver injury from niacin ingestion will improve quickly following removal, and the prognosis in otherwise healthy individuals is good.

Patients, especially young, healthy patients who present with symptoms concerning for hepatitis, should be asked specifically about any nutritional, herbal, or other supplement usage. During the history intake, many patients do not consider vitamins or other nutritional or herbal supplements as “medication” or as being significant, and only report prescription and OTC medications.

Niacin, also known as vitamin B3, is an important cofactor in many metabolic processes necessary to life. Over the past 15 to 20 years, niacin has been prescribed to patients with hyperlipidemia to increase high-density lipoprotein and lower low-density lipoprotein.1 As a naturally occurring vitamin, niacin is also available over-the-counter (OTC) as a dietary supplement, and is also a common ingredient in energy drinks and multivitamins.2

In addition to treating hyperlipidemia and as a nutritional supplement, some anecdotal reports amongst lay-persons suggests that niacin offers other health benefits, such as promoting weight loss and expediting the elimination of alcohol and illicit drugs from one’s system (eg, marijuana).3,4 The increased use of niacin supplementation in the general population for all of the aforementioned reasons has resulted in an increased incidence of niacin toxicity.

Formulations

Niacin is available in three formulations: extended-release (ER, also referred to as intermediate-release), immediate-release (IR), and sustained-release (SR).

The ER formulations of niacin are typically prescribed to treat hyperlipidemia. Patients are usually started on ER niacin at an initial dose of 250 mg once daily. The dose is gradually increased, as tolerated or necessary, to 2 g per day, taken in three doses. It is not uncommon for patients with hyperlipidemia to take more than 1 g of niacin per day after titration by their primary physicians.

Side Effects

Since niacin increases the release of arachidonic acid from cell membranes that metabolizes into prostaglandins, specifically prostaglandins E2 and D2, many patients taking niacin experience uncomfortable flushing and itching.5 Nonsteroidal anti-inflammatory drugs (NSAIDs) prevent this side effect by inhibiting the metabolism of arachidonic acid into those vasodilatory prostaglandins. The newer ER and SR formulations of niacin, which are approved for OTC use as a dietary supplement, are less likely to cause flushing.5

Extended-release niacin, however, is associated with a higher incidence of hepatotoxicity than the other prescription formulations of niacin.6 Toxicity has been well recognized in patients taking niacin chronically for hyperlipidemia, with reports of such cases dating back to the 1980s.7,8 We report a unique case of niacin toxicity following a single-dose ingestion in a young man.

Case

A 22-year-old man presented to the ED for evaluation of a 2-week history of intermittent periumbilical abdominal pain. This visit represented the patient’s second visit to the ED over the past week for the same complaint.

Upon presentation the patient’s vital signs were: blood pressure (BP), 113/64 mm Hg; heart rate, 82 beats/min; respiratory rate, 16 breaths/min; and temperature 36.6°C. Oxygen saturation was 100% on room air. The patient was otherwise healthy and had no significant recent or remote medical history. He denied taking any medications prior to his initial presentation, and reported only occasional alcohol use.

At the patient’s initial presentation 1 week earlier, he was diagnosed with acute gastroenteritis and treated with famotidine and ondansetron in the ED. The patient appeared well clinically at this visit, and laboratory values were within normal limits, including normal blood glucose and urinalysis.

The patient was discharged home from this first visit with prescriptions of famotidine and ondansetron, and was advised to follow up with his primary care physician in 1 week. Throughout the week after discharge from the ED, the patient experienced worsening abdominal pain, and he developed frequent nonbloody emesis, prompting his second presentation to the ED. At this second visit, the patient stated that he had taken one dose of ondansetron at home, without effect. He also noted subjective fevers, but had no diarrhea or melena.

Vital signs remained within normal limits with BPs ranging from 115 to 130 mm Hg systolic and 50 to 89 mm Hg diastolic. The patient was never tachycardic, tachypneic, febrile, or hypoxic. Physical examination was remarkable for periumbilical tenderness. The patient had no jaundice. A more thorough laboratory evaluation revealed elevated anion gap and blood urea nitrogen/creatinine values, and leukocytosis. The patient’s hepatic enzymes were also elevated, with aspartate aminotransferase (AST) over 2,000 U/L and alanine aminotransferase (ALT) of 1,698 U/L. Lipase, bilirubin, and alkaline phosphatase were all within normal limits. The patient’s prothrombin time (PT) was elevated at 14 seconds, and the international normalized ratio (INR) was elevated at 1.28. Laboratory analysis for acetaminophen and alcohol was negative.

A computed tomography (CT) scan of the abdomen/pelvis with intravenous (IV) contrast was unremarkable, demonstrating a liver devoid of any masses, portal or biliary dilation, or cirrhotic changes.

The patient received IV famotidine and ondansetron, and morphine for pain control, and was admitted to the general medical floor for hepatitis of uncertain etiology. A viral hepatitis panel was negative.

On the recommendation of the toxicology service, the patient was given N-acetylcysteine (NAC), and his hepatic enzymes trended down to an AST of 642 U/L and an ALT of 456 U/L by hospital day 2. (The patient essentially completed a positive dechallenge test).9

A gastroenterology consult was ordered, during which additional history-taking and chart review noted that the patient admitted to taking one or two tablets of OTC niacin as a dietary supplement the day before his initial presentation. Although the patient could not recall the exact dosage, he stated that he had been taking supplemental niacin approximately once a month over the past several years without any issues. Since OTC niacin is most commonly available in 500-mg tablets, this suggested the patient’s recent one-time ingested dose was approximately 500 to 1,000 mg.

Based on the patient’s admission to niacin use, additional studies were ordered, including an abdominal ultrasound and a urine drug screen. Ultrasound findings were unremarkable for portal venous thrombosis. The urine drug screen, however, was positive for marijuana and opiates. While the patient denied any history of opioid use, the positive opiate assay could have been attributed to the morphine given in the ED.

Throughout the patient’s hospital course, he remained normotensive and had no change in mental status. His liver enzymes, PT, and INR continued to normalize, and he was discharged home after 3 days, with instructions to follow up with the gastrointestinal clinic within 11 days. An appointment was made for him, which he did not attend.

Given the patient’s negative autoimmune and viral workup, and rapid resolution of symptoms after discontinuing niacin use, it is believed that he had an acute drug-induced hepatitis due to niacin ingestion. Regarding any coingestants that could have contributed to the hepatitis, the patient denied taking other common coingestants such as alcohol and acetaminophen; this assertion was supported by laboratory results.

Since we were unable to attain a qualitative measurement of the patient’s niacin concentration, our diagnosis was primarily based on the patient’s reported history.10 It is possible the patient had been taking more niacin than that to which he admitted, or that he was taking another hepatotoxic substance not detected on our toxicology workup. As previously noted, there are many medications and/or dietary supplements that could cause or contribute to a synergistic effect of drug-induced hepatitis for which the patient was not tested at his initial presentation. The patient could have co-ingested this large dose of niacin with acetaminophen and/or other supplements, energy drinks, or alcohol. A combination such as this could have contributed to his hepatitis, and the metabolites of these other substances would have been eliminated by the time of his second ED presentation.

 

 

Discussion

There are over 900 different drugs, toxins, and supplements known to cause hepatic injury.11,12 Clinical manifestations of toxicity range from asymptomatic incidental elevations in transaminases to fulminant liver failure causing mortality. Ingestion of commonly used medications such as statins (although not in overdose quantities) can cause transient asymptomatic transaminitis.13 These elevations are usually mild—ie, less than twice the upper limit of normal. Patients who experience such elevations can usually continue to take the medications with frequent and vigilant monitoring of hepatic function.

Signs and Symptoms

Acute Liver Injury. Acute liver injury is diagnosed when AST and ALT levels are greater than twice the upper limit of normal. Patients also typically have mild-to-moderate abdominal findings, such as pain, nausea, and vomiting—as was experienced by our patient. Along with niacin, angiotensin-converting enzyme inhibitors, NSAIDs, and antifungal medications are examples of other medications that can cause this degree of drug-induced hepatitis.

Severe Liver Injury. Severe liver injury features elevations in not only AST and ALT, but also alkaline phosphate and bilirubin. Patients with severe hepatic injury appear clinically ill and may exhibit altered mental status and jaundice. This type of subfulminant hepatic failure commonly results from acetaminophen toxicity, anesthetic gases, iron toxicity, phosphorus toxicity, and cocaine toxicity. Examples of drugs that result in massive liver necrosis and fulminant hepatitis are acetaminophen, isoniazid, phenelzine, phenytoin, propylthiouracil, and sertraline. Patients with massive hepatic necrosis and hepatitis may require liver transplantation.

Etiology

Identifying the etiology of liver injury is made largely through the patient’s history because there are simply too many possible hepatotoxic agents to test for them all. Diagnostic suspicion of hepatic toxicity should be increased with signs of more serious disease; however, drug-induced liver injury should be included in the differential diagnosis for all cases of abdominal pain.

With respect to the patient in our case, obtaining a more complete history involving supplement and vitamin use would have allowed us to make the diagnosis in the ED. Unfortunately, these subtle aspects of a patient’s history are often overlooked in the emergent care setting.

Treatment

The treatment of niacin-induced liver injury is similar to the guidelines for treating most other drug-induced pathology.14 Removal of the offending agent and providing supportive care is the primary treatment modality.15 In addition, it is important that the clinician exclude and rule-out other causes of hepatitis such as those of viral, autoimmune, or ischemic etiology.

N-acetylcysteine. A medication classically used in patients with acetaminophen overdose, NAC is a safe and effective treatment for non-acetaminophen-induced liver injury, and was given to treat our patient.16L-carnitine. L-carnitine has been shown to be effective in cases of chronic steatosis from hepatitis C and in valproic acid induced hepatitis.17Since L-carnitine is not included on our hospital’s formulary, it was not a treatment option for our patient.Glucocorticoid Therapy. Although glucocorticoids are occasionally given to patients with systemic symptoms of drug reactions, its effectiveness has not been adequately studied.18

Prognosis

The prognosis of patients with acute drug-induced hepatitis is generally good, and most patients fully recover once the offending agent is removed. Poor prognostic factors include the presence of jaundice, requirement for dialysis, underlying chronic liver conditions, or elevated serum creatinine. While most patients will experience a complete recovery, approximately 5% to 10% will develop chronic hepatitis and/or cirrhosis.

Conclusion

Niacin is now available as prescription and OTC formulations and is a potentially hepatotoxic medication and dietary supplement. Niacin can cause an acute hepatitis, especially when taken in conjunction with other hepatotoxic substances. Drug-induced liver injury from niacin ingestion will improve quickly following removal, and the prognosis in otherwise healthy individuals is good.

Patients, especially young, healthy patients who present with symptoms concerning for hepatitis, should be asked specifically about any nutritional, herbal, or other supplement usage. During the history intake, many patients do not consider vitamins or other nutritional or herbal supplements as “medication” or as being significant, and only report prescription and OTC medications.

References

1. Berglund L, Brunzell JD, Goldberg AC, et al. Evaluation and treatment of hypertriglyceridemia: an Endocrine Society clinical practice guideline. J Clin Endocrinol Metab. 2012;97(9):2969-2989. doi:10.1210/jc.2011-3213.

2. Vivekanandarajah A, Ni S, Waked A. Acute hepatitis in a woman following excessive ingestion of an energy drink: a case report. J Med Case Rep. 2011;5:227. doi:10.1186/1752-1947-5-227.

3. Niacin. U.S. National Library of Medicine. https://medlineplus.gov/druginfo/meds/a682518.html. Updated July 15, 2017. Accessed February 21, 2018.

4. Addiction Resource. Niacin flush for drug detox. https://addictionresource.com/drug-testing/niacin-drug-test/. Accessed February 20, 2018.

5. Kamanna VS, Ganji SH, Kashyap ML. The mechanism and mitigation of niacin-induced flushing. Int J Clin Pract. 2009;63(9):1369-1377. doi:10.1111/j.1742-1241.2009.02099.x.

6. Etchason JA, Miller TD, Squires RW, et al. Niacin-induced hepatitis: a potential side effect with low-dose time-release niacin. Mayo Clin Proc. 1991;66(1):23-28.

7. Patterson DJ, Dew EW, Gyorkey F, Graham DY. Niacin hepatitis. South Med J. 1983;76(2):239-241.

8. Ferenchick G, Rovner D. Hepatitis and hematemesis complicating nicotinic acid use. Am J Med Sci. 1989;298(3):191-193.

9. Henkin Y, Johnson KC, Segrest JP. Rechallenge with crystalline niacin after drug-induced hepatitis from sustained-release niacin. JAMA. 1990;264(2):241-243.

10. Barritt AS 4th, Lee J, Hayashi PH. Detective work in drug-induced liver injury: sometimes it is all about interviewing the right witness. Clin Gastroenterol Hepatol. 2010;8(7):635-637. doi:10.1016/j.cgh.2010.03.020.

11. Chitturi S, Teoh NC, Farrell GC. Hepatic drug metabolism and liver disease caused by drugs. In: Feldman M, Friedman LS, Brandt LJ, eds. Sleisenger and Fordtran’s Gastrointestinal and Liver Disease. 10th ed. Philadelphia, PA: Elsevier Saunders; 2016:1442-1477.

12. National Institutes of Health. LiverTox: Clinical and Research Information on Drug-Induced Liver Injury. Niacin. https://livertox.nih.gov/Niacin.htm. Updated January 18, 2018. Accessed February 21, 2018.

13. Chalasani N. Statins and hepatotoxicity: focus on patients with fatty liver. Hepatology. 2005;41(4):690-695. 

14. Chalasani NP, Hayashi PH, Bonkovsky HL, et al. ACG Clinical Guideline: the diagnosis and management of idiosyncratic drug-induced liver injury. Am J Gastroenterol. 2014;109(7):950-966. doi:10.1038/ajg.2014.131.

15. Larson AM. Hepatotoxicity due to herbal medication and dietary supplements. UpToDate Web site. https://www.uptodate.com/contents/hepatotoxicity-due-to-herbal-medications-and-dietary-supplements?source=search_result&search=niacin%20induced%20hepatitis&selectedTitle=3~150. Updated December 21, 2017. Accessed February 21, 2018.

16. Mumtaz K, Azam Z, Hamid S, et al. Role of N-acetylcysteine in adults with non-acetaminophen-induced acute liver failure in a center without the facility of liver transplantation. Hepatol Int. 2009;3(4):563-570. doi:10.1007/s12072-009-9151-0.

17. Perrott J, Murphy NG, Zed PJ. L-carnitine for acute valproic acid overdose: a systematic review of published cases. Ann Pharmacother. 2010;44(7-8):1287-1293. doi:10.1345/aph.1P135.

18. O’Grady JG, Alexander GJ, Hayllar KM, Williams R. Early indicators of prognosis in fulminant hepatic failure. Gastroenterology. 1989;97(2):439-445.

References

1. Berglund L, Brunzell JD, Goldberg AC, et al. Evaluation and treatment of hypertriglyceridemia: an Endocrine Society clinical practice guideline. J Clin Endocrinol Metab. 2012;97(9):2969-2989. doi:10.1210/jc.2011-3213.

2. Vivekanandarajah A, Ni S, Waked A. Acute hepatitis in a woman following excessive ingestion of an energy drink: a case report. J Med Case Rep. 2011;5:227. doi:10.1186/1752-1947-5-227.

3. Niacin. U.S. National Library of Medicine. https://medlineplus.gov/druginfo/meds/a682518.html. Updated July 15, 2017. Accessed February 21, 2018.

4. Addiction Resource. Niacin flush for drug detox. https://addictionresource.com/drug-testing/niacin-drug-test/. Accessed February 20, 2018.

5. Kamanna VS, Ganji SH, Kashyap ML. The mechanism and mitigation of niacin-induced flushing. Int J Clin Pract. 2009;63(9):1369-1377. doi:10.1111/j.1742-1241.2009.02099.x.

6. Etchason JA, Miller TD, Squires RW, et al. Niacin-induced hepatitis: a potential side effect with low-dose time-release niacin. Mayo Clin Proc. 1991;66(1):23-28.

7. Patterson DJ, Dew EW, Gyorkey F, Graham DY. Niacin hepatitis. South Med J. 1983;76(2):239-241.

8. Ferenchick G, Rovner D. Hepatitis and hematemesis complicating nicotinic acid use. Am J Med Sci. 1989;298(3):191-193.

9. Henkin Y, Johnson KC, Segrest JP. Rechallenge with crystalline niacin after drug-induced hepatitis from sustained-release niacin. JAMA. 1990;264(2):241-243.

10. Barritt AS 4th, Lee J, Hayashi PH. Detective work in drug-induced liver injury: sometimes it is all about interviewing the right witness. Clin Gastroenterol Hepatol. 2010;8(7):635-637. doi:10.1016/j.cgh.2010.03.020.

11. Chitturi S, Teoh NC, Farrell GC. Hepatic drug metabolism and liver disease caused by drugs. In: Feldman M, Friedman LS, Brandt LJ, eds. Sleisenger and Fordtran’s Gastrointestinal and Liver Disease. 10th ed. Philadelphia, PA: Elsevier Saunders; 2016:1442-1477.

12. National Institutes of Health. LiverTox: Clinical and Research Information on Drug-Induced Liver Injury. Niacin. https://livertox.nih.gov/Niacin.htm. Updated January 18, 2018. Accessed February 21, 2018.

13. Chalasani N. Statins and hepatotoxicity: focus on patients with fatty liver. Hepatology. 2005;41(4):690-695. 

14. Chalasani NP, Hayashi PH, Bonkovsky HL, et al. ACG Clinical Guideline: the diagnosis and management of idiosyncratic drug-induced liver injury. Am J Gastroenterol. 2014;109(7):950-966. doi:10.1038/ajg.2014.131.

15. Larson AM. Hepatotoxicity due to herbal medication and dietary supplements. UpToDate Web site. https://www.uptodate.com/contents/hepatotoxicity-due-to-herbal-medications-and-dietary-supplements?source=search_result&search=niacin%20induced%20hepatitis&selectedTitle=3~150. Updated December 21, 2017. Accessed February 21, 2018.

16. Mumtaz K, Azam Z, Hamid S, et al. Role of N-acetylcysteine in adults with non-acetaminophen-induced acute liver failure in a center without the facility of liver transplantation. Hepatol Int. 2009;3(4):563-570. doi:10.1007/s12072-009-9151-0.

17. Perrott J, Murphy NG, Zed PJ. L-carnitine for acute valproic acid overdose: a systematic review of published cases. Ann Pharmacother. 2010;44(7-8):1287-1293. doi:10.1345/aph.1P135.

18. O’Grady JG, Alexander GJ, Hayllar KM, Williams R. Early indicators of prognosis in fulminant hepatic failure. Gastroenterology. 1989;97(2):439-445.

Issue
Emergency Medicine - 50(2)
Issue
Emergency Medicine - 50(2)
Page Number
60-64
Page Number
60-64
Publications
Publications
Topics
Article Type
Sections
Disallow All Ads
Content Gating
No Gating (article Unlocked/Free)
Alternative CME
Disqus Comments
Default
Use ProPublica
Article PDF Media

His Old Pain Is Back

Article Type
Changed
Mon, 07/09/2018 - 10:49
Display Headline
His Old Pain Is Back

ANSWER

The radiograph demonstrates a fracture within the femoral neck. These types of fractures, which are slightly angulated and located at the base of the neck, are typically referred to as basicervical femur fractures

Prompt orthopedic surgery consultation was obtained, and the patient underwent an open reduction and internal fixation of his fracture.

Article PDF
Author and Disclosure Information

Nandan R. Hichkad, PA-C, MMSc, practices at the Georgia Neurosurgical Institute in Macon and is a clinical instructor at the Mercer University School of Medicine, Macon.

Issue
Clinician Reviews - 28(3)
Publications
Topics
Page Number
17,43
Sections
Author and Disclosure Information

Nandan R. Hichkad, PA-C, MMSc, practices at the Georgia Neurosurgical Institute in Macon and is a clinical instructor at the Mercer University School of Medicine, Macon.

Author and Disclosure Information

Nandan R. Hichkad, PA-C, MMSc, practices at the Georgia Neurosurgical Institute in Macon and is a clinical instructor at the Mercer University School of Medicine, Macon.

Article PDF
Article PDF

ANSWER

The radiograph demonstrates a fracture within the femoral neck. These types of fractures, which are slightly angulated and located at the base of the neck, are typically referred to as basicervical femur fractures

Prompt orthopedic surgery consultation was obtained, and the patient underwent an open reduction and internal fixation of his fracture.

ANSWER

The radiograph demonstrates a fracture within the femoral neck. These types of fractures, which are slightly angulated and located at the base of the neck, are typically referred to as basicervical femur fractures

Prompt orthopedic surgery consultation was obtained, and the patient underwent an open reduction and internal fixation of his fracture.

Issue
Clinician Reviews - 28(3)
Issue
Clinician Reviews - 28(3)
Page Number
17,43
Page Number
17,43
Publications
Publications
Topics
Article Type
Display Headline
His Old Pain Is Back
Display Headline
His Old Pain Is Back
Sections
Questionnaire Body

You are called in for a neurosurgical consult of a 60-year-old man who underwent a lumbar decompression approximately 18 months ago. For the past several weeks, he has been experiencing worsening back pain that radiates down his left leg; at times, he has to use his cane and walker. The patient denies any acute injury or trauma preceding the pain; however, last night he slipped and fell in his bedroom, which made it much worse.

Medical history is significant for hypertension and hyperlipidemia. The patient responded well to his previous back surgery. The emergency department clinician is concerned that his symptoms may be caused by a new lumbar stenosis at an adjacent level.

On exam, you note a mildly obese male who appears uncomfortable but is in no obvious distress. His vital signs are stable. He has some tenderness within his left hip. Distally, his strength is good, and there is no evidence of neurovascular compromise. Internal and external rotation causes some discomfort.

A portable pelvic radiograph is obtained (shown). What is your impression?

Disallow All Ads
Content Gating
No Gating (article Unlocked/Free)
Alternative CME
Disqus Comments
Default
Article PDF Media

Woman, 57, With Painful, Swollen Ankle

Article Type
Changed
Wed, 08/01/2018 - 12:00
Display Headline
Woman, 57, With Painful, Swollen Ankle

IN THIS ARTICLE

  • Diagnosis
  • Treatment
  • Care outcome

A 57-year-old horticulturist is working on a ladder leaned up against a tree trunk when the ladder slips, causing her to fall six feet onto concrete. Her right foot and ankle sustain the force of the fall; she is in excruciating pain and unable to bear weight on the foot. She is immediately transported to a local emergency department for evaluation.

Physical exam reveals a tearful middle-aged female in moderate distress and acute pain. There is moderate swelling of the right medial and lateral malleolus, as well as the midfoot, with blue and purple discoloration on the medial and lateral malleolus. Radiographs of the right ankle identify nondisplaced fractures of the distal fibula and tibia. Foot x-rays are unremarkable. A splint is ordered. The patient is given crutches (non-weight-bearing status), pain medication, and a referral to orthopedics.

On day 3, the patient presents to orthopedics, where the splint is removed. An irregular, 4 × 3–in (at largest diameter), serohemorrhagic blister is discovered on the medial aspect of the lower leg, above the right malleolus (see Figure 1). Multiple 1- to 3-mm vesicles surround much of the anterior border. Moderate edema is noted from the top of the lesion to the midfoot, concentrated around the lateral and medial malleolus. Extensive blue, purple, and black discoloration is seen below the malleolus. The patient is diagnosed with a fracture blister.

DISCUSSION

Fracture blisters are taut, bullous, subepidermal vesicles that can accompany fractures or severe twisting injuries. They overlie markedly edematous soft tissue and histologically resemble a second-degree burn.1,2

Physiologically, blisters are caused by increased interstitial pressure due to swelling, with subsequent increased filtration pressure and colloid osmotic pressure in the epidermal gap.3 This causes a disruption that allows fluid to move into the weakened area.3 Areas most at risk for fracture blister formation are those with tight, closely adhered skin without muscle or enveloping fascia, where there is less soft tissue between the skin and bone prominences (eg, ankle, elbow, foot, distal tibia).2-4

Approximately 3% of all patients with acute fractures requiring hospitalization develop a fracture blister.4 Any condition that predisposes a patient to poor wound healing (eg, peripheral vascular disease, diabetes, hypertension) increases risk for a fracture blister.2 Recognizing which patients are at greatest risk is vital, as implementing prevention strategies and intervening when fracture blisters do form can help decrease complications—including infection and delayed surgery—and improve fracture resolution. In this patient’s case, the extent of the injury and force of the fall caused the fracture blister to form.

Diagnosis

Diagnosis of a fracture blister is based on clinical presentation. There are two types: hemorrhagic blisters and clear fluid-filled blisters. Hemorrhagic blisters indicate more severe injury and longer healing time (approximately 16 d), while clear fluid-filled blisters demonstrate minimal injury and therefore are quicker to heal.2,4

The differential diagnosis for fracture blisters includes friction blisters and disorders such as epidermolysis bullosa and bullous pemphigoid. Friction blisters form when the epidermis is subjected to repeated friction or shear forces (eg, from a cast or splint).5,6 These forces mechanically separate epidermal cells at the stratum spinosum layer.7 The pressure that moves across the skin forces fluid into the deeper open spaces, filling them but leaving the surface layer intact.1

Epidermolysis bullosa (EB) is a group of rare inherited cutaneous and mucus membrane disorders. EB involves fragility and detachment of subepithelial tissues, which results in blistering and erosions.8,9 The blisters tend to develop in areas subject to minor trauma, such as the extensor aspects of the elbows and the dorsal aspects of the hands and feet.9 They can also be triggered by exposure to heat, friction, scratching, and adhesive tape.10

Bullous pemphigoid, a chronic autoimmune skin disorder, is characterized by pruritic, bullous lesions. When IgG autoantibodies bind to certain hemidesmosomal antigens, complement activation causes a subepidermal blister.11While bullous pemphigoid most commonly affects those older than 60, it can also occur in children. Diagnosis is confirmed by skin biopsy and immunofluorescence testing.11

Treatment and management

Although several recommendations have been published, there is no gold standard and treatment of fracture blisters remains controversial. Early surgical intervention for fractures could decrease the incidence of fracture blisters.1,3

The goal of treatment is to achieve re-epithelialization of the dermis.3,12,13 Once a blister forms, management techniques vary. Some recommend keeping closed blisters covered with a dry dressing to protect them from damage.3 Strauss et al recommend unroofing to avoid traumatic rupture; however, this does increase risk for infection.12 Recommendations differ depending on provider preference and each patient’s individual situation. 

Elective unroofing of a blister is typically followed with one of several treatment options. These include covering the open blister with a topical antibiotic cream (eg, silver sulfadiazine 2%); applying a nonadherent, occlusive bismuth-tribromophenate-petroleum gauze dressing; or elevating and immobilizing the affected extremity.12,13

 

 

Treatment of spontaneously ruptured fracture blisters entails

  • Unroofing the blister completely and applying a topical antimicrobial (eg, silver sulfadiazine, polymyxin B, neomycin, bacitracin).
  • Applying a hydrocolloid dressing to keep the environment moist.
  • Using a first-aid gel containing melaleuca (tea tree) oil.
  • Initiating prophylactic oral antibiotics.
  • Using whirlpool treatments.
  • Elevating and immobilizing the affected extremity.3,12,14

OUTCOME FOR THE CASE PATIENT

The fracture blister was electively unroofed (see Figure 2) based on provider preference. The patient was instructed to clean the wound daily and apply topical cream (silver sulfadiazine 2% bid) to the wound and cover it with gauze. The patient was made non-weight-bearing to the right lower extremity. Continuous elevation was highly encouraged except for bathing and restroom use, and an NSAID was recommended as needed for pain. She was reassessed the following day and, due to partial refilling, the blister required additional unroofing. The patient was instructed to resume previous wound care orders.

No surgical intervention was required. CT of the right foot and ankle without contrast (performed on day 4 postinjury) confirmed a nondisplaced transverse fracture of the medial malleolus and a sagittal avulsion fracture of the anterior-inferior lateral malleolus. Multiple smaller fracture fragments were noted posterior and medial to the medial malleolus as well as inferiorly along the course of the deltoid ligament. There was a small, nondisplaced avulsion fracture of the medial malleolus at the anterolateral and posterolateral tibial plafond.

Due to the extent of the swelling, multiple fractures, and blister formation, the patient was essentially bed bound for the first three weeks; complete resolution of the fracture blister occurred 21 days after initial discovery (see Figure 3). The patient did not experience cutaneous complications. Her lower extremity was then casted in a short-leg removable cast for 10 weeks. She underwent physical therapy, and after 12 weeks, the patient was weight-bearing and was discharged from orthopedics. The patient reported refractory pain and swelling for an additional eight weeks following injury, warranting daily ibuprofen.

CONCLUSION

Fracture blisters are rare, and experience and knowledge about them in primary care is lacking. But clinicians need to be able to identify, diagnose, and refer at-risk patients to orthopedics in a timely manner.

Current management and treatment recommendations are inconsistent. Treatment varies depending on the site, severity, type, and status of the blister and the overall health of the patient. Fracture blisters may be left intact, electively unroofed, or treated after spontaneous rupture. More research is needed to clarify management recommendations, specifically regarding the decision to unroof a blister or leave it intact. Early surgical intervention may prevent the development of a fracture blister.

References

1. Wallace GF, Sullivan J. Fracture blisters.  Clin Podiatr Med Surg. 1995;12(4):801-811.
2. Halawi MJ. Fracture blisters after primary total knee arthroplasty. Am J Orthop. 2015; 44(8):E291-E293.
3. McCann S, Gruen G. Fracture blisters: a review of the literature. Orthop Nurs. 1997; 16(2):17-24.
4. Uebbing CM, Walsh M, Miller JB, et al.  Fracture blister. West J Emerg Med. 2011; 12(1):131-133.
5. Kirkham S, Lam S, Nester C, Hashmi F. The effect of hydration on the risk of friction blister formation on the heel of the foot. Skin Res Tech. 2014;20:246-253.
6. Boyd A, Benjamin H, Asplund C. Principles of casting and splinting. Am Fam Physician. 2009;79(1):16-24.
7. Knapik J, Reynolds K, Duplantis K, Jones B. Friction blisters. Pathophysiology, prevention and treatment. Sports Med. 1995; 20(3):136-147.
8. Iranzo P, Herrero-González JE, Mascaró-Galy JM, et al. Epidermolysis bullosa acquisita: a retrospective analysis of 12 patients evaluated in four tertiary hospitals in Spain. Br J Dermatol. 2014;171(5):1022-1030.
9. Peraza DM. Epidermolysis bullosa acquisita. Merck Manual Professional Version. August 2016. www.merckmanuals.com/professional/dermatologic-disorders/bullous-diseases/epidermolysis-bullosa-acquisita. Accessed January 26, 2018.
10. Lyons F, Ousley L. Dermatology for the Advanced Practice Nurse. New York, NY: Springer; 2015.
11. Peraza D. Bullous pemphigoid. Merck Manual Professional Version. August 2016. www.merckmanuals.com/professional/dermatologic-disorders/bullous-diseases/bullous-pemphigoid. Accessed January 26, 2018.
12. Strauss EJ, Petrucelli G, Bong M, et al. Blisters associated with lower-extremity fracture: Results of a prospective treatment protocol. J Orthop Trauma. 2006;20(9): 618-622.
13. Tolpinrud WL, Rebolledo BJ, Lorich DG, Grossman ME. A case of extensive fracture bullae: a multidisciplinary approach for acute management. JAAD Case Rep. 2015;1(3):132-135.
14. Cox H, Nealon L. Case report: the use of Burnaid Gel on fracture blisters. Wound Practice and Research. 2008;16(1):32-36.

Article PDF
Author and Disclosure Information

Candice N. Short, Retha Gentry, and Lisa Ousley are Assistant Professors at East Tennessee State University in Johnson City. 

Issue
Clinician Reviews - 28(3)
Publications
Topics
Page Number
29-30,32-33
Sections
Author and Disclosure Information

Candice N. Short, Retha Gentry, and Lisa Ousley are Assistant Professors at East Tennessee State University in Johnson City. 

Author and Disclosure Information

Candice N. Short, Retha Gentry, and Lisa Ousley are Assistant Professors at East Tennessee State University in Johnson City. 

Article PDF
Article PDF

IN THIS ARTICLE

  • Diagnosis
  • Treatment
  • Care outcome

A 57-year-old horticulturist is working on a ladder leaned up against a tree trunk when the ladder slips, causing her to fall six feet onto concrete. Her right foot and ankle sustain the force of the fall; she is in excruciating pain and unable to bear weight on the foot. She is immediately transported to a local emergency department for evaluation.

Physical exam reveals a tearful middle-aged female in moderate distress and acute pain. There is moderate swelling of the right medial and lateral malleolus, as well as the midfoot, with blue and purple discoloration on the medial and lateral malleolus. Radiographs of the right ankle identify nondisplaced fractures of the distal fibula and tibia. Foot x-rays are unremarkable. A splint is ordered. The patient is given crutches (non-weight-bearing status), pain medication, and a referral to orthopedics.

On day 3, the patient presents to orthopedics, where the splint is removed. An irregular, 4 × 3–in (at largest diameter), serohemorrhagic blister is discovered on the medial aspect of the lower leg, above the right malleolus (see Figure 1). Multiple 1- to 3-mm vesicles surround much of the anterior border. Moderate edema is noted from the top of the lesion to the midfoot, concentrated around the lateral and medial malleolus. Extensive blue, purple, and black discoloration is seen below the malleolus. The patient is diagnosed with a fracture blister.

DISCUSSION

Fracture blisters are taut, bullous, subepidermal vesicles that can accompany fractures or severe twisting injuries. They overlie markedly edematous soft tissue and histologically resemble a second-degree burn.1,2

Physiologically, blisters are caused by increased interstitial pressure due to swelling, with subsequent increased filtration pressure and colloid osmotic pressure in the epidermal gap.3 This causes a disruption that allows fluid to move into the weakened area.3 Areas most at risk for fracture blister formation are those with tight, closely adhered skin without muscle or enveloping fascia, where there is less soft tissue between the skin and bone prominences (eg, ankle, elbow, foot, distal tibia).2-4

Approximately 3% of all patients with acute fractures requiring hospitalization develop a fracture blister.4 Any condition that predisposes a patient to poor wound healing (eg, peripheral vascular disease, diabetes, hypertension) increases risk for a fracture blister.2 Recognizing which patients are at greatest risk is vital, as implementing prevention strategies and intervening when fracture blisters do form can help decrease complications—including infection and delayed surgery—and improve fracture resolution. In this patient’s case, the extent of the injury and force of the fall caused the fracture blister to form.

Diagnosis

Diagnosis of a fracture blister is based on clinical presentation. There are two types: hemorrhagic blisters and clear fluid-filled blisters. Hemorrhagic blisters indicate more severe injury and longer healing time (approximately 16 d), while clear fluid-filled blisters demonstrate minimal injury and therefore are quicker to heal.2,4

The differential diagnosis for fracture blisters includes friction blisters and disorders such as epidermolysis bullosa and bullous pemphigoid. Friction blisters form when the epidermis is subjected to repeated friction or shear forces (eg, from a cast or splint).5,6 These forces mechanically separate epidermal cells at the stratum spinosum layer.7 The pressure that moves across the skin forces fluid into the deeper open spaces, filling them but leaving the surface layer intact.1

Epidermolysis bullosa (EB) is a group of rare inherited cutaneous and mucus membrane disorders. EB involves fragility and detachment of subepithelial tissues, which results in blistering and erosions.8,9 The blisters tend to develop in areas subject to minor trauma, such as the extensor aspects of the elbows and the dorsal aspects of the hands and feet.9 They can also be triggered by exposure to heat, friction, scratching, and adhesive tape.10

Bullous pemphigoid, a chronic autoimmune skin disorder, is characterized by pruritic, bullous lesions. When IgG autoantibodies bind to certain hemidesmosomal antigens, complement activation causes a subepidermal blister.11While bullous pemphigoid most commonly affects those older than 60, it can also occur in children. Diagnosis is confirmed by skin biopsy and immunofluorescence testing.11

Treatment and management

Although several recommendations have been published, there is no gold standard and treatment of fracture blisters remains controversial. Early surgical intervention for fractures could decrease the incidence of fracture blisters.1,3

The goal of treatment is to achieve re-epithelialization of the dermis.3,12,13 Once a blister forms, management techniques vary. Some recommend keeping closed blisters covered with a dry dressing to protect them from damage.3 Strauss et al recommend unroofing to avoid traumatic rupture; however, this does increase risk for infection.12 Recommendations differ depending on provider preference and each patient’s individual situation. 

Elective unroofing of a blister is typically followed with one of several treatment options. These include covering the open blister with a topical antibiotic cream (eg, silver sulfadiazine 2%); applying a nonadherent, occlusive bismuth-tribromophenate-petroleum gauze dressing; or elevating and immobilizing the affected extremity.12,13

 

 

Treatment of spontaneously ruptured fracture blisters entails

  • Unroofing the blister completely and applying a topical antimicrobial (eg, silver sulfadiazine, polymyxin B, neomycin, bacitracin).
  • Applying a hydrocolloid dressing to keep the environment moist.
  • Using a first-aid gel containing melaleuca (tea tree) oil.
  • Initiating prophylactic oral antibiotics.
  • Using whirlpool treatments.
  • Elevating and immobilizing the affected extremity.3,12,14

OUTCOME FOR THE CASE PATIENT

The fracture blister was electively unroofed (see Figure 2) based on provider preference. The patient was instructed to clean the wound daily and apply topical cream (silver sulfadiazine 2% bid) to the wound and cover it with gauze. The patient was made non-weight-bearing to the right lower extremity. Continuous elevation was highly encouraged except for bathing and restroom use, and an NSAID was recommended as needed for pain. She was reassessed the following day and, due to partial refilling, the blister required additional unroofing. The patient was instructed to resume previous wound care orders.

No surgical intervention was required. CT of the right foot and ankle without contrast (performed on day 4 postinjury) confirmed a nondisplaced transverse fracture of the medial malleolus and a sagittal avulsion fracture of the anterior-inferior lateral malleolus. Multiple smaller fracture fragments were noted posterior and medial to the medial malleolus as well as inferiorly along the course of the deltoid ligament. There was a small, nondisplaced avulsion fracture of the medial malleolus at the anterolateral and posterolateral tibial plafond.

Due to the extent of the swelling, multiple fractures, and blister formation, the patient was essentially bed bound for the first three weeks; complete resolution of the fracture blister occurred 21 days after initial discovery (see Figure 3). The patient did not experience cutaneous complications. Her lower extremity was then casted in a short-leg removable cast for 10 weeks. She underwent physical therapy, and after 12 weeks, the patient was weight-bearing and was discharged from orthopedics. The patient reported refractory pain and swelling for an additional eight weeks following injury, warranting daily ibuprofen.

CONCLUSION

Fracture blisters are rare, and experience and knowledge about them in primary care is lacking. But clinicians need to be able to identify, diagnose, and refer at-risk patients to orthopedics in a timely manner.

Current management and treatment recommendations are inconsistent. Treatment varies depending on the site, severity, type, and status of the blister and the overall health of the patient. Fracture blisters may be left intact, electively unroofed, or treated after spontaneous rupture. More research is needed to clarify management recommendations, specifically regarding the decision to unroof a blister or leave it intact. Early surgical intervention may prevent the development of a fracture blister.

IN THIS ARTICLE

  • Diagnosis
  • Treatment
  • Care outcome

A 57-year-old horticulturist is working on a ladder leaned up against a tree trunk when the ladder slips, causing her to fall six feet onto concrete. Her right foot and ankle sustain the force of the fall; she is in excruciating pain and unable to bear weight on the foot. She is immediately transported to a local emergency department for evaluation.

Physical exam reveals a tearful middle-aged female in moderate distress and acute pain. There is moderate swelling of the right medial and lateral malleolus, as well as the midfoot, with blue and purple discoloration on the medial and lateral malleolus. Radiographs of the right ankle identify nondisplaced fractures of the distal fibula and tibia. Foot x-rays are unremarkable. A splint is ordered. The patient is given crutches (non-weight-bearing status), pain medication, and a referral to orthopedics.

On day 3, the patient presents to orthopedics, where the splint is removed. An irregular, 4 × 3–in (at largest diameter), serohemorrhagic blister is discovered on the medial aspect of the lower leg, above the right malleolus (see Figure 1). Multiple 1- to 3-mm vesicles surround much of the anterior border. Moderate edema is noted from the top of the lesion to the midfoot, concentrated around the lateral and medial malleolus. Extensive blue, purple, and black discoloration is seen below the malleolus. The patient is diagnosed with a fracture blister.

DISCUSSION

Fracture blisters are taut, bullous, subepidermal vesicles that can accompany fractures or severe twisting injuries. They overlie markedly edematous soft tissue and histologically resemble a second-degree burn.1,2

Physiologically, blisters are caused by increased interstitial pressure due to swelling, with subsequent increased filtration pressure and colloid osmotic pressure in the epidermal gap.3 This causes a disruption that allows fluid to move into the weakened area.3 Areas most at risk for fracture blister formation are those with tight, closely adhered skin without muscle or enveloping fascia, where there is less soft tissue between the skin and bone prominences (eg, ankle, elbow, foot, distal tibia).2-4

Approximately 3% of all patients with acute fractures requiring hospitalization develop a fracture blister.4 Any condition that predisposes a patient to poor wound healing (eg, peripheral vascular disease, diabetes, hypertension) increases risk for a fracture blister.2 Recognizing which patients are at greatest risk is vital, as implementing prevention strategies and intervening when fracture blisters do form can help decrease complications—including infection and delayed surgery—and improve fracture resolution. In this patient’s case, the extent of the injury and force of the fall caused the fracture blister to form.

Diagnosis

Diagnosis of a fracture blister is based on clinical presentation. There are two types: hemorrhagic blisters and clear fluid-filled blisters. Hemorrhagic blisters indicate more severe injury and longer healing time (approximately 16 d), while clear fluid-filled blisters demonstrate minimal injury and therefore are quicker to heal.2,4

The differential diagnosis for fracture blisters includes friction blisters and disorders such as epidermolysis bullosa and bullous pemphigoid. Friction blisters form when the epidermis is subjected to repeated friction or shear forces (eg, from a cast or splint).5,6 These forces mechanically separate epidermal cells at the stratum spinosum layer.7 The pressure that moves across the skin forces fluid into the deeper open spaces, filling them but leaving the surface layer intact.1

Epidermolysis bullosa (EB) is a group of rare inherited cutaneous and mucus membrane disorders. EB involves fragility and detachment of subepithelial tissues, which results in blistering and erosions.8,9 The blisters tend to develop in areas subject to minor trauma, such as the extensor aspects of the elbows and the dorsal aspects of the hands and feet.9 They can also be triggered by exposure to heat, friction, scratching, and adhesive tape.10

Bullous pemphigoid, a chronic autoimmune skin disorder, is characterized by pruritic, bullous lesions. When IgG autoantibodies bind to certain hemidesmosomal antigens, complement activation causes a subepidermal blister.11While bullous pemphigoid most commonly affects those older than 60, it can also occur in children. Diagnosis is confirmed by skin biopsy and immunofluorescence testing.11

Treatment and management

Although several recommendations have been published, there is no gold standard and treatment of fracture blisters remains controversial. Early surgical intervention for fractures could decrease the incidence of fracture blisters.1,3

The goal of treatment is to achieve re-epithelialization of the dermis.3,12,13 Once a blister forms, management techniques vary. Some recommend keeping closed blisters covered with a dry dressing to protect them from damage.3 Strauss et al recommend unroofing to avoid traumatic rupture; however, this does increase risk for infection.12 Recommendations differ depending on provider preference and each patient’s individual situation. 

Elective unroofing of a blister is typically followed with one of several treatment options. These include covering the open blister with a topical antibiotic cream (eg, silver sulfadiazine 2%); applying a nonadherent, occlusive bismuth-tribromophenate-petroleum gauze dressing; or elevating and immobilizing the affected extremity.12,13

 

 

Treatment of spontaneously ruptured fracture blisters entails

  • Unroofing the blister completely and applying a topical antimicrobial (eg, silver sulfadiazine, polymyxin B, neomycin, bacitracin).
  • Applying a hydrocolloid dressing to keep the environment moist.
  • Using a first-aid gel containing melaleuca (tea tree) oil.
  • Initiating prophylactic oral antibiotics.
  • Using whirlpool treatments.
  • Elevating and immobilizing the affected extremity.3,12,14

OUTCOME FOR THE CASE PATIENT

The fracture blister was electively unroofed (see Figure 2) based on provider preference. The patient was instructed to clean the wound daily and apply topical cream (silver sulfadiazine 2% bid) to the wound and cover it with gauze. The patient was made non-weight-bearing to the right lower extremity. Continuous elevation was highly encouraged except for bathing and restroom use, and an NSAID was recommended as needed for pain. She was reassessed the following day and, due to partial refilling, the blister required additional unroofing. The patient was instructed to resume previous wound care orders.

No surgical intervention was required. CT of the right foot and ankle without contrast (performed on day 4 postinjury) confirmed a nondisplaced transverse fracture of the medial malleolus and a sagittal avulsion fracture of the anterior-inferior lateral malleolus. Multiple smaller fracture fragments were noted posterior and medial to the medial malleolus as well as inferiorly along the course of the deltoid ligament. There was a small, nondisplaced avulsion fracture of the medial malleolus at the anterolateral and posterolateral tibial plafond.

Due to the extent of the swelling, multiple fractures, and blister formation, the patient was essentially bed bound for the first three weeks; complete resolution of the fracture blister occurred 21 days after initial discovery (see Figure 3). The patient did not experience cutaneous complications. Her lower extremity was then casted in a short-leg removable cast for 10 weeks. She underwent physical therapy, and after 12 weeks, the patient was weight-bearing and was discharged from orthopedics. The patient reported refractory pain and swelling for an additional eight weeks following injury, warranting daily ibuprofen.

CONCLUSION

Fracture blisters are rare, and experience and knowledge about them in primary care is lacking. But clinicians need to be able to identify, diagnose, and refer at-risk patients to orthopedics in a timely manner.

Current management and treatment recommendations are inconsistent. Treatment varies depending on the site, severity, type, and status of the blister and the overall health of the patient. Fracture blisters may be left intact, electively unroofed, or treated after spontaneous rupture. More research is needed to clarify management recommendations, specifically regarding the decision to unroof a blister or leave it intact. Early surgical intervention may prevent the development of a fracture blister.

References

1. Wallace GF, Sullivan J. Fracture blisters.  Clin Podiatr Med Surg. 1995;12(4):801-811.
2. Halawi MJ. Fracture blisters after primary total knee arthroplasty. Am J Orthop. 2015; 44(8):E291-E293.
3. McCann S, Gruen G. Fracture blisters: a review of the literature. Orthop Nurs. 1997; 16(2):17-24.
4. Uebbing CM, Walsh M, Miller JB, et al.  Fracture blister. West J Emerg Med. 2011; 12(1):131-133.
5. Kirkham S, Lam S, Nester C, Hashmi F. The effect of hydration on the risk of friction blister formation on the heel of the foot. Skin Res Tech. 2014;20:246-253.
6. Boyd A, Benjamin H, Asplund C. Principles of casting and splinting. Am Fam Physician. 2009;79(1):16-24.
7. Knapik J, Reynolds K, Duplantis K, Jones B. Friction blisters. Pathophysiology, prevention and treatment. Sports Med. 1995; 20(3):136-147.
8. Iranzo P, Herrero-González JE, Mascaró-Galy JM, et al. Epidermolysis bullosa acquisita: a retrospective analysis of 12 patients evaluated in four tertiary hospitals in Spain. Br J Dermatol. 2014;171(5):1022-1030.
9. Peraza DM. Epidermolysis bullosa acquisita. Merck Manual Professional Version. August 2016. www.merckmanuals.com/professional/dermatologic-disorders/bullous-diseases/epidermolysis-bullosa-acquisita. Accessed January 26, 2018.
10. Lyons F, Ousley L. Dermatology for the Advanced Practice Nurse. New York, NY: Springer; 2015.
11. Peraza D. Bullous pemphigoid. Merck Manual Professional Version. August 2016. www.merckmanuals.com/professional/dermatologic-disorders/bullous-diseases/bullous-pemphigoid. Accessed January 26, 2018.
12. Strauss EJ, Petrucelli G, Bong M, et al. Blisters associated with lower-extremity fracture: Results of a prospective treatment protocol. J Orthop Trauma. 2006;20(9): 618-622.
13. Tolpinrud WL, Rebolledo BJ, Lorich DG, Grossman ME. A case of extensive fracture bullae: a multidisciplinary approach for acute management. JAAD Case Rep. 2015;1(3):132-135.
14. Cox H, Nealon L. Case report: the use of Burnaid Gel on fracture blisters. Wound Practice and Research. 2008;16(1):32-36.

References

1. Wallace GF, Sullivan J. Fracture blisters.  Clin Podiatr Med Surg. 1995;12(4):801-811.
2. Halawi MJ. Fracture blisters after primary total knee arthroplasty. Am J Orthop. 2015; 44(8):E291-E293.
3. McCann S, Gruen G. Fracture blisters: a review of the literature. Orthop Nurs. 1997; 16(2):17-24.
4. Uebbing CM, Walsh M, Miller JB, et al.  Fracture blister. West J Emerg Med. 2011; 12(1):131-133.
5. Kirkham S, Lam S, Nester C, Hashmi F. The effect of hydration on the risk of friction blister formation on the heel of the foot. Skin Res Tech. 2014;20:246-253.
6. Boyd A, Benjamin H, Asplund C. Principles of casting and splinting. Am Fam Physician. 2009;79(1):16-24.
7. Knapik J, Reynolds K, Duplantis K, Jones B. Friction blisters. Pathophysiology, prevention and treatment. Sports Med. 1995; 20(3):136-147.
8. Iranzo P, Herrero-González JE, Mascaró-Galy JM, et al. Epidermolysis bullosa acquisita: a retrospective analysis of 12 patients evaluated in four tertiary hospitals in Spain. Br J Dermatol. 2014;171(5):1022-1030.
9. Peraza DM. Epidermolysis bullosa acquisita. Merck Manual Professional Version. August 2016. www.merckmanuals.com/professional/dermatologic-disorders/bullous-diseases/epidermolysis-bullosa-acquisita. Accessed January 26, 2018.
10. Lyons F, Ousley L. Dermatology for the Advanced Practice Nurse. New York, NY: Springer; 2015.
11. Peraza D. Bullous pemphigoid. Merck Manual Professional Version. August 2016. www.merckmanuals.com/professional/dermatologic-disorders/bullous-diseases/bullous-pemphigoid. Accessed January 26, 2018.
12. Strauss EJ, Petrucelli G, Bong M, et al. Blisters associated with lower-extremity fracture: Results of a prospective treatment protocol. J Orthop Trauma. 2006;20(9): 618-622.
13. Tolpinrud WL, Rebolledo BJ, Lorich DG, Grossman ME. A case of extensive fracture bullae: a multidisciplinary approach for acute management. JAAD Case Rep. 2015;1(3):132-135.
14. Cox H, Nealon L. Case report: the use of Burnaid Gel on fracture blisters. Wound Practice and Research. 2008;16(1):32-36.

Issue
Clinician Reviews - 28(3)
Issue
Clinician Reviews - 28(3)
Page Number
29-30,32-33
Page Number
29-30,32-33
Publications
Publications
Topics
Article Type
Display Headline
Woman, 57, With Painful, Swollen Ankle
Display Headline
Woman, 57, With Painful, Swollen Ankle
Sections
Disallow All Ads
Content Gating
No Gating (article Unlocked/Free)
Alternative CME
Disqus Comments
Default
Use ProPublica
Article PDF Media

A 67-year-old woman with bilateral hand numbness

Article Type
Changed
Tue, 05/03/2022 - 15:20
Display Headline
A 67-year-old woman with bilateral hand numbness

A 67-year-old woman presents to the emergency department after 8 weeks of progressive numbness and tingling in both hands, involving all fingers. The numbness has increased in severity in the last 3 days. She also has occasional numbness around her mouth. She reports no numbness in her feet.

She says she underwent thyroid surgery twice for thyroid cancer 10 years ago. Her medical history also includes type 2 diabetes mellitus (diagnosed 1 year ago), hypertension, dyslipidemia, and diastolic heart failure (diagnosed 5 years ago).

Her current medications are:

  • Metformin 1 g twice a day
  • Candesartan 16 mg once a day
  • Atorvastatin 20 mg once a day
  • Furosemide 40 mg twice a day
  • Levothyroxine 100 μg per day
  • Calcium carbonate 1,500 mg twice a day
  • A vitamin D tablet twice a day, which she has not taken for the last 2 months.

She admits she has not been taking her medications regularly because she has been feeling depressed.

On physical examination, she is alert and oriented but appears anxious. She is not in res­piratory distress. Her blood pressure is 150/90 mm Hg and her pulse is 92 beats per minute and regular. There is a thyroidectomy scar on the anterior neck. Her jugular venous pressure is not elevated. Her heart sounds are normal without extra sounds. She has no pulmonary rales and no lower-extremity edema.

The Phalen test and Tinel test for carpal tunnel syndrome are negative in both hands. Using a Katz hand diagram, the patient reports tingling and numbness in all fingers, both palms, and the dorsum of both hands. Tapping the area over the facial nerve does not elicit twitching of the facial muscles (ie, no Chvostek sign), but compression of the upper arm elicits carpal spasm (ie, positive Trousseau sign). There is no evidence of motor weakness in her hands. The rest of the physical examination is unremarkable.

POSSIBLE CAUSES OF NUMBNESS

1. Based on the initial evaluation, which of the following is the most likely cause of our patient’s bilateral hand numbness?

  • Hypocalcemia due to primary hypoparathyroidism
  • Carpal tunnel syndrome due to primary hypothyroidism
  • Diabetic peripheral neuropathy
  • Vitamin B12 deficiency due to metformin
  • Hypocalcemia due to low serum calcitonin

All the conditions above except low serum calcitonin can cause bilateral hand paresthesia. Our patient most likely has hypocalcemia due to primary hypoparathyroidism.

Hypocalcemia

In our patient, bilateral hand numbness and perioral numbness after stopping vitamin D and a positive Trousseau sign strongly suggest hypocalcemia. The classic physical findings in patients with hypocalcemia are the Trousseau sign and the Chvostek sign. The Trousseau sign is elicited by inflating a blood pressure cuff above the systolic blood pressure for 3 minutes and observing for ischemia-induced carpopedal spasm, wrist and metacarpophalangeal joint flexion, thumb adduction, and interphalangeal joint extension. The Chvostek sign is elicited by tapping over the area of the facial nerve below the zygoma in front of the tragus, resulting in ipsilateral twitching of facial muscles.

Although the Trousseau sign is more sensitive and specific than the Chvostek sign, neither is pathognomonic for hypocalcemia.1 The Chvostek sign has been reported to be negative in 30% of patients with hypocalcemia and positive in 10% of normocalcemic individuals.1 The Trousseau sign, however, is present in 94% of hypocalcemic patients vs 1% of normocalcemic individuals.2

Primary hypoparathyroidism secondary to thyroidectomy. Postsurgical hypoparathyroidism is the most common cause of primary hypoparathyroidism. It results from ischemic injury or accidental removal of the parathyroid glands during anterior neck surgery.3,4 The consequent hypocalcemia can be transient, intermittent, or permanent. Permanent postsurgical hypoparathyroidism is defined as persistent hypocalcemia with insufficient parathyroid hormone (PTH) for more than 12 months after neck surgery; however, some consider 6 months to be enough to define the condition.5–7

The incidence of postsurgical hypoparathyroidism varies considerably with the extent of thyroid surgery and the experience of the surgeon.6,8 In the hands of experienced surgeons, permanent hypoparathyroidism occurs in fewer than 1% of patients after total thyroidectomy, whereas the rate may be higher than 6% with less-experienced surgeons.5,9 Other risk factors for postsurgical hypoparathyroidism include female sex, autoimmune thyroid disease, pregnancy, and lactation.5

Pseudohypoparathyroidism is a group of disorders characterized by renal resistance to PTH, leading to hypocalcemia, hyperphosphatemia, and elevated serum PTH. It is also associated with phenotypic features such as short stature and short fourth metacarpal bones.

Calcitonin deficiency. Calcitonin is a polypeptide hormone secreted from the parafollicular (C) cells of the thyroid gland. After total thyroidectomy, calcitonin levels are expected to be reduced. However, the role of calcitonin in humans is unclear. One study has shown that calcitonin is possibly a vestigial hormone, given that no calcitonin-related disorders (excess or deficiency) have been reported in humans.10

 

 

Carpal tunnel syndrome due to hypothyroidism

Our patient also could have primary hypothyroidism as a result of thyroidectomy. Hypothyroidism can cause bilateral hand numbness due to carpal tunnel syndrome, which is mediated by mucopolysaccharide deposition and synovial membrane swelling.11 One study reported that 29% of patients with hypothyroidism had carpal tunnel syndrome.12 Symptoms of carpal tunnel syndrome in hypothyroid patients may occur despite thyroid replacement therapy.13

Katz hand diagram classification of carpal tunnel syndrome

Carpal tunnel syndrome is a clinical diagnosis. Patients usually experience hand paresthesia in the distribution of the median nerve. Provocative physical tests for carpal tunnel syndrome include the Tinel test, the Phalen test, and the Katz hand diagram, which is considered the best of the 3 tests.14,15 Based on how the patient marks the location and type of symptoms on the diagram, carpal tunnel syndrome is rated as classic, probable, possible, or unlikely (Table 1).14,16,17 The sensitivity of a classic or probable diagram ranges from 64% to 80%, while the specificity ranges from 73% to 90%.14,15

Carpal tunnel syndrome is less likely to be the cause of our patient’s symptoms, as her Katz hand diagram indicates only “possible” carpal tunnel syndrome. Her perioral numbness and positive Trousseau sign make hypocalcemia a more likely cause.

Diabetic peripheral neuropathy

Sensory peripheral neuropathy is a recognized complication of diabetes mellitus. However, neuropathy in diabetic patients most commonly manifests initially as distal symmetrical ascending neuropathy starting in the lower extremities.18 Therefore, diabetic peripheral neuropathy is less likely in this patient since her symptoms are limited to her hands.

Vitamin B12 deficiency

Metformin-induced vitamin B12 deficiency is another possible cause of peripheral neuropathy. It might be secondary to metformin-induced changes in intrinsic factor levels and small-intestine motility with resultant bacterial overgrowth, as well as inhibition of vitamin B12 absorption in the terminal ileum.19

However, metformin-induced vitamin B12 deficiency is not the most likely cause of our patient’s neuropathy, since she has been taking this drug for only 1 year. Vitamin B12 deficiency with consequent peripheral neuropathy is more likely in patients taking metformin in high doses for 10 or more years.20

Laboratory results and electrocardiography

Results of initial laboratory testing

Table 2 shows the patient’s initial laboratory results. Of note, her serum calcium level is 5.7 mg/dL (reference range 8.9–10.1). Electrocardiography in the emergency department shows:

  • Prolonged PR interval (23 msec)
  • Wide QRS complexes (13 msec)
  • Flat T waves
  • Prolonged corrected QT interval (475 msec)
  • Occasional premature ventricular complexes.

CLINICAL MANIFESTATIONS OF HYPOCALCEMIA

2. Which of the following is not a manifestation of hypocalcemia?

  • Tonic-clonic seizures
  • Cyanosis
  • Cardiac ventricular arrhythmias
  • Acute pancreatitis
  • Depression

 Clinical manifestations of hypocalcemia

Hypocalcemia can cause a wide range of clinical manifestations (Table 3), the extent and severity of which depend on the severity of hypocalcemia and how quickly it develops. The more acute the hypocalcemia, the more severe the manifestations.21

Tetany can cause seizures

Hypocalcemia is characterized by neuromuscular hyperexcitability, manifested clinically by tetany.22 Manifestations of tetany are numerous and include acral paresthesia, perioral numbness, muscle cramps, carpopedal spasm, and seizures. Tetany is the hallmark of hypocalcemia regardless of etiology. However, certain causes are associated with peculiar clinical manifestations. For example, chronic primary hypoparathyroidism may be associated with basal ganglia calcifications that can result in parkinsonism, other extrapyramidal disorders, and dementia (Table 4).6

Clinical manifestations of chronic primary hypoparathyroidism

Airway spasm can be fatal

A serious manifestation of acute severe hypocalcemia is spasm of the glottis muscles, which may cause cyanosis and, if untreated, death.21

Ventricular arrhythmias

Another potential fatal complication of acute severe hypocalcemia is polymorphic ventricular tachycardia due to prolongation of the QT interval, which is readily identified with electrocardiography.23

Hypocalcemia does not cause pancreatitis

Hypercalcemia, rather than hypocalcemia, may cause acute pancreatitis.24 Conversely, acute pancreatitis may cause hypocalcemia due to precipitation of calcium in the abdominal cavity.25

Psychiatric manifestations

In addition to depression, hypocalcemia is associated with psychiatric manifestations including anxiety, confusion, and emotional instability.

 

 

STEPS TO DIAGNOSIS OF HYPOCALCEMIA

First step: Confirm true hypocalcemia

Calcium circulates in the blood in 3 forms: bound to albumin (40% to 45%), bound to anions (10% to 15%), and free (ionized) (45%). Although ionized calcium is the active form, most laboratories report total serum calcium.

Since changes in serum albumin concentration affect the total serum calcium level, it is imperative to correct the measured serum calcium to the serum albumin concentration. Each 1-g/dL decrease in serum albumin lowers the total serum calcium by 0.8 mg/dL. Thus:

Corrected serum calcium (mg/dL) =
measured total serum calcium (mg/dL) +
0.8 (4 − serum albumin [g/dL])
.

If the patient’s serum calcium level remains low when corrected for serum albumin, he or she has true hypocalcemia, which implies a low ionized serum calcium. Conversely, pseudohypocalcemia means that the measured calcium level is low but the corrected serum calcium is normal.

Using this formula, our patient’s corrected calcium level is calculated as 5.7 + 0.8 (4 – 3.2) = 6.3 mg/dL, indicating true hypocalcemia.

PHOSPHATE IS OFTEN HIGH WHEN CALCIUM IS LOW

In addition to hypocalcemia, our patient has an elevated phosphate level (Table 2).

3. Which of the following hypocalcemic disorders is not associated with hyperphosphatemia?

  • End-stage renal disease
  • Primary hypoparathyroidism
  • Pseudohypoparathyroidism
  • Vitamin D3 deficiency
  • Rhabdomyolysis

Vitamin D deficiency is not associated with hyperphosphatemia.

Second step in evaluating hypocalcemia: Check phosphate, magnesium, creatinine

Major causes of hypocalcemia, according to phosphate level

The major causes of hypocalcemia can be categorized according to the serum phosphate level: high vs normal or low (Table 5).

High-phosphate, low-calcium states. In the absence of concurrent end-stage renal disease and an excessive phosphate load, primary hypoparathyroidism is the most likely cause of hypocalcemia associated with hyperphosphatemia.

PTH increases serum ionized calcium by26,27:

  • Increasing bone resorption
  • Increasing reabsorption of calcium from the distal renal tubules
  • Increasing the activity of 1-alpha-hydroxylase, responsible for conversion of 25-hydroxyvitamin D3 to 1,25-dihydroxyvitamin D3 (the most biologically active vitamin D metabolite); 1,25-dihydroxyvitamin D increases the absorption of calcium and phosphate from the intestine.

Conversely, PTH decreases reabsorption of phosphate from proximal renal tubules, resulting in hypophosphatemia. Therefore, low serum PTH (primary hypoparathyroidism) or a PTH-resistant state (pseudohypoparathyroidism) results in hypocalcemia and hyperphosphatemia.26,27

Both end-stage renal disease and rhabdomyolysis are associated with high serum phosphate levels. The kidney normally excretes excess dietary phosphate to maintain phosphate homeostasis; however, this is impaired in end-stage renal disease, leading to hyperphosphatemia. In rhabdomyolysis, it is mainly the transcellular shift of phosphate into the extracellular space from myocyte injury that raises phosphate levels.

Normal- or low-phosphate, low calcium states. Hypocalcemia can also result from vitamin D deficiency, but this cause is associated with a low or normal serum phosphate level. In such cases, hypocalcemia causes secondary hyperparathyroidism with consequent renal phosphate loss and, thus, hypophosphatemia.27

Third step: Check serum intact PTH and 25-hydroxyvitamin D levels

Hypocalcemia stimulates secretion of PTH. Therefore, hypocalcemia with elevated serum PTH is caused by disorders that do not impair PTH secretion, including chronic renal failure and vitamin D deficiency (Table 5). Conversely, hypocalcemia with low or normal serum PTH levels suggests primary hypoparathyroidism.

Our patient’s serum PTH level is 20 ng/mL, which is within the reference range. This does not discount the diagnosis of primary hypoparathyroidism. Although most patients with primary hypoparathyroidism have low or undetectable serum PTH levels, some have normal PTH levels if some degree of PTH production is preserved.5,7,28–30 In these patients, the remaining functioning parathyroid tissue is not enough to maintain a normal serum calcium level, resulting in hypocalcemia. As a result, hypocalcemia stimulates the remaining parathyroid tissue to its maximum output, producing PTH levels usually within the lower or middle-normal range.30 In such patients, the terms parathyroid insufficiency and relative primary hypoparathyroidism are more precise than primary hypoparathyroidism.

Postsurgical hypoparathyroidism with an inappropriately normal PTH level is usually seen in patients with disorders that impair intestinal calcium absorption or bone resorption.31 In our patient’s case, the “normal” serum PTH level is likely due to maximal stimulation of remaining functioning parathyroid tissue by severe hypocalcemia, which is a result of her discontinuation of calcium and calcitriol therapy and her vitamin D deficiency.

 

 

CASE RESUMED: NO RESPONSE TO INTRAVENOUS CALCIUM GLUCONATE

The patient is given 2 10-mL ampules of 10% calcium gluconate diluted in 100 mL of 5% dextrose in water over 20 minutes intravenously. Electrocardiographic monitoring is continued. Two hours later, her measured serum calcium is only 5.8 mg/dL, with no improvement in her symptoms.

A continuous infusion of calcium gluconate is started: 12 ampules of calcium gluconate are added to 380 mL of 5% dextrose in water and infused at 40 mL/hour (infused rate of elemental calcium = 1.3 mg/kg/hour); 3 hours later, her measured serum calcium level is still only 5.8 mg//dL; at 6 hours it is 5.9 mg/dL, and her symptoms have not improved.

4. Which of the following is the most appropriate next step?

  • Change the calcium gluconate to calcium chloride
  • Increase the infusion rate to 1.5 mg of elemental calcium/kg/hour
  • Give a bolus of 2 10-mL ampules of 10% calcium gluconate intravenously over 1 minute
  • Give additional oral calcium tablets
  • Check the serum magnesium level

Treatment of hypocalcemia can involve intravenous or oral calcium therapy.

Intravenous calcium is indicated for patients with any of the following6,32:

  • Moderate to severe neuromuscular irritability (eg, acral paresthesia, carpopedal spasm, prolonged QT interval, seizures, laryngospasm, bronchospasm)
  • Acute hypocalcemia with corrected serum calcium level less than 7.6 mg/dL, even if the patient is asymptomatic
  • Cardiac failure.

One 10-mL ampule of 10% calcium gluconate contains 93 mg of elemental calcium; 1 or 2 ampules are typically diluted in 50 to 100 mL of 5% dextrose in water and infused slowly over 15 to 20 minutes. Rapid administration of intravenous calcium is contraindicated, as it may produce cardiac arrhythmias and possibly cardiac arrest. Therefore, intravenous calcium should be given slowly while continuing electrocardiographic monitoring.33

Since the effect of 1 ampule of calcium gluconate lasts only 2 to 3 hours, most patients with symptomatic hypocalcemia require continuous intravenous calcium infusion. The recommended dose of infused elemental calcium is 0.5 to 1.5 mg/kg/hour.34 Several ampules are added to 500 to 1,000 mL of 5% dextrose in water or 0.9% normal saline and infused at a rate appropriate for the patient’s corrected calcium and symptoms.

Oral calcium and vitamin D supplements can be given initially to patients with a corrected serum calcium level of 7.6 mg/dL or greater, with or without mild symptoms, if they can tolerate oral intake. However, this is not the treatment of choice for resistant acute hypocalcemia, as in this case.

Calcium chloride has no advantages over calcium gluconate. Further, it can be associated with local irritation and may result in tissue necrosis if extravasation occurs.35

Increasing the infusion rate of calcium gluconate to the maximum recommended dose may improve the patient’s ionized calcium level and symptoms somewhat. However, it is not the best option for this patient, given that she did not respond to 2 ampules of calcium gluconate followed by continuous infusion of 1.3 mg/kg/hour for 6 hours.

Calcium gluconate bolus. Similarly, giving the patient an additional 2 ampules of calcium gluconate over 1 minute would not be recommended, as rapid administration of intravenous calcium gluconate (eg, over 1 minute) is contraindicated.

Check magnesium

If hypocalcemia persists despite intravenous calcium therapy, as in our patient, further investigation or action is required. An important cause of persistent hypocalcemia is severe hypomagnesemia. Severe hypomagnesemia (serum magnesium < 0.8 mg/dL) causes resistant hypocalcemia by several mechanisms:

  • Inducing PTH resistance32,36,37
  • Decreasing PTH secretion32,36
  • Decreasing calcitriol production.

The decrease in calcitriol production is a direct effect of hypomagnesemia, but it is also an indirect effect of low PTH secretion, which inhibits the enzyme 1-alpha-hydroxylase. Thus, conversion of 25-hydroxyvitamin D3 to calcitriol is impaired, leading to low calcitriol production.

Our patient could have hypomagnesemia due to furosemide use and uncontrolled diabetes mellitus. Hypocalcemia resistant to calcium therapy may occasionally respond to magnesium therapy even if the serum magnesium level is normal. This may be due to depleted intracellular magnesium salt levels.6,38 Rarely, severe hypermagnesemia can also be associated with hypocalcemia due to inhibition of PTH secretion.37,39

CASE RESUMED

Our patient’s serum magnesium level is 0.6 mg/dL (reference range 1.7–2.4 mg/dL). She is given 2 g of magnesium sulfate in 60 mL of 0.9% normal saline infused over 1 hour, followed by a continuous infusion of magnesium sulfate (12 g diluted in 250 mL of 0.9% normal saline, infused over 24 hours). On repeat testing 4 hours later, her serum magnesium level is 0.7 mg/dL, and at 8 hours later it is 0.9 mg/dL. She is subsequently started on oral magnesium oxide 600 mg per day. The magnesium sulfate infusion is continued for another 24 hours.

PREVENTING HYPERCALCIURIA

Patients with low PTH (primary hypoparathyroidism) may have hypercalciuria due to decreased renal tubular calcium reabsorption. Two important measures can minimize hypercalciuria in such patients:

  • Keeping the serum calcium level in the low-normal range4,5,40
  • Giving a thiazide diuretic (eg, hydrochlorothiazide 12.5–50 mg daily) with a low-salt diet.41,42

A thiazide diuretic is usually started once the 24-hour urine calcium reaches 250 mg.6 Thiazides are thought to enhance both proximal and distal renal tubular calcium reabsorption.43,44

PRIMARY HYPOPARATHYROIDISM: LONG-TERM MANAGEMENT

Long-term management of primary hypoparathyroidism requires calcium and vitamin D supplementation.

Calcium supplements. The most commonly prescribed calcium preparations are calcium carbonate and calcium citrate (containing 40% and 20% elemental calcium, respectively). Calcium carbonate, which is less expensive than calcium citrate, binds with phosphate intake and requires an acidic environment for absorption, and so it is better absorbed when taken with meals. Because calcium citrate does not require an acidic environment for absorption, it is the calcium preparation of choice for patients on proton pump inhibitors, or patients with achlorhydria or constipation.45 Calcium doses vary widely, with most hypoparathyroid patients requiring 1 to 2 g of elemental calcium daily.6

Vitamin D supplements. To promote intestinal absorption, calcium is combined with vitamin D in a fixed-dose preparation given in divided doses.46 Calcitriol (1,25-dihydroxyvitamin D3) is the most active metabolite of vitamin D, with rapid onset and offset of action, and it is the preferred form of vitamin D therapy for patients with hypoparathyroidism. If calcitriol is not available or is not affordable, alphacalcidol (1-alpha-hydroxyvitamin D3) is another option. This is a synthetic analogue of vitamin D that is already hyroxylated at the C1 position. After oral intake, it is hydroxylated in the liver to form calcitriol.

Since renal production of calcitriol is PTH-dependent, in hypoparathyroidism the conversion of 25-hydroxyvitamin D3 to calcitriol is limited. Therefore, vitamin D3 (cholecalciferol) and vitamin D2 (ergocalciferol) are not the preferred forms of vitamin D for such patients. However, either can be added to calcitriol, as they may have extraskeletal benefits.7

CASE CONCLUDED

Our patient presented with primary parathyroid insufficiency associated with vitamin D deficiency. Therefore, in addition to calcitriol and calcium combined with vitamin D in a fixed-dose preparation, her management included vitamin D3 for her vitamin D deficiency.

She was discharged on these medications. At a follow-up visit 3 weeks later, her measured serum calcium level was 8.6 mg/dL. She reported gradual resolution of her symptoms. She was also referred to a psychiatrist for her depression.

TAKE-HOME POINTS

  • Hypocalcemia causes neuromuscular excitability, manifested clinically by tetany.
  • Common causes of hypocalcemia include vitamin D deficiency, hypomagnesemia, renal failure, and primary hypoparathyroidism.
  • The first step in evaluating hypocalcemia is to correct the measured serum calcium to the serum albumin concentration.
  • Laboratory testing for hypocalcemia should include serum phosphorus, magnesium, creatinine, PTH, and 25-hydroxyvitamin D3.
  • Primary hypoparathyroidism is characterized by hypocalcemia, hyperphosphatemia, and low serum PTH.
  • Moderate to severe manifestations of hypo-
    calcemia and acute hypocalcemia (< 7.6 mg/dL), even if asymptomatic, warrant intravenous calcium therapy.
  • Correction of hypomagnesemia is essential to treat hypocalcemia, especially if resistant to intravenous calcium therapy.
  • The goal of chronic management of primary hypoparathyroidism includes correcting the serum calcium level to a low-normal range, the serum phosphorus level to an upper-normal range, and prevention of hypercalciuria.

Acknowledgments: The authors wish to thank Mr. Michael Edward Tierney of the School of Medicine, University of Sydney, Australia, for his linguistic editing of the manuscript.

References
  1. Jesus JE, Landry A. Images in clinical medicine. Chvostek’s and Trousseau’s signs. N Engl J Med 2012; 367:e15.
  2. Urbano FL. Signs of hypocalcemia: Chvostek’s and Trousseau’s. Hosp Physician 2000; 36:43–45.
  3. Chisthi MM, Nair RS, Kuttanchettiyar KG, Yadev I. Mechanisms behind post-thyroidectomy hypocalcemia: interplay of calcitonin, parathormone, and albumin—a prospective study. J Invest Surg 2017; 30:217–225.
  4. Shoback DM, Bilezikian JP, Costa AG, et al. Presentation of hypoparathyroidism: etiologies and clinical features. J Clin Endocrinol Metab 2016; 101:2300–2312.
  5. Stack BC Jr, Bimston DN, Bodenner DL, et al. American Association of Clinical Endocrinologists and American College of Endocrinology disease state clinical review: postoperative hypoparathyroidism—definitions and management. Endocr Pract 2015; 21:674–685.
  6. Shoback D. Clinical practice. Hypoparathyroidism. N Engl J Med 2008; 359:391–403.
  7. Abate EG, Clarke BL. Review of hypoparathyroidism. Front Endocrinol (Lausanne) 2017; 7:172.
  8. Coimbra C, Monteiro F, Oliveira P, Ribeiro L, de Almeida MG, Condé A. Hypoparathyroidism following thyroidectomy: predictive factors. Acta Otorrinolaringol Esp 2017; 68:106–111.
  9. Thomusch O, Machens A, Sekulla C, Ukkat J, Brauckhoff M, Dralle H. The impact of surgical technique on postoperative hypoparathyroidism in bilateral thyroid surgery: a multivariate analysis of 5846 consecutive patients. Surgery 2003; 133:180–185.
  10. Hirsch PF, Lester GE, Talmage RV. Calcitonin, an enigmatic hormone: does it have a function? J Musculoskelet Neuronal Interact 2001; 1:299–305.
  11. Karne SS, Bhalerao NS. Carpal tunnel syndrome in hypothyroidism. J Clin Diagn Res 2016; 10:OC36–OC38.
  12. Duyff RF, Van den Bosch J, Laman DM, van Loon BJ, Linssen WH. Neuromuscular findings in thyroid dysfunction: a prospective clinical and electrodiagnostic study. J Neurol Neurosurg Psychiatry 2000; 68:750–755.
  13. Palumbo CF, Szabo RM, Olmsted SL. The effects of hypothyroidism and thyroid replacement on the development of carpal tunnel syndrome. J Hand Surg Am 2000; 25:734–739.
  14. Katz JN, Stirrat CR, Larson MG, Fossel AH, Eaton HM, Liang MH. A self-administered hand symptom diagram for the diagnosis and epidemiologic study of carpal tunnel syndrome. J Rheumatol 1990; 17:1495–1498.
  15. Katz JN, Stirrat CR. A self-administered hand diagram for the diagnosis of carpal tunnel syndrome. J Hand Surg Am 1990; 15:360–363.
  16. Calfee RP, Dale AM, Ryan D, Descatha A, Franzblau A, Evanoff B. Performance of simplified scoring systems for hand diagrams in carpal tunnel syndrome screening. J Hand Surg Am 2012; 37:10–17.
  17. D’Arcy CA, McGee S. The rational clinical examination. Does this patient have carpal tunnel syndrome? JAMA 2000; 283:3110–3117.
  18. Marchettini P, Lacerenza M, Mauri E, Marangoni C. Painful peripheral neuropathies. Curr Neuropharmacol 2006; 4:175–181.
  19. Kibirige D, Mwebaze R. Vitamin B12 deficiency among patients with diabetes mellitus: is routine screening and supplementation justified? J Diabetes Metab Disord 2013;12:17.
  20. Akinlade KS, Agbebaku SO, Rahamon SK, Balogun WO. Vitamin B12 levels in patients with type 2 diabetes mellitus on metformin. Ann Ib Postgrad Med 2015; 13:79–83.
  21. Tohme JF, Bilezikian JP. Hypocalcemic emergencies. Endocrinol Metab Clin North Am 1993; 22:363–375.
  22. Macefield G, Burke D. Paraesthesiae and tetany induced by voluntary hyperventilation. Increased excitability of human cutaneous and motor axons. Brain 1991; 114:527–540.
  23. Benoit SR, Mendelsohn AB, Nourjah P, Staffa JA, Graham DJ. Risk factors for prolonged QTc among US adults: Third National Health and Nutrition Examination Survey. Eur J Cardiovasc Prev Rehabil 2005; 12:363–368.
  24. Khoo TK, Vege SS, Abu-Lebdeh HS, Ryu E, Nadeem S, Wermers RA. Acute pancreatitis in primary hyperparathyroidism: a population-based study. J Clin Endocrinol Metab 2009; 94:2115–2118.
  25. McKay C, Beastall GH, Imrie CW, Baxter JN. Circulating intact parathyroid hormone levels in acute pancreatitis. Br J Surg 1994; 81:357–360.
  26. Talmage RV, Mobley HT. Calcium homeostasis: reassessment of the actions of parathyroid hormone. Gen Comp Endocrinol 2008; 156:1–8.
  27. Friedman PA, Gesek FA. Calcium transport in renal epithelial cells. Am J Physiol 1993; 264:F181–F198.
  28. Jensen PV, Jelstrup SM, Homøe P. Long-term outcomes after total thyroidectomy. Dan Med J 2015; 62:A5156.
  29. Ritter K, Elfenbein D, Schneider DF, Chen H, Sippel RS. Hypoparathyroidism after total thyroidectomy: incidence and resolution. J Surg Res 2015; 197:348–353.
  30. Promberger R, Ott J, Kober F, Karik M, Freissmuth M, Hermann M. Normal parathyroid hormone levels do not exclude permanent hypoparathyroidism after thyroidectomy. Thyroid 2011; 21:145–150.
  31. Lorente-Poch L, Sancho JJ, Muñoz-Nova JL, Sánchez-Velázquez P, Sitges-Serra A. Defining the syndromes of parathyroid failure after total thyroidectomy. Gland Surgery 2015; 4:82–90.
  32. Cooper MS, Gittoes NJ. Diagnosis and management of hypocalcaemia. BMJ 2008; 336:1298–1302.
  33. Tohme JF, Bilezikian JP. Diagnosis and treatment of hypocalcemic emergencies. Endocrinologist 1996; 6:10–18.
  34. Carroll R, Matfin G. Endocrine and metabolic emergencies: hypocalcaemia. Ther Adv Endocrinol Metab 2010; 1:29–33.
  35. Kim MP, Raho VJ, Mak J, Kaynar AM. Skin and soft tissue necrosis from calcium chloride in a deicer. J Emerg Med 2007; 32:41–44.
  36. Tong GM, Rude RK. Magnesium deficiency in critical illness. J Intensive Care Med 2005; 20:3–17.
  37. Cholst IN, Steinberg SF, Tropper PJ, Fox HE, Segre GV, Bilezikian JP. The influence of hypermagnesemia on serum calcium and parathyroid hormone levels in human subjects. N Engl J Med 1984; 310:1221–1225.
  38. Ryzen E, Nelson TA, Rude RK. Low blood mononuclear cell magnesium content and hypocalcemia in normomagnesemic patients. West J Med 1987; 147:549–553.
  39. Koontz SL, Friedman SA, Schwartz ML. Symptomatic hypocalcemia after tocolytic therapy with magnesium sulfate and nifedipine. Am J Obstet Gynecol 2004; 190:1773–1776.
  40. Brandi ML, Bilezikian JP, Shoback D, et al. Management of hypoparathyroidism: summary statement and guidelines. J Clin Endocrinol Metab 2016; 101:2273–2283.
  41. Porter RH, Cox BG, Heaney D, Hostetter TH, Stinebaugh BJ, Suki WN. Treatment of hypoparathyroid patients with chlorthalidone. N Engl J Med 1978; 298:577–581.
  42. Clarke BL, Brown EM, Collins MT, et al. Epidemiology and diagnosis of hypoparathyroidism. J Clin Endocrinol Metab 2016; 101:2284–2299.
  43. Nijenhuis T, Vallon V, van der Kemp AW, Loffing J, Hoenderop JG, Bindels RJ. Enhanced passive Ca2+ reabsorption and reduced Mg2+ channel abundance explains thiazide-induced hypocalciuria and hypomagnesemia. J Clin Invest 2005; 115:1651–1658.
  44. Costanzo LS. Localization of diuretic action in microperfused rat distal tubules: Ca and Na transport. Am J Physiol 1985; 248:F527–F535.
  45. Brandi ML, Bilezikian JP, Shoback D, et al. Management of hypoparathyroidism: summary statement and guidelines. J Clin Endocrinol Metab 2016; 101:2273–2283.
  46. Scotti A, Bianchini C, Abbiati G, Marzo A. Absorption of calcium administered alone or in fixed combination with vitamin D to healthy volunteers. Arzneimittelforschung 2001; 51:493–500.
Article PDF
Author and Disclosure Information

Sohab S. Radwan, MD
Division of Endocrinology, Diabetes, and Metabolism, Department of Internal Medicine, Jordan University Hospital, Amman, Jordan

Khair M. Hamo
School of Medicine, Jordan University Hospital, Amman, Jordan

Ayman A. Zayed, MD, MSc, FACE, FACP
Chief, Division of Endocrinology, Diabetes, and Metabolism Department of Internal Medicine, Jordan University Hospital, Amman, Jordan

Address: Ayman A. Zayed, MD, MSc, FACE, FACP, Division of Endocrinology, Diabetes, and Metabolism, Department of Internal Medicine, Jordan University Hospital, The University of Jordan, Queen Rania Street, Amman, Jordan, 11942; baraaayman@gmail.com

Issue
Cleveland Clinic Journal of Medicine - 85(3)
Publications
Topics
Page Number
200-208
Legacy Keywords
numbness, paresthesia, hypocalcemia, calcium, Trousseau sign, Chvostek sign, hypoparathyroidism, carpal tunnel syndrome, hypothyroidism, diabetic peripheral neuropathy, vitamin B12 deficiency, parathyroid, parathyroid hormone, PTH, thyroid surgery, electrolytes, Sohab Radwan, Khair Hamo, Ayman Zayed
Sections
Author and Disclosure Information

Sohab S. Radwan, MD
Division of Endocrinology, Diabetes, and Metabolism, Department of Internal Medicine, Jordan University Hospital, Amman, Jordan

Khair M. Hamo
School of Medicine, Jordan University Hospital, Amman, Jordan

Ayman A. Zayed, MD, MSc, FACE, FACP
Chief, Division of Endocrinology, Diabetes, and Metabolism Department of Internal Medicine, Jordan University Hospital, Amman, Jordan

Address: Ayman A. Zayed, MD, MSc, FACE, FACP, Division of Endocrinology, Diabetes, and Metabolism, Department of Internal Medicine, Jordan University Hospital, The University of Jordan, Queen Rania Street, Amman, Jordan, 11942; baraaayman@gmail.com

Author and Disclosure Information

Sohab S. Radwan, MD
Division of Endocrinology, Diabetes, and Metabolism, Department of Internal Medicine, Jordan University Hospital, Amman, Jordan

Khair M. Hamo
School of Medicine, Jordan University Hospital, Amman, Jordan

Ayman A. Zayed, MD, MSc, FACE, FACP
Chief, Division of Endocrinology, Diabetes, and Metabolism Department of Internal Medicine, Jordan University Hospital, Amman, Jordan

Address: Ayman A. Zayed, MD, MSc, FACE, FACP, Division of Endocrinology, Diabetes, and Metabolism, Department of Internal Medicine, Jordan University Hospital, The University of Jordan, Queen Rania Street, Amman, Jordan, 11942; baraaayman@gmail.com

Article PDF
Article PDF
Related Articles

A 67-year-old woman presents to the emergency department after 8 weeks of progressive numbness and tingling in both hands, involving all fingers. The numbness has increased in severity in the last 3 days. She also has occasional numbness around her mouth. She reports no numbness in her feet.

She says she underwent thyroid surgery twice for thyroid cancer 10 years ago. Her medical history also includes type 2 diabetes mellitus (diagnosed 1 year ago), hypertension, dyslipidemia, and diastolic heart failure (diagnosed 5 years ago).

Her current medications are:

  • Metformin 1 g twice a day
  • Candesartan 16 mg once a day
  • Atorvastatin 20 mg once a day
  • Furosemide 40 mg twice a day
  • Levothyroxine 100 μg per day
  • Calcium carbonate 1,500 mg twice a day
  • A vitamin D tablet twice a day, which she has not taken for the last 2 months.

She admits she has not been taking her medications regularly because she has been feeling depressed.

On physical examination, she is alert and oriented but appears anxious. She is not in res­piratory distress. Her blood pressure is 150/90 mm Hg and her pulse is 92 beats per minute and regular. There is a thyroidectomy scar on the anterior neck. Her jugular venous pressure is not elevated. Her heart sounds are normal without extra sounds. She has no pulmonary rales and no lower-extremity edema.

The Phalen test and Tinel test for carpal tunnel syndrome are negative in both hands. Using a Katz hand diagram, the patient reports tingling and numbness in all fingers, both palms, and the dorsum of both hands. Tapping the area over the facial nerve does not elicit twitching of the facial muscles (ie, no Chvostek sign), but compression of the upper arm elicits carpal spasm (ie, positive Trousseau sign). There is no evidence of motor weakness in her hands. The rest of the physical examination is unremarkable.

POSSIBLE CAUSES OF NUMBNESS

1. Based on the initial evaluation, which of the following is the most likely cause of our patient’s bilateral hand numbness?

  • Hypocalcemia due to primary hypoparathyroidism
  • Carpal tunnel syndrome due to primary hypothyroidism
  • Diabetic peripheral neuropathy
  • Vitamin B12 deficiency due to metformin
  • Hypocalcemia due to low serum calcitonin

All the conditions above except low serum calcitonin can cause bilateral hand paresthesia. Our patient most likely has hypocalcemia due to primary hypoparathyroidism.

Hypocalcemia

In our patient, bilateral hand numbness and perioral numbness after stopping vitamin D and a positive Trousseau sign strongly suggest hypocalcemia. The classic physical findings in patients with hypocalcemia are the Trousseau sign and the Chvostek sign. The Trousseau sign is elicited by inflating a blood pressure cuff above the systolic blood pressure for 3 minutes and observing for ischemia-induced carpopedal spasm, wrist and metacarpophalangeal joint flexion, thumb adduction, and interphalangeal joint extension. The Chvostek sign is elicited by tapping over the area of the facial nerve below the zygoma in front of the tragus, resulting in ipsilateral twitching of facial muscles.

Although the Trousseau sign is more sensitive and specific than the Chvostek sign, neither is pathognomonic for hypocalcemia.1 The Chvostek sign has been reported to be negative in 30% of patients with hypocalcemia and positive in 10% of normocalcemic individuals.1 The Trousseau sign, however, is present in 94% of hypocalcemic patients vs 1% of normocalcemic individuals.2

Primary hypoparathyroidism secondary to thyroidectomy. Postsurgical hypoparathyroidism is the most common cause of primary hypoparathyroidism. It results from ischemic injury or accidental removal of the parathyroid glands during anterior neck surgery.3,4 The consequent hypocalcemia can be transient, intermittent, or permanent. Permanent postsurgical hypoparathyroidism is defined as persistent hypocalcemia with insufficient parathyroid hormone (PTH) for more than 12 months after neck surgery; however, some consider 6 months to be enough to define the condition.5–7

The incidence of postsurgical hypoparathyroidism varies considerably with the extent of thyroid surgery and the experience of the surgeon.6,8 In the hands of experienced surgeons, permanent hypoparathyroidism occurs in fewer than 1% of patients after total thyroidectomy, whereas the rate may be higher than 6% with less-experienced surgeons.5,9 Other risk factors for postsurgical hypoparathyroidism include female sex, autoimmune thyroid disease, pregnancy, and lactation.5

Pseudohypoparathyroidism is a group of disorders characterized by renal resistance to PTH, leading to hypocalcemia, hyperphosphatemia, and elevated serum PTH. It is also associated with phenotypic features such as short stature and short fourth metacarpal bones.

Calcitonin deficiency. Calcitonin is a polypeptide hormone secreted from the parafollicular (C) cells of the thyroid gland. After total thyroidectomy, calcitonin levels are expected to be reduced. However, the role of calcitonin in humans is unclear. One study has shown that calcitonin is possibly a vestigial hormone, given that no calcitonin-related disorders (excess or deficiency) have been reported in humans.10

 

 

Carpal tunnel syndrome due to hypothyroidism

Our patient also could have primary hypothyroidism as a result of thyroidectomy. Hypothyroidism can cause bilateral hand numbness due to carpal tunnel syndrome, which is mediated by mucopolysaccharide deposition and synovial membrane swelling.11 One study reported that 29% of patients with hypothyroidism had carpal tunnel syndrome.12 Symptoms of carpal tunnel syndrome in hypothyroid patients may occur despite thyroid replacement therapy.13

Katz hand diagram classification of carpal tunnel syndrome

Carpal tunnel syndrome is a clinical diagnosis. Patients usually experience hand paresthesia in the distribution of the median nerve. Provocative physical tests for carpal tunnel syndrome include the Tinel test, the Phalen test, and the Katz hand diagram, which is considered the best of the 3 tests.14,15 Based on how the patient marks the location and type of symptoms on the diagram, carpal tunnel syndrome is rated as classic, probable, possible, or unlikely (Table 1).14,16,17 The sensitivity of a classic or probable diagram ranges from 64% to 80%, while the specificity ranges from 73% to 90%.14,15

Carpal tunnel syndrome is less likely to be the cause of our patient’s symptoms, as her Katz hand diagram indicates only “possible” carpal tunnel syndrome. Her perioral numbness and positive Trousseau sign make hypocalcemia a more likely cause.

Diabetic peripheral neuropathy

Sensory peripheral neuropathy is a recognized complication of diabetes mellitus. However, neuropathy in diabetic patients most commonly manifests initially as distal symmetrical ascending neuropathy starting in the lower extremities.18 Therefore, diabetic peripheral neuropathy is less likely in this patient since her symptoms are limited to her hands.

Vitamin B12 deficiency

Metformin-induced vitamin B12 deficiency is another possible cause of peripheral neuropathy. It might be secondary to metformin-induced changes in intrinsic factor levels and small-intestine motility with resultant bacterial overgrowth, as well as inhibition of vitamin B12 absorption in the terminal ileum.19

However, metformin-induced vitamin B12 deficiency is not the most likely cause of our patient’s neuropathy, since she has been taking this drug for only 1 year. Vitamin B12 deficiency with consequent peripheral neuropathy is more likely in patients taking metformin in high doses for 10 or more years.20

Laboratory results and electrocardiography

Results of initial laboratory testing

Table 2 shows the patient’s initial laboratory results. Of note, her serum calcium level is 5.7 mg/dL (reference range 8.9–10.1). Electrocardiography in the emergency department shows:

  • Prolonged PR interval (23 msec)
  • Wide QRS complexes (13 msec)
  • Flat T waves
  • Prolonged corrected QT interval (475 msec)
  • Occasional premature ventricular complexes.

CLINICAL MANIFESTATIONS OF HYPOCALCEMIA

2. Which of the following is not a manifestation of hypocalcemia?

  • Tonic-clonic seizures
  • Cyanosis
  • Cardiac ventricular arrhythmias
  • Acute pancreatitis
  • Depression

 Clinical manifestations of hypocalcemia

Hypocalcemia can cause a wide range of clinical manifestations (Table 3), the extent and severity of which depend on the severity of hypocalcemia and how quickly it develops. The more acute the hypocalcemia, the more severe the manifestations.21

Tetany can cause seizures

Hypocalcemia is characterized by neuromuscular hyperexcitability, manifested clinically by tetany.22 Manifestations of tetany are numerous and include acral paresthesia, perioral numbness, muscle cramps, carpopedal spasm, and seizures. Tetany is the hallmark of hypocalcemia regardless of etiology. However, certain causes are associated with peculiar clinical manifestations. For example, chronic primary hypoparathyroidism may be associated with basal ganglia calcifications that can result in parkinsonism, other extrapyramidal disorders, and dementia (Table 4).6

Clinical manifestations of chronic primary hypoparathyroidism

Airway spasm can be fatal

A serious manifestation of acute severe hypocalcemia is spasm of the glottis muscles, which may cause cyanosis and, if untreated, death.21

Ventricular arrhythmias

Another potential fatal complication of acute severe hypocalcemia is polymorphic ventricular tachycardia due to prolongation of the QT interval, which is readily identified with electrocardiography.23

Hypocalcemia does not cause pancreatitis

Hypercalcemia, rather than hypocalcemia, may cause acute pancreatitis.24 Conversely, acute pancreatitis may cause hypocalcemia due to precipitation of calcium in the abdominal cavity.25

Psychiatric manifestations

In addition to depression, hypocalcemia is associated with psychiatric manifestations including anxiety, confusion, and emotional instability.

 

 

STEPS TO DIAGNOSIS OF HYPOCALCEMIA

First step: Confirm true hypocalcemia

Calcium circulates in the blood in 3 forms: bound to albumin (40% to 45%), bound to anions (10% to 15%), and free (ionized) (45%). Although ionized calcium is the active form, most laboratories report total serum calcium.

Since changes in serum albumin concentration affect the total serum calcium level, it is imperative to correct the measured serum calcium to the serum albumin concentration. Each 1-g/dL decrease in serum albumin lowers the total serum calcium by 0.8 mg/dL. Thus:

Corrected serum calcium (mg/dL) =
measured total serum calcium (mg/dL) +
0.8 (4 − serum albumin [g/dL])
.

If the patient’s serum calcium level remains low when corrected for serum albumin, he or she has true hypocalcemia, which implies a low ionized serum calcium. Conversely, pseudohypocalcemia means that the measured calcium level is low but the corrected serum calcium is normal.

Using this formula, our patient’s corrected calcium level is calculated as 5.7 + 0.8 (4 – 3.2) = 6.3 mg/dL, indicating true hypocalcemia.

PHOSPHATE IS OFTEN HIGH WHEN CALCIUM IS LOW

In addition to hypocalcemia, our patient has an elevated phosphate level (Table 2).

3. Which of the following hypocalcemic disorders is not associated with hyperphosphatemia?

  • End-stage renal disease
  • Primary hypoparathyroidism
  • Pseudohypoparathyroidism
  • Vitamin D3 deficiency
  • Rhabdomyolysis

Vitamin D deficiency is not associated with hyperphosphatemia.

Second step in evaluating hypocalcemia: Check phosphate, magnesium, creatinine

Major causes of hypocalcemia, according to phosphate level

The major causes of hypocalcemia can be categorized according to the serum phosphate level: high vs normal or low (Table 5).

High-phosphate, low-calcium states. In the absence of concurrent end-stage renal disease and an excessive phosphate load, primary hypoparathyroidism is the most likely cause of hypocalcemia associated with hyperphosphatemia.

PTH increases serum ionized calcium by26,27:

  • Increasing bone resorption
  • Increasing reabsorption of calcium from the distal renal tubules
  • Increasing the activity of 1-alpha-hydroxylase, responsible for conversion of 25-hydroxyvitamin D3 to 1,25-dihydroxyvitamin D3 (the most biologically active vitamin D metabolite); 1,25-dihydroxyvitamin D increases the absorption of calcium and phosphate from the intestine.

Conversely, PTH decreases reabsorption of phosphate from proximal renal tubules, resulting in hypophosphatemia. Therefore, low serum PTH (primary hypoparathyroidism) or a PTH-resistant state (pseudohypoparathyroidism) results in hypocalcemia and hyperphosphatemia.26,27

Both end-stage renal disease and rhabdomyolysis are associated with high serum phosphate levels. The kidney normally excretes excess dietary phosphate to maintain phosphate homeostasis; however, this is impaired in end-stage renal disease, leading to hyperphosphatemia. In rhabdomyolysis, it is mainly the transcellular shift of phosphate into the extracellular space from myocyte injury that raises phosphate levels.

Normal- or low-phosphate, low calcium states. Hypocalcemia can also result from vitamin D deficiency, but this cause is associated with a low or normal serum phosphate level. In such cases, hypocalcemia causes secondary hyperparathyroidism with consequent renal phosphate loss and, thus, hypophosphatemia.27

Third step: Check serum intact PTH and 25-hydroxyvitamin D levels

Hypocalcemia stimulates secretion of PTH. Therefore, hypocalcemia with elevated serum PTH is caused by disorders that do not impair PTH secretion, including chronic renal failure and vitamin D deficiency (Table 5). Conversely, hypocalcemia with low or normal serum PTH levels suggests primary hypoparathyroidism.

Our patient’s serum PTH level is 20 ng/mL, which is within the reference range. This does not discount the diagnosis of primary hypoparathyroidism. Although most patients with primary hypoparathyroidism have low or undetectable serum PTH levels, some have normal PTH levels if some degree of PTH production is preserved.5,7,28–30 In these patients, the remaining functioning parathyroid tissue is not enough to maintain a normal serum calcium level, resulting in hypocalcemia. As a result, hypocalcemia stimulates the remaining parathyroid tissue to its maximum output, producing PTH levels usually within the lower or middle-normal range.30 In such patients, the terms parathyroid insufficiency and relative primary hypoparathyroidism are more precise than primary hypoparathyroidism.

Postsurgical hypoparathyroidism with an inappropriately normal PTH level is usually seen in patients with disorders that impair intestinal calcium absorption or bone resorption.31 In our patient’s case, the “normal” serum PTH level is likely due to maximal stimulation of remaining functioning parathyroid tissue by severe hypocalcemia, which is a result of her discontinuation of calcium and calcitriol therapy and her vitamin D deficiency.

 

 

CASE RESUMED: NO RESPONSE TO INTRAVENOUS CALCIUM GLUCONATE

The patient is given 2 10-mL ampules of 10% calcium gluconate diluted in 100 mL of 5% dextrose in water over 20 minutes intravenously. Electrocardiographic monitoring is continued. Two hours later, her measured serum calcium is only 5.8 mg/dL, with no improvement in her symptoms.

A continuous infusion of calcium gluconate is started: 12 ampules of calcium gluconate are added to 380 mL of 5% dextrose in water and infused at 40 mL/hour (infused rate of elemental calcium = 1.3 mg/kg/hour); 3 hours later, her measured serum calcium level is still only 5.8 mg//dL; at 6 hours it is 5.9 mg/dL, and her symptoms have not improved.

4. Which of the following is the most appropriate next step?

  • Change the calcium gluconate to calcium chloride
  • Increase the infusion rate to 1.5 mg of elemental calcium/kg/hour
  • Give a bolus of 2 10-mL ampules of 10% calcium gluconate intravenously over 1 minute
  • Give additional oral calcium tablets
  • Check the serum magnesium level

Treatment of hypocalcemia can involve intravenous or oral calcium therapy.

Intravenous calcium is indicated for patients with any of the following6,32:

  • Moderate to severe neuromuscular irritability (eg, acral paresthesia, carpopedal spasm, prolonged QT interval, seizures, laryngospasm, bronchospasm)
  • Acute hypocalcemia with corrected serum calcium level less than 7.6 mg/dL, even if the patient is asymptomatic
  • Cardiac failure.

One 10-mL ampule of 10% calcium gluconate contains 93 mg of elemental calcium; 1 or 2 ampules are typically diluted in 50 to 100 mL of 5% dextrose in water and infused slowly over 15 to 20 minutes. Rapid administration of intravenous calcium is contraindicated, as it may produce cardiac arrhythmias and possibly cardiac arrest. Therefore, intravenous calcium should be given slowly while continuing electrocardiographic monitoring.33

Since the effect of 1 ampule of calcium gluconate lasts only 2 to 3 hours, most patients with symptomatic hypocalcemia require continuous intravenous calcium infusion. The recommended dose of infused elemental calcium is 0.5 to 1.5 mg/kg/hour.34 Several ampules are added to 500 to 1,000 mL of 5% dextrose in water or 0.9% normal saline and infused at a rate appropriate for the patient’s corrected calcium and symptoms.

Oral calcium and vitamin D supplements can be given initially to patients with a corrected serum calcium level of 7.6 mg/dL or greater, with or without mild symptoms, if they can tolerate oral intake. However, this is not the treatment of choice for resistant acute hypocalcemia, as in this case.

Calcium chloride has no advantages over calcium gluconate. Further, it can be associated with local irritation and may result in tissue necrosis if extravasation occurs.35

Increasing the infusion rate of calcium gluconate to the maximum recommended dose may improve the patient’s ionized calcium level and symptoms somewhat. However, it is not the best option for this patient, given that she did not respond to 2 ampules of calcium gluconate followed by continuous infusion of 1.3 mg/kg/hour for 6 hours.

Calcium gluconate bolus. Similarly, giving the patient an additional 2 ampules of calcium gluconate over 1 minute would not be recommended, as rapid administration of intravenous calcium gluconate (eg, over 1 minute) is contraindicated.

Check magnesium

If hypocalcemia persists despite intravenous calcium therapy, as in our patient, further investigation or action is required. An important cause of persistent hypocalcemia is severe hypomagnesemia. Severe hypomagnesemia (serum magnesium < 0.8 mg/dL) causes resistant hypocalcemia by several mechanisms:

  • Inducing PTH resistance32,36,37
  • Decreasing PTH secretion32,36
  • Decreasing calcitriol production.

The decrease in calcitriol production is a direct effect of hypomagnesemia, but it is also an indirect effect of low PTH secretion, which inhibits the enzyme 1-alpha-hydroxylase. Thus, conversion of 25-hydroxyvitamin D3 to calcitriol is impaired, leading to low calcitriol production.

Our patient could have hypomagnesemia due to furosemide use and uncontrolled diabetes mellitus. Hypocalcemia resistant to calcium therapy may occasionally respond to magnesium therapy even if the serum magnesium level is normal. This may be due to depleted intracellular magnesium salt levels.6,38 Rarely, severe hypermagnesemia can also be associated with hypocalcemia due to inhibition of PTH secretion.37,39

CASE RESUMED

Our patient’s serum magnesium level is 0.6 mg/dL (reference range 1.7–2.4 mg/dL). She is given 2 g of magnesium sulfate in 60 mL of 0.9% normal saline infused over 1 hour, followed by a continuous infusion of magnesium sulfate (12 g diluted in 250 mL of 0.9% normal saline, infused over 24 hours). On repeat testing 4 hours later, her serum magnesium level is 0.7 mg/dL, and at 8 hours later it is 0.9 mg/dL. She is subsequently started on oral magnesium oxide 600 mg per day. The magnesium sulfate infusion is continued for another 24 hours.

PREVENTING HYPERCALCIURIA

Patients with low PTH (primary hypoparathyroidism) may have hypercalciuria due to decreased renal tubular calcium reabsorption. Two important measures can minimize hypercalciuria in such patients:

  • Keeping the serum calcium level in the low-normal range4,5,40
  • Giving a thiazide diuretic (eg, hydrochlorothiazide 12.5–50 mg daily) with a low-salt diet.41,42

A thiazide diuretic is usually started once the 24-hour urine calcium reaches 250 mg.6 Thiazides are thought to enhance both proximal and distal renal tubular calcium reabsorption.43,44

PRIMARY HYPOPARATHYROIDISM: LONG-TERM MANAGEMENT

Long-term management of primary hypoparathyroidism requires calcium and vitamin D supplementation.

Calcium supplements. The most commonly prescribed calcium preparations are calcium carbonate and calcium citrate (containing 40% and 20% elemental calcium, respectively). Calcium carbonate, which is less expensive than calcium citrate, binds with phosphate intake and requires an acidic environment for absorption, and so it is better absorbed when taken with meals. Because calcium citrate does not require an acidic environment for absorption, it is the calcium preparation of choice for patients on proton pump inhibitors, or patients with achlorhydria or constipation.45 Calcium doses vary widely, with most hypoparathyroid patients requiring 1 to 2 g of elemental calcium daily.6

Vitamin D supplements. To promote intestinal absorption, calcium is combined with vitamin D in a fixed-dose preparation given in divided doses.46 Calcitriol (1,25-dihydroxyvitamin D3) is the most active metabolite of vitamin D, with rapid onset and offset of action, and it is the preferred form of vitamin D therapy for patients with hypoparathyroidism. If calcitriol is not available or is not affordable, alphacalcidol (1-alpha-hydroxyvitamin D3) is another option. This is a synthetic analogue of vitamin D that is already hyroxylated at the C1 position. After oral intake, it is hydroxylated in the liver to form calcitriol.

Since renal production of calcitriol is PTH-dependent, in hypoparathyroidism the conversion of 25-hydroxyvitamin D3 to calcitriol is limited. Therefore, vitamin D3 (cholecalciferol) and vitamin D2 (ergocalciferol) are not the preferred forms of vitamin D for such patients. However, either can be added to calcitriol, as they may have extraskeletal benefits.7

CASE CONCLUDED

Our patient presented with primary parathyroid insufficiency associated with vitamin D deficiency. Therefore, in addition to calcitriol and calcium combined with vitamin D in a fixed-dose preparation, her management included vitamin D3 for her vitamin D deficiency.

She was discharged on these medications. At a follow-up visit 3 weeks later, her measured serum calcium level was 8.6 mg/dL. She reported gradual resolution of her symptoms. She was also referred to a psychiatrist for her depression.

TAKE-HOME POINTS

  • Hypocalcemia causes neuromuscular excitability, manifested clinically by tetany.
  • Common causes of hypocalcemia include vitamin D deficiency, hypomagnesemia, renal failure, and primary hypoparathyroidism.
  • The first step in evaluating hypocalcemia is to correct the measured serum calcium to the serum albumin concentration.
  • Laboratory testing for hypocalcemia should include serum phosphorus, magnesium, creatinine, PTH, and 25-hydroxyvitamin D3.
  • Primary hypoparathyroidism is characterized by hypocalcemia, hyperphosphatemia, and low serum PTH.
  • Moderate to severe manifestations of hypo-
    calcemia and acute hypocalcemia (< 7.6 mg/dL), even if asymptomatic, warrant intravenous calcium therapy.
  • Correction of hypomagnesemia is essential to treat hypocalcemia, especially if resistant to intravenous calcium therapy.
  • The goal of chronic management of primary hypoparathyroidism includes correcting the serum calcium level to a low-normal range, the serum phosphorus level to an upper-normal range, and prevention of hypercalciuria.

Acknowledgments: The authors wish to thank Mr. Michael Edward Tierney of the School of Medicine, University of Sydney, Australia, for his linguistic editing of the manuscript.

A 67-year-old woman presents to the emergency department after 8 weeks of progressive numbness and tingling in both hands, involving all fingers. The numbness has increased in severity in the last 3 days. She also has occasional numbness around her mouth. She reports no numbness in her feet.

She says she underwent thyroid surgery twice for thyroid cancer 10 years ago. Her medical history also includes type 2 diabetes mellitus (diagnosed 1 year ago), hypertension, dyslipidemia, and diastolic heart failure (diagnosed 5 years ago).

Her current medications are:

  • Metformin 1 g twice a day
  • Candesartan 16 mg once a day
  • Atorvastatin 20 mg once a day
  • Furosemide 40 mg twice a day
  • Levothyroxine 100 μg per day
  • Calcium carbonate 1,500 mg twice a day
  • A vitamin D tablet twice a day, which she has not taken for the last 2 months.

She admits she has not been taking her medications regularly because she has been feeling depressed.

On physical examination, she is alert and oriented but appears anxious. She is not in res­piratory distress. Her blood pressure is 150/90 mm Hg and her pulse is 92 beats per minute and regular. There is a thyroidectomy scar on the anterior neck. Her jugular venous pressure is not elevated. Her heart sounds are normal without extra sounds. She has no pulmonary rales and no lower-extremity edema.

The Phalen test and Tinel test for carpal tunnel syndrome are negative in both hands. Using a Katz hand diagram, the patient reports tingling and numbness in all fingers, both palms, and the dorsum of both hands. Tapping the area over the facial nerve does not elicit twitching of the facial muscles (ie, no Chvostek sign), but compression of the upper arm elicits carpal spasm (ie, positive Trousseau sign). There is no evidence of motor weakness in her hands. The rest of the physical examination is unremarkable.

POSSIBLE CAUSES OF NUMBNESS

1. Based on the initial evaluation, which of the following is the most likely cause of our patient’s bilateral hand numbness?

  • Hypocalcemia due to primary hypoparathyroidism
  • Carpal tunnel syndrome due to primary hypothyroidism
  • Diabetic peripheral neuropathy
  • Vitamin B12 deficiency due to metformin
  • Hypocalcemia due to low serum calcitonin

All the conditions above except low serum calcitonin can cause bilateral hand paresthesia. Our patient most likely has hypocalcemia due to primary hypoparathyroidism.

Hypocalcemia

In our patient, bilateral hand numbness and perioral numbness after stopping vitamin D and a positive Trousseau sign strongly suggest hypocalcemia. The classic physical findings in patients with hypocalcemia are the Trousseau sign and the Chvostek sign. The Trousseau sign is elicited by inflating a blood pressure cuff above the systolic blood pressure for 3 minutes and observing for ischemia-induced carpopedal spasm, wrist and metacarpophalangeal joint flexion, thumb adduction, and interphalangeal joint extension. The Chvostek sign is elicited by tapping over the area of the facial nerve below the zygoma in front of the tragus, resulting in ipsilateral twitching of facial muscles.

Although the Trousseau sign is more sensitive and specific than the Chvostek sign, neither is pathognomonic for hypocalcemia.1 The Chvostek sign has been reported to be negative in 30% of patients with hypocalcemia and positive in 10% of normocalcemic individuals.1 The Trousseau sign, however, is present in 94% of hypocalcemic patients vs 1% of normocalcemic individuals.2

Primary hypoparathyroidism secondary to thyroidectomy. Postsurgical hypoparathyroidism is the most common cause of primary hypoparathyroidism. It results from ischemic injury or accidental removal of the parathyroid glands during anterior neck surgery.3,4 The consequent hypocalcemia can be transient, intermittent, or permanent. Permanent postsurgical hypoparathyroidism is defined as persistent hypocalcemia with insufficient parathyroid hormone (PTH) for more than 12 months after neck surgery; however, some consider 6 months to be enough to define the condition.5–7

The incidence of postsurgical hypoparathyroidism varies considerably with the extent of thyroid surgery and the experience of the surgeon.6,8 In the hands of experienced surgeons, permanent hypoparathyroidism occurs in fewer than 1% of patients after total thyroidectomy, whereas the rate may be higher than 6% with less-experienced surgeons.5,9 Other risk factors for postsurgical hypoparathyroidism include female sex, autoimmune thyroid disease, pregnancy, and lactation.5

Pseudohypoparathyroidism is a group of disorders characterized by renal resistance to PTH, leading to hypocalcemia, hyperphosphatemia, and elevated serum PTH. It is also associated with phenotypic features such as short stature and short fourth metacarpal bones.

Calcitonin deficiency. Calcitonin is a polypeptide hormone secreted from the parafollicular (C) cells of the thyroid gland. After total thyroidectomy, calcitonin levels are expected to be reduced. However, the role of calcitonin in humans is unclear. One study has shown that calcitonin is possibly a vestigial hormone, given that no calcitonin-related disorders (excess or deficiency) have been reported in humans.10

 

 

Carpal tunnel syndrome due to hypothyroidism

Our patient also could have primary hypothyroidism as a result of thyroidectomy. Hypothyroidism can cause bilateral hand numbness due to carpal tunnel syndrome, which is mediated by mucopolysaccharide deposition and synovial membrane swelling.11 One study reported that 29% of patients with hypothyroidism had carpal tunnel syndrome.12 Symptoms of carpal tunnel syndrome in hypothyroid patients may occur despite thyroid replacement therapy.13

Katz hand diagram classification of carpal tunnel syndrome

Carpal tunnel syndrome is a clinical diagnosis. Patients usually experience hand paresthesia in the distribution of the median nerve. Provocative physical tests for carpal tunnel syndrome include the Tinel test, the Phalen test, and the Katz hand diagram, which is considered the best of the 3 tests.14,15 Based on how the patient marks the location and type of symptoms on the diagram, carpal tunnel syndrome is rated as classic, probable, possible, or unlikely (Table 1).14,16,17 The sensitivity of a classic or probable diagram ranges from 64% to 80%, while the specificity ranges from 73% to 90%.14,15

Carpal tunnel syndrome is less likely to be the cause of our patient’s symptoms, as her Katz hand diagram indicates only “possible” carpal tunnel syndrome. Her perioral numbness and positive Trousseau sign make hypocalcemia a more likely cause.

Diabetic peripheral neuropathy

Sensory peripheral neuropathy is a recognized complication of diabetes mellitus. However, neuropathy in diabetic patients most commonly manifests initially as distal symmetrical ascending neuropathy starting in the lower extremities.18 Therefore, diabetic peripheral neuropathy is less likely in this patient since her symptoms are limited to her hands.

Vitamin B12 deficiency

Metformin-induced vitamin B12 deficiency is another possible cause of peripheral neuropathy. It might be secondary to metformin-induced changes in intrinsic factor levels and small-intestine motility with resultant bacterial overgrowth, as well as inhibition of vitamin B12 absorption in the terminal ileum.19

However, metformin-induced vitamin B12 deficiency is not the most likely cause of our patient’s neuropathy, since she has been taking this drug for only 1 year. Vitamin B12 deficiency with consequent peripheral neuropathy is more likely in patients taking metformin in high doses for 10 or more years.20

Laboratory results and electrocardiography

Results of initial laboratory testing

Table 2 shows the patient’s initial laboratory results. Of note, her serum calcium level is 5.7 mg/dL (reference range 8.9–10.1). Electrocardiography in the emergency department shows:

  • Prolonged PR interval (23 msec)
  • Wide QRS complexes (13 msec)
  • Flat T waves
  • Prolonged corrected QT interval (475 msec)
  • Occasional premature ventricular complexes.

CLINICAL MANIFESTATIONS OF HYPOCALCEMIA

2. Which of the following is not a manifestation of hypocalcemia?

  • Tonic-clonic seizures
  • Cyanosis
  • Cardiac ventricular arrhythmias
  • Acute pancreatitis
  • Depression

 Clinical manifestations of hypocalcemia

Hypocalcemia can cause a wide range of clinical manifestations (Table 3), the extent and severity of which depend on the severity of hypocalcemia and how quickly it develops. The more acute the hypocalcemia, the more severe the manifestations.21

Tetany can cause seizures

Hypocalcemia is characterized by neuromuscular hyperexcitability, manifested clinically by tetany.22 Manifestations of tetany are numerous and include acral paresthesia, perioral numbness, muscle cramps, carpopedal spasm, and seizures. Tetany is the hallmark of hypocalcemia regardless of etiology. However, certain causes are associated with peculiar clinical manifestations. For example, chronic primary hypoparathyroidism may be associated with basal ganglia calcifications that can result in parkinsonism, other extrapyramidal disorders, and dementia (Table 4).6

Clinical manifestations of chronic primary hypoparathyroidism

Airway spasm can be fatal

A serious manifestation of acute severe hypocalcemia is spasm of the glottis muscles, which may cause cyanosis and, if untreated, death.21

Ventricular arrhythmias

Another potential fatal complication of acute severe hypocalcemia is polymorphic ventricular tachycardia due to prolongation of the QT interval, which is readily identified with electrocardiography.23

Hypocalcemia does not cause pancreatitis

Hypercalcemia, rather than hypocalcemia, may cause acute pancreatitis.24 Conversely, acute pancreatitis may cause hypocalcemia due to precipitation of calcium in the abdominal cavity.25

Psychiatric manifestations

In addition to depression, hypocalcemia is associated with psychiatric manifestations including anxiety, confusion, and emotional instability.

 

 

STEPS TO DIAGNOSIS OF HYPOCALCEMIA

First step: Confirm true hypocalcemia

Calcium circulates in the blood in 3 forms: bound to albumin (40% to 45%), bound to anions (10% to 15%), and free (ionized) (45%). Although ionized calcium is the active form, most laboratories report total serum calcium.

Since changes in serum albumin concentration affect the total serum calcium level, it is imperative to correct the measured serum calcium to the serum albumin concentration. Each 1-g/dL decrease in serum albumin lowers the total serum calcium by 0.8 mg/dL. Thus:

Corrected serum calcium (mg/dL) =
measured total serum calcium (mg/dL) +
0.8 (4 − serum albumin [g/dL])
.

If the patient’s serum calcium level remains low when corrected for serum albumin, he or she has true hypocalcemia, which implies a low ionized serum calcium. Conversely, pseudohypocalcemia means that the measured calcium level is low but the corrected serum calcium is normal.

Using this formula, our patient’s corrected calcium level is calculated as 5.7 + 0.8 (4 – 3.2) = 6.3 mg/dL, indicating true hypocalcemia.

PHOSPHATE IS OFTEN HIGH WHEN CALCIUM IS LOW

In addition to hypocalcemia, our patient has an elevated phosphate level (Table 2).

3. Which of the following hypocalcemic disorders is not associated with hyperphosphatemia?

  • End-stage renal disease
  • Primary hypoparathyroidism
  • Pseudohypoparathyroidism
  • Vitamin D3 deficiency
  • Rhabdomyolysis

Vitamin D deficiency is not associated with hyperphosphatemia.

Second step in evaluating hypocalcemia: Check phosphate, magnesium, creatinine

Major causes of hypocalcemia, according to phosphate level

The major causes of hypocalcemia can be categorized according to the serum phosphate level: high vs normal or low (Table 5).

High-phosphate, low-calcium states. In the absence of concurrent end-stage renal disease and an excessive phosphate load, primary hypoparathyroidism is the most likely cause of hypocalcemia associated with hyperphosphatemia.

PTH increases serum ionized calcium by26,27:

  • Increasing bone resorption
  • Increasing reabsorption of calcium from the distal renal tubules
  • Increasing the activity of 1-alpha-hydroxylase, responsible for conversion of 25-hydroxyvitamin D3 to 1,25-dihydroxyvitamin D3 (the most biologically active vitamin D metabolite); 1,25-dihydroxyvitamin D increases the absorption of calcium and phosphate from the intestine.

Conversely, PTH decreases reabsorption of phosphate from proximal renal tubules, resulting in hypophosphatemia. Therefore, low serum PTH (primary hypoparathyroidism) or a PTH-resistant state (pseudohypoparathyroidism) results in hypocalcemia and hyperphosphatemia.26,27

Both end-stage renal disease and rhabdomyolysis are associated with high serum phosphate levels. The kidney normally excretes excess dietary phosphate to maintain phosphate homeostasis; however, this is impaired in end-stage renal disease, leading to hyperphosphatemia. In rhabdomyolysis, it is mainly the transcellular shift of phosphate into the extracellular space from myocyte injury that raises phosphate levels.

Normal- or low-phosphate, low calcium states. Hypocalcemia can also result from vitamin D deficiency, but this cause is associated with a low or normal serum phosphate level. In such cases, hypocalcemia causes secondary hyperparathyroidism with consequent renal phosphate loss and, thus, hypophosphatemia.27

Third step: Check serum intact PTH and 25-hydroxyvitamin D levels

Hypocalcemia stimulates secretion of PTH. Therefore, hypocalcemia with elevated serum PTH is caused by disorders that do not impair PTH secretion, including chronic renal failure and vitamin D deficiency (Table 5). Conversely, hypocalcemia with low or normal serum PTH levels suggests primary hypoparathyroidism.

Our patient’s serum PTH level is 20 ng/mL, which is within the reference range. This does not discount the diagnosis of primary hypoparathyroidism. Although most patients with primary hypoparathyroidism have low or undetectable serum PTH levels, some have normal PTH levels if some degree of PTH production is preserved.5,7,28–30 In these patients, the remaining functioning parathyroid tissue is not enough to maintain a normal serum calcium level, resulting in hypocalcemia. As a result, hypocalcemia stimulates the remaining parathyroid tissue to its maximum output, producing PTH levels usually within the lower or middle-normal range.30 In such patients, the terms parathyroid insufficiency and relative primary hypoparathyroidism are more precise than primary hypoparathyroidism.

Postsurgical hypoparathyroidism with an inappropriately normal PTH level is usually seen in patients with disorders that impair intestinal calcium absorption or bone resorption.31 In our patient’s case, the “normal” serum PTH level is likely due to maximal stimulation of remaining functioning parathyroid tissue by severe hypocalcemia, which is a result of her discontinuation of calcium and calcitriol therapy and her vitamin D deficiency.

 

 

CASE RESUMED: NO RESPONSE TO INTRAVENOUS CALCIUM GLUCONATE

The patient is given 2 10-mL ampules of 10% calcium gluconate diluted in 100 mL of 5% dextrose in water over 20 minutes intravenously. Electrocardiographic monitoring is continued. Two hours later, her measured serum calcium is only 5.8 mg/dL, with no improvement in her symptoms.

A continuous infusion of calcium gluconate is started: 12 ampules of calcium gluconate are added to 380 mL of 5% dextrose in water and infused at 40 mL/hour (infused rate of elemental calcium = 1.3 mg/kg/hour); 3 hours later, her measured serum calcium level is still only 5.8 mg//dL; at 6 hours it is 5.9 mg/dL, and her symptoms have not improved.

4. Which of the following is the most appropriate next step?

  • Change the calcium gluconate to calcium chloride
  • Increase the infusion rate to 1.5 mg of elemental calcium/kg/hour
  • Give a bolus of 2 10-mL ampules of 10% calcium gluconate intravenously over 1 minute
  • Give additional oral calcium tablets
  • Check the serum magnesium level

Treatment of hypocalcemia can involve intravenous or oral calcium therapy.

Intravenous calcium is indicated for patients with any of the following6,32:

  • Moderate to severe neuromuscular irritability (eg, acral paresthesia, carpopedal spasm, prolonged QT interval, seizures, laryngospasm, bronchospasm)
  • Acute hypocalcemia with corrected serum calcium level less than 7.6 mg/dL, even if the patient is asymptomatic
  • Cardiac failure.

One 10-mL ampule of 10% calcium gluconate contains 93 mg of elemental calcium; 1 or 2 ampules are typically diluted in 50 to 100 mL of 5% dextrose in water and infused slowly over 15 to 20 minutes. Rapid administration of intravenous calcium is contraindicated, as it may produce cardiac arrhythmias and possibly cardiac arrest. Therefore, intravenous calcium should be given slowly while continuing electrocardiographic monitoring.33

Since the effect of 1 ampule of calcium gluconate lasts only 2 to 3 hours, most patients with symptomatic hypocalcemia require continuous intravenous calcium infusion. The recommended dose of infused elemental calcium is 0.5 to 1.5 mg/kg/hour.34 Several ampules are added to 500 to 1,000 mL of 5% dextrose in water or 0.9% normal saline and infused at a rate appropriate for the patient’s corrected calcium and symptoms.

Oral calcium and vitamin D supplements can be given initially to patients with a corrected serum calcium level of 7.6 mg/dL or greater, with or without mild symptoms, if they can tolerate oral intake. However, this is not the treatment of choice for resistant acute hypocalcemia, as in this case.

Calcium chloride has no advantages over calcium gluconate. Further, it can be associated with local irritation and may result in tissue necrosis if extravasation occurs.35

Increasing the infusion rate of calcium gluconate to the maximum recommended dose may improve the patient’s ionized calcium level and symptoms somewhat. However, it is not the best option for this patient, given that she did not respond to 2 ampules of calcium gluconate followed by continuous infusion of 1.3 mg/kg/hour for 6 hours.

Calcium gluconate bolus. Similarly, giving the patient an additional 2 ampules of calcium gluconate over 1 minute would not be recommended, as rapid administration of intravenous calcium gluconate (eg, over 1 minute) is contraindicated.

Check magnesium

If hypocalcemia persists despite intravenous calcium therapy, as in our patient, further investigation or action is required. An important cause of persistent hypocalcemia is severe hypomagnesemia. Severe hypomagnesemia (serum magnesium < 0.8 mg/dL) causes resistant hypocalcemia by several mechanisms:

  • Inducing PTH resistance32,36,37
  • Decreasing PTH secretion32,36
  • Decreasing calcitriol production.

The decrease in calcitriol production is a direct effect of hypomagnesemia, but it is also an indirect effect of low PTH secretion, which inhibits the enzyme 1-alpha-hydroxylase. Thus, conversion of 25-hydroxyvitamin D3 to calcitriol is impaired, leading to low calcitriol production.

Our patient could have hypomagnesemia due to furosemide use and uncontrolled diabetes mellitus. Hypocalcemia resistant to calcium therapy may occasionally respond to magnesium therapy even if the serum magnesium level is normal. This may be due to depleted intracellular magnesium salt levels.6,38 Rarely, severe hypermagnesemia can also be associated with hypocalcemia due to inhibition of PTH secretion.37,39

CASE RESUMED

Our patient’s serum magnesium level is 0.6 mg/dL (reference range 1.7–2.4 mg/dL). She is given 2 g of magnesium sulfate in 60 mL of 0.9% normal saline infused over 1 hour, followed by a continuous infusion of magnesium sulfate (12 g diluted in 250 mL of 0.9% normal saline, infused over 24 hours). On repeat testing 4 hours later, her serum magnesium level is 0.7 mg/dL, and at 8 hours later it is 0.9 mg/dL. She is subsequently started on oral magnesium oxide 600 mg per day. The magnesium sulfate infusion is continued for another 24 hours.

PREVENTING HYPERCALCIURIA

Patients with low PTH (primary hypoparathyroidism) may have hypercalciuria due to decreased renal tubular calcium reabsorption. Two important measures can minimize hypercalciuria in such patients:

  • Keeping the serum calcium level in the low-normal range4,5,40
  • Giving a thiazide diuretic (eg, hydrochlorothiazide 12.5–50 mg daily) with a low-salt diet.41,42

A thiazide diuretic is usually started once the 24-hour urine calcium reaches 250 mg.6 Thiazides are thought to enhance both proximal and distal renal tubular calcium reabsorption.43,44

PRIMARY HYPOPARATHYROIDISM: LONG-TERM MANAGEMENT

Long-term management of primary hypoparathyroidism requires calcium and vitamin D supplementation.

Calcium supplements. The most commonly prescribed calcium preparations are calcium carbonate and calcium citrate (containing 40% and 20% elemental calcium, respectively). Calcium carbonate, which is less expensive than calcium citrate, binds with phosphate intake and requires an acidic environment for absorption, and so it is better absorbed when taken with meals. Because calcium citrate does not require an acidic environment for absorption, it is the calcium preparation of choice for patients on proton pump inhibitors, or patients with achlorhydria or constipation.45 Calcium doses vary widely, with most hypoparathyroid patients requiring 1 to 2 g of elemental calcium daily.6

Vitamin D supplements. To promote intestinal absorption, calcium is combined with vitamin D in a fixed-dose preparation given in divided doses.46 Calcitriol (1,25-dihydroxyvitamin D3) is the most active metabolite of vitamin D, with rapid onset and offset of action, and it is the preferred form of vitamin D therapy for patients with hypoparathyroidism. If calcitriol is not available or is not affordable, alphacalcidol (1-alpha-hydroxyvitamin D3) is another option. This is a synthetic analogue of vitamin D that is already hyroxylated at the C1 position. After oral intake, it is hydroxylated in the liver to form calcitriol.

Since renal production of calcitriol is PTH-dependent, in hypoparathyroidism the conversion of 25-hydroxyvitamin D3 to calcitriol is limited. Therefore, vitamin D3 (cholecalciferol) and vitamin D2 (ergocalciferol) are not the preferred forms of vitamin D for such patients. However, either can be added to calcitriol, as they may have extraskeletal benefits.7

CASE CONCLUDED

Our patient presented with primary parathyroid insufficiency associated with vitamin D deficiency. Therefore, in addition to calcitriol and calcium combined with vitamin D in a fixed-dose preparation, her management included vitamin D3 for her vitamin D deficiency.

She was discharged on these medications. At a follow-up visit 3 weeks later, her measured serum calcium level was 8.6 mg/dL. She reported gradual resolution of her symptoms. She was also referred to a psychiatrist for her depression.

TAKE-HOME POINTS

  • Hypocalcemia causes neuromuscular excitability, manifested clinically by tetany.
  • Common causes of hypocalcemia include vitamin D deficiency, hypomagnesemia, renal failure, and primary hypoparathyroidism.
  • The first step in evaluating hypocalcemia is to correct the measured serum calcium to the serum albumin concentration.
  • Laboratory testing for hypocalcemia should include serum phosphorus, magnesium, creatinine, PTH, and 25-hydroxyvitamin D3.
  • Primary hypoparathyroidism is characterized by hypocalcemia, hyperphosphatemia, and low serum PTH.
  • Moderate to severe manifestations of hypo-
    calcemia and acute hypocalcemia (< 7.6 mg/dL), even if asymptomatic, warrant intravenous calcium therapy.
  • Correction of hypomagnesemia is essential to treat hypocalcemia, especially if resistant to intravenous calcium therapy.
  • The goal of chronic management of primary hypoparathyroidism includes correcting the serum calcium level to a low-normal range, the serum phosphorus level to an upper-normal range, and prevention of hypercalciuria.

Acknowledgments: The authors wish to thank Mr. Michael Edward Tierney of the School of Medicine, University of Sydney, Australia, for his linguistic editing of the manuscript.

References
  1. Jesus JE, Landry A. Images in clinical medicine. Chvostek’s and Trousseau’s signs. N Engl J Med 2012; 367:e15.
  2. Urbano FL. Signs of hypocalcemia: Chvostek’s and Trousseau’s. Hosp Physician 2000; 36:43–45.
  3. Chisthi MM, Nair RS, Kuttanchettiyar KG, Yadev I. Mechanisms behind post-thyroidectomy hypocalcemia: interplay of calcitonin, parathormone, and albumin—a prospective study. J Invest Surg 2017; 30:217–225.
  4. Shoback DM, Bilezikian JP, Costa AG, et al. Presentation of hypoparathyroidism: etiologies and clinical features. J Clin Endocrinol Metab 2016; 101:2300–2312.
  5. Stack BC Jr, Bimston DN, Bodenner DL, et al. American Association of Clinical Endocrinologists and American College of Endocrinology disease state clinical review: postoperative hypoparathyroidism—definitions and management. Endocr Pract 2015; 21:674–685.
  6. Shoback D. Clinical practice. Hypoparathyroidism. N Engl J Med 2008; 359:391–403.
  7. Abate EG, Clarke BL. Review of hypoparathyroidism. Front Endocrinol (Lausanne) 2017; 7:172.
  8. Coimbra C, Monteiro F, Oliveira P, Ribeiro L, de Almeida MG, Condé A. Hypoparathyroidism following thyroidectomy: predictive factors. Acta Otorrinolaringol Esp 2017; 68:106–111.
  9. Thomusch O, Machens A, Sekulla C, Ukkat J, Brauckhoff M, Dralle H. The impact of surgical technique on postoperative hypoparathyroidism in bilateral thyroid surgery: a multivariate analysis of 5846 consecutive patients. Surgery 2003; 133:180–185.
  10. Hirsch PF, Lester GE, Talmage RV. Calcitonin, an enigmatic hormone: does it have a function? J Musculoskelet Neuronal Interact 2001; 1:299–305.
  11. Karne SS, Bhalerao NS. Carpal tunnel syndrome in hypothyroidism. J Clin Diagn Res 2016; 10:OC36–OC38.
  12. Duyff RF, Van den Bosch J, Laman DM, van Loon BJ, Linssen WH. Neuromuscular findings in thyroid dysfunction: a prospective clinical and electrodiagnostic study. J Neurol Neurosurg Psychiatry 2000; 68:750–755.
  13. Palumbo CF, Szabo RM, Olmsted SL. The effects of hypothyroidism and thyroid replacement on the development of carpal tunnel syndrome. J Hand Surg Am 2000; 25:734–739.
  14. Katz JN, Stirrat CR, Larson MG, Fossel AH, Eaton HM, Liang MH. A self-administered hand symptom diagram for the diagnosis and epidemiologic study of carpal tunnel syndrome. J Rheumatol 1990; 17:1495–1498.
  15. Katz JN, Stirrat CR. A self-administered hand diagram for the diagnosis of carpal tunnel syndrome. J Hand Surg Am 1990; 15:360–363.
  16. Calfee RP, Dale AM, Ryan D, Descatha A, Franzblau A, Evanoff B. Performance of simplified scoring systems for hand diagrams in carpal tunnel syndrome screening. J Hand Surg Am 2012; 37:10–17.
  17. D’Arcy CA, McGee S. The rational clinical examination. Does this patient have carpal tunnel syndrome? JAMA 2000; 283:3110–3117.
  18. Marchettini P, Lacerenza M, Mauri E, Marangoni C. Painful peripheral neuropathies. Curr Neuropharmacol 2006; 4:175–181.
  19. Kibirige D, Mwebaze R. Vitamin B12 deficiency among patients with diabetes mellitus: is routine screening and supplementation justified? J Diabetes Metab Disord 2013;12:17.
  20. Akinlade KS, Agbebaku SO, Rahamon SK, Balogun WO. Vitamin B12 levels in patients with type 2 diabetes mellitus on metformin. Ann Ib Postgrad Med 2015; 13:79–83.
  21. Tohme JF, Bilezikian JP. Hypocalcemic emergencies. Endocrinol Metab Clin North Am 1993; 22:363–375.
  22. Macefield G, Burke D. Paraesthesiae and tetany induced by voluntary hyperventilation. Increased excitability of human cutaneous and motor axons. Brain 1991; 114:527–540.
  23. Benoit SR, Mendelsohn AB, Nourjah P, Staffa JA, Graham DJ. Risk factors for prolonged QTc among US adults: Third National Health and Nutrition Examination Survey. Eur J Cardiovasc Prev Rehabil 2005; 12:363–368.
  24. Khoo TK, Vege SS, Abu-Lebdeh HS, Ryu E, Nadeem S, Wermers RA. Acute pancreatitis in primary hyperparathyroidism: a population-based study. J Clin Endocrinol Metab 2009; 94:2115–2118.
  25. McKay C, Beastall GH, Imrie CW, Baxter JN. Circulating intact parathyroid hormone levels in acute pancreatitis. Br J Surg 1994; 81:357–360.
  26. Talmage RV, Mobley HT. Calcium homeostasis: reassessment of the actions of parathyroid hormone. Gen Comp Endocrinol 2008; 156:1–8.
  27. Friedman PA, Gesek FA. Calcium transport in renal epithelial cells. Am J Physiol 1993; 264:F181–F198.
  28. Jensen PV, Jelstrup SM, Homøe P. Long-term outcomes after total thyroidectomy. Dan Med J 2015; 62:A5156.
  29. Ritter K, Elfenbein D, Schneider DF, Chen H, Sippel RS. Hypoparathyroidism after total thyroidectomy: incidence and resolution. J Surg Res 2015; 197:348–353.
  30. Promberger R, Ott J, Kober F, Karik M, Freissmuth M, Hermann M. Normal parathyroid hormone levels do not exclude permanent hypoparathyroidism after thyroidectomy. Thyroid 2011; 21:145–150.
  31. Lorente-Poch L, Sancho JJ, Muñoz-Nova JL, Sánchez-Velázquez P, Sitges-Serra A. Defining the syndromes of parathyroid failure after total thyroidectomy. Gland Surgery 2015; 4:82–90.
  32. Cooper MS, Gittoes NJ. Diagnosis and management of hypocalcaemia. BMJ 2008; 336:1298–1302.
  33. Tohme JF, Bilezikian JP. Diagnosis and treatment of hypocalcemic emergencies. Endocrinologist 1996; 6:10–18.
  34. Carroll R, Matfin G. Endocrine and metabolic emergencies: hypocalcaemia. Ther Adv Endocrinol Metab 2010; 1:29–33.
  35. Kim MP, Raho VJ, Mak J, Kaynar AM. Skin and soft tissue necrosis from calcium chloride in a deicer. J Emerg Med 2007; 32:41–44.
  36. Tong GM, Rude RK. Magnesium deficiency in critical illness. J Intensive Care Med 2005; 20:3–17.
  37. Cholst IN, Steinberg SF, Tropper PJ, Fox HE, Segre GV, Bilezikian JP. The influence of hypermagnesemia on serum calcium and parathyroid hormone levels in human subjects. N Engl J Med 1984; 310:1221–1225.
  38. Ryzen E, Nelson TA, Rude RK. Low blood mononuclear cell magnesium content and hypocalcemia in normomagnesemic patients. West J Med 1987; 147:549–553.
  39. Koontz SL, Friedman SA, Schwartz ML. Symptomatic hypocalcemia after tocolytic therapy with magnesium sulfate and nifedipine. Am J Obstet Gynecol 2004; 190:1773–1776.
  40. Brandi ML, Bilezikian JP, Shoback D, et al. Management of hypoparathyroidism: summary statement and guidelines. J Clin Endocrinol Metab 2016; 101:2273–2283.
  41. Porter RH, Cox BG, Heaney D, Hostetter TH, Stinebaugh BJ, Suki WN. Treatment of hypoparathyroid patients with chlorthalidone. N Engl J Med 1978; 298:577–581.
  42. Clarke BL, Brown EM, Collins MT, et al. Epidemiology and diagnosis of hypoparathyroidism. J Clin Endocrinol Metab 2016; 101:2284–2299.
  43. Nijenhuis T, Vallon V, van der Kemp AW, Loffing J, Hoenderop JG, Bindels RJ. Enhanced passive Ca2+ reabsorption and reduced Mg2+ channel abundance explains thiazide-induced hypocalciuria and hypomagnesemia. J Clin Invest 2005; 115:1651–1658.
  44. Costanzo LS. Localization of diuretic action in microperfused rat distal tubules: Ca and Na transport. Am J Physiol 1985; 248:F527–F535.
  45. Brandi ML, Bilezikian JP, Shoback D, et al. Management of hypoparathyroidism: summary statement and guidelines. J Clin Endocrinol Metab 2016; 101:2273–2283.
  46. Scotti A, Bianchini C, Abbiati G, Marzo A. Absorption of calcium administered alone or in fixed combination with vitamin D to healthy volunteers. Arzneimittelforschung 2001; 51:493–500.
References
  1. Jesus JE, Landry A. Images in clinical medicine. Chvostek’s and Trousseau’s signs. N Engl J Med 2012; 367:e15.
  2. Urbano FL. Signs of hypocalcemia: Chvostek’s and Trousseau’s. Hosp Physician 2000; 36:43–45.
  3. Chisthi MM, Nair RS, Kuttanchettiyar KG, Yadev I. Mechanisms behind post-thyroidectomy hypocalcemia: interplay of calcitonin, parathormone, and albumin—a prospective study. J Invest Surg 2017; 30:217–225.
  4. Shoback DM, Bilezikian JP, Costa AG, et al. Presentation of hypoparathyroidism: etiologies and clinical features. J Clin Endocrinol Metab 2016; 101:2300–2312.
  5. Stack BC Jr, Bimston DN, Bodenner DL, et al. American Association of Clinical Endocrinologists and American College of Endocrinology disease state clinical review: postoperative hypoparathyroidism—definitions and management. Endocr Pract 2015; 21:674–685.
  6. Shoback D. Clinical practice. Hypoparathyroidism. N Engl J Med 2008; 359:391–403.
  7. Abate EG, Clarke BL. Review of hypoparathyroidism. Front Endocrinol (Lausanne) 2017; 7:172.
  8. Coimbra C, Monteiro F, Oliveira P, Ribeiro L, de Almeida MG, Condé A. Hypoparathyroidism following thyroidectomy: predictive factors. Acta Otorrinolaringol Esp 2017; 68:106–111.
  9. Thomusch O, Machens A, Sekulla C, Ukkat J, Brauckhoff M, Dralle H. The impact of surgical technique on postoperative hypoparathyroidism in bilateral thyroid surgery: a multivariate analysis of 5846 consecutive patients. Surgery 2003; 133:180–185.
  10. Hirsch PF, Lester GE, Talmage RV. Calcitonin, an enigmatic hormone: does it have a function? J Musculoskelet Neuronal Interact 2001; 1:299–305.
  11. Karne SS, Bhalerao NS. Carpal tunnel syndrome in hypothyroidism. J Clin Diagn Res 2016; 10:OC36–OC38.
  12. Duyff RF, Van den Bosch J, Laman DM, van Loon BJ, Linssen WH. Neuromuscular findings in thyroid dysfunction: a prospective clinical and electrodiagnostic study. J Neurol Neurosurg Psychiatry 2000; 68:750–755.
  13. Palumbo CF, Szabo RM, Olmsted SL. The effects of hypothyroidism and thyroid replacement on the development of carpal tunnel syndrome. J Hand Surg Am 2000; 25:734–739.
  14. Katz JN, Stirrat CR, Larson MG, Fossel AH, Eaton HM, Liang MH. A self-administered hand symptom diagram for the diagnosis and epidemiologic study of carpal tunnel syndrome. J Rheumatol 1990; 17:1495–1498.
  15. Katz JN, Stirrat CR. A self-administered hand diagram for the diagnosis of carpal tunnel syndrome. J Hand Surg Am 1990; 15:360–363.
  16. Calfee RP, Dale AM, Ryan D, Descatha A, Franzblau A, Evanoff B. Performance of simplified scoring systems for hand diagrams in carpal tunnel syndrome screening. J Hand Surg Am 2012; 37:10–17.
  17. D’Arcy CA, McGee S. The rational clinical examination. Does this patient have carpal tunnel syndrome? JAMA 2000; 283:3110–3117.
  18. Marchettini P, Lacerenza M, Mauri E, Marangoni C. Painful peripheral neuropathies. Curr Neuropharmacol 2006; 4:175–181.
  19. Kibirige D, Mwebaze R. Vitamin B12 deficiency among patients with diabetes mellitus: is routine screening and supplementation justified? J Diabetes Metab Disord 2013;12:17.
  20. Akinlade KS, Agbebaku SO, Rahamon SK, Balogun WO. Vitamin B12 levels in patients with type 2 diabetes mellitus on metformin. Ann Ib Postgrad Med 2015; 13:79–83.
  21. Tohme JF, Bilezikian JP. Hypocalcemic emergencies. Endocrinol Metab Clin North Am 1993; 22:363–375.
  22. Macefield G, Burke D. Paraesthesiae and tetany induced by voluntary hyperventilation. Increased excitability of human cutaneous and motor axons. Brain 1991; 114:527–540.
  23. Benoit SR, Mendelsohn AB, Nourjah P, Staffa JA, Graham DJ. Risk factors for prolonged QTc among US adults: Third National Health and Nutrition Examination Survey. Eur J Cardiovasc Prev Rehabil 2005; 12:363–368.
  24. Khoo TK, Vege SS, Abu-Lebdeh HS, Ryu E, Nadeem S, Wermers RA. Acute pancreatitis in primary hyperparathyroidism: a population-based study. J Clin Endocrinol Metab 2009; 94:2115–2118.
  25. McKay C, Beastall GH, Imrie CW, Baxter JN. Circulating intact parathyroid hormone levels in acute pancreatitis. Br J Surg 1994; 81:357–360.
  26. Talmage RV, Mobley HT. Calcium homeostasis: reassessment of the actions of parathyroid hormone. Gen Comp Endocrinol 2008; 156:1–8.
  27. Friedman PA, Gesek FA. Calcium transport in renal epithelial cells. Am J Physiol 1993; 264:F181–F198.
  28. Jensen PV, Jelstrup SM, Homøe P. Long-term outcomes after total thyroidectomy. Dan Med J 2015; 62:A5156.
  29. Ritter K, Elfenbein D, Schneider DF, Chen H, Sippel RS. Hypoparathyroidism after total thyroidectomy: incidence and resolution. J Surg Res 2015; 197:348–353.
  30. Promberger R, Ott J, Kober F, Karik M, Freissmuth M, Hermann M. Normal parathyroid hormone levels do not exclude permanent hypoparathyroidism after thyroidectomy. Thyroid 2011; 21:145–150.
  31. Lorente-Poch L, Sancho JJ, Muñoz-Nova JL, Sánchez-Velázquez P, Sitges-Serra A. Defining the syndromes of parathyroid failure after total thyroidectomy. Gland Surgery 2015; 4:82–90.
  32. Cooper MS, Gittoes NJ. Diagnosis and management of hypocalcaemia. BMJ 2008; 336:1298–1302.
  33. Tohme JF, Bilezikian JP. Diagnosis and treatment of hypocalcemic emergencies. Endocrinologist 1996; 6:10–18.
  34. Carroll R, Matfin G. Endocrine and metabolic emergencies: hypocalcaemia. Ther Adv Endocrinol Metab 2010; 1:29–33.
  35. Kim MP, Raho VJ, Mak J, Kaynar AM. Skin and soft tissue necrosis from calcium chloride in a deicer. J Emerg Med 2007; 32:41–44.
  36. Tong GM, Rude RK. Magnesium deficiency in critical illness. J Intensive Care Med 2005; 20:3–17.
  37. Cholst IN, Steinberg SF, Tropper PJ, Fox HE, Segre GV, Bilezikian JP. The influence of hypermagnesemia on serum calcium and parathyroid hormone levels in human subjects. N Engl J Med 1984; 310:1221–1225.
  38. Ryzen E, Nelson TA, Rude RK. Low blood mononuclear cell magnesium content and hypocalcemia in normomagnesemic patients. West J Med 1987; 147:549–553.
  39. Koontz SL, Friedman SA, Schwartz ML. Symptomatic hypocalcemia after tocolytic therapy with magnesium sulfate and nifedipine. Am J Obstet Gynecol 2004; 190:1773–1776.
  40. Brandi ML, Bilezikian JP, Shoback D, et al. Management of hypoparathyroidism: summary statement and guidelines. J Clin Endocrinol Metab 2016; 101:2273–2283.
  41. Porter RH, Cox BG, Heaney D, Hostetter TH, Stinebaugh BJ, Suki WN. Treatment of hypoparathyroid patients with chlorthalidone. N Engl J Med 1978; 298:577–581.
  42. Clarke BL, Brown EM, Collins MT, et al. Epidemiology and diagnosis of hypoparathyroidism. J Clin Endocrinol Metab 2016; 101:2284–2299.
  43. Nijenhuis T, Vallon V, van der Kemp AW, Loffing J, Hoenderop JG, Bindels RJ. Enhanced passive Ca2+ reabsorption and reduced Mg2+ channel abundance explains thiazide-induced hypocalciuria and hypomagnesemia. J Clin Invest 2005; 115:1651–1658.
  44. Costanzo LS. Localization of diuretic action in microperfused rat distal tubules: Ca and Na transport. Am J Physiol 1985; 248:F527–F535.
  45. Brandi ML, Bilezikian JP, Shoback D, et al. Management of hypoparathyroidism: summary statement and guidelines. J Clin Endocrinol Metab 2016; 101:2273–2283.
  46. Scotti A, Bianchini C, Abbiati G, Marzo A. Absorption of calcium administered alone or in fixed combination with vitamin D to healthy volunteers. Arzneimittelforschung 2001; 51:493–500.
Issue
Cleveland Clinic Journal of Medicine - 85(3)
Issue
Cleveland Clinic Journal of Medicine - 85(3)
Page Number
200-208
Page Number
200-208
Publications
Publications
Topics
Article Type
Display Headline
A 67-year-old woman with bilateral hand numbness
Display Headline
A 67-year-old woman with bilateral hand numbness
Legacy Keywords
numbness, paresthesia, hypocalcemia, calcium, Trousseau sign, Chvostek sign, hypoparathyroidism, carpal tunnel syndrome, hypothyroidism, diabetic peripheral neuropathy, vitamin B12 deficiency, parathyroid, parathyroid hormone, PTH, thyroid surgery, electrolytes, Sohab Radwan, Khair Hamo, Ayman Zayed
Legacy Keywords
numbness, paresthesia, hypocalcemia, calcium, Trousseau sign, Chvostek sign, hypoparathyroidism, carpal tunnel syndrome, hypothyroidism, diabetic peripheral neuropathy, vitamin B12 deficiency, parathyroid, parathyroid hormone, PTH, thyroid surgery, electrolytes, Sohab Radwan, Khair Hamo, Ayman Zayed
Sections
Disallow All Ads
Content Gating
No Gating (article Unlocked/Free)
Alternative CME
Disqus Comments
Default
Gate On Date
Tue, 02/27/2018 - 08:30
Un-Gate On Date
Tue, 02/27/2018 - 08:30
Use ProPublica
Hide sidebar & use full width
render the right sidebar.
Article PDF Media